board vitals part II

Ace your homework & exams now with Quizwiz!

During contact phase of the gait cycle, what are the position and motion observed at the pelvis? A. 0-2 degrees in internal, and 4-5 degrees internal rotation B. 4-5 degrees internal and 4-5 degrees internal rotation C. 0-2 degrees in external and 4-5 internal rotation D. 0 degrees in external and 4-5 degrees on internal rotation

Correct answer: (A) 0-2 degrees in internal, and 4-5 degrees internal rotation. Explanation: The motion at the pelvis is minimal so as to stabilize the trunk over the limb. position motion pelvis-0-2 degree internal 4-5 degree internal femur-0 degree neutral 6 degree internal tibia - 2 degree internal 9 degree internal

The "Mayo Block" is effective for providing local anesthesia to the: A. 1st metatarsal B. 5th metatarsal C. Midfoot D. Calcaneus

Correct answer: (A) 1st metatarsal. Explanation: The Mayo block provides local anesthesia to the forefoot for surgical correction of hallux deformity, bunionectomy, and first metatarsal surgery.

Iris and Mayo instruments are types of: A. Blades B. Scissors C. Forceps D. Pin cutters

Correct: B Explanation: Both instruments are scissors. The fine cutting blades of the lightweight iris scissors are suited only for delicate tissue dissection. Heavier blunt-tipped Mayo scissors are used for cutting thick bands of fascia and ligament; they are available with either straight or curved blades.

A patient presents with pain to the lesser digits, involving the distal phalanx which is plantarflexed upon the middle phalanx, what is your diagnosis? A. Hammertoe B. Mallet toe C. Claw toe D. Flail toe

Explanation: Mallet toe is deformity of the distal interphalangeal joint (DIPJ) in which the distal phalanx is plantarflexed upon the middle phalanx. (A) A hammertoe is defined by dorsiflexion of the MPJ, plantarflexion of the PIPJ, and hyperexetension of the DIPJ. The patient will present with pain and tenderness on the dorsal PIPJ, typically due to pressure from the shoe. This will lead to an extension of the MTPJ and eventually a plantar callus under the corresponding metatarsal head. (C) Claw toe is a deformity involving the dorsiflexion of the MTPJ, and plantarflexion of both PIPJ and DIPJ. (D) Flail toe is a bothersome complication following PIP resection arthroplasty, usually as a result of excessive bone resection. Patients complain of toe instability and catching on socks or shoes.

What are the minimum degrees needed at the midtarsal joint to maintain the foot plantigrade in contact phase? A. 4 to 6 degrees of longitudinal axis motion with the STJ pronated B. 4 to 6 degrees oblique axis motion with the STJ supinated C. 8 to 10 degrees with the STJ pronated D. 8 to 10 degrees with the STJ supinated

Correct answer: (A) 4 to 6 degrees of longitudinal axis motion with the STJ pronated. Explanation: A minimum of 4-to- 6 degrees of longitudinal axis of motion in gait is needed to maintain the foot plantigrade when the subtalar joint pronates to that degree during the contact phase.

In a patient with a history of glucocorticoid use and a high suspicion of hypothalamic-pituitary- adrenal axis insufficiency, how long should steroids be continued post-operatively? A. 48 hours B. Taper for the next 5 days C. 72 hours D. 24 hours

Correct answer: (A) 48 hours. Explanation: Even if perioperative ACTH stimulation is normal, and the patient is at high risk for perioperative adrenal insufficiency, if unexplained hypotension persists despite volume repletion, steroids should be administered in a dose consistent with the level of injury. Postoperatively, steroids should be continued until the stress response diminishes, usually 48 hours.

A UCBL (University of California Biomechanical Laboratory) device is best indicated for: A. A pediatric flexible deformity B. A pediatric rigid deformity C. A pediatric deformity requiring sagittal plane correction D. Adult or pediatric heel ulceration

Correct answer: (A) A pediatric flexible deformity. Explanation: UCBL devices are appropriate for pediatric patients with flexible rearfoot deformities such as flexible pes planus. (B) UCBL devices are not intended for rigid deformities as there needs to be adequate flexibility to achieve manipulation and correction of the rearfoot. (C) A UCBL device is not used to correct sagittal plane deformity, as there is no mechanism within the device to manipulate in this plane. If sagittal stabilization is desired, an AFO may be a more appropriate choice. (D) UCBL devices are not intended for patients with ulcerations. An accommodative insert would be more appropriate for this type of pathology.

You prescribed prednisone to your patient to relieve inflammation and you are concerned about suppression of the hypothalamic-pituitary-adrenal (HPA) axis . Which of the following treatment lengths and doses will not risk HPA axis suppression? A. Any period of time, and 5mg B. 30 days and 20 mg C. 20 days and 30 mg D. 40 days and 15 mg

Correct answer: (A) Any period of time, and 5mg. Explanation: In general any patient who has received the equivalent of 20 mg/d of prednisone for greater than 5 days is at risk for suppression of the hypothalamic-pituitary- adrenal axis, and if they have been on therapy for approximately a month they may have HPA suppression for up to 6 to 12 months after stopping therapy. Similarly, an equivalent dose of prednisone 5 mg or less for any period of time will usually not significantly suppress the HPA axis. Other modes of steroid administration should be noted preoperatively, since topical, inhaled and regional administration of glucocorticoids may cause adrenal suppression.

What are the 3 phases of host biologic response to a implanted biomaterials? A. Cellular response, tissue remodeling, infection potential B. Inflammatory phase, proliferative phase, remodeling C. Lag phase, growth phase, consolidation phase D. Cellular response, hemostasis, consolidation

Correct answer: (A) Cellular response, tissue remodeling, infection potential. Explanation: Cellular response, tissue remodeling, infection potential. The response of the host to implanted biomaterials, which in essence serve as foreign bodies, may be divided into three areas of concern. 1- the cellular response, which deals with issues of inflammation, immunology, toxicity, and cancinogenicity, 2- the tissue remodeling response, which includes issues of implant encapsulation, bone resorption, and sclerosis, and 3- infection potential, in that any surgical procedure involving implanted material may also increase the patient's risk of infection.

A few months after a bunionectomy, a healthy 30-year-old female develops a constant burning pain to the surgical limb as well as notices her skin become glossy and hairless with occasional bullous eruptions, erythema, edema, and a cold-to-touch foot. She has also developed a cold- intolerance to that foot. What is the most likely diagnosis? A. Complex regional pain syndrome B. Levido reticularis C. Vascular compromise from the surgery D. Cellulitis

Correct answer: (A) Complex region pain syndrome. Explanation: Complex region pain syndrome (CRPS) can present in many ways but the above description is a common presentation. CRPS has five major components that are commonly seen clinically: pain, edema, autonomic dysfunction, motor dysfunction, and trophic changes. Most notable is pain at rest and pain out of proportion to the injury or surgery performed, often provoked by trivial stimuli such as gusts of wind, bed sheets, and shower water pulsations. Unfortunately, diagnosis of CRPS is lacking a gold standard and is often a diagnosis of exclusion. Treatment is based on patient motivation, desensitization, reactivation with or without pharmacologic and interventional procedures, and cognitive behavior techniques.

Which of the following terms is defined as the amount of deformation or stretch that a tensile stress or load will produce? A. Elongation B. Fatigue life C. Fatigue limit D. Passivation

Correct answer: (A) Elongation. Explanation: Elongation is known as the amount of deformation or stretch that a tensile stress or load will produce. Fatigue life is the number of cycles a metal can undergo without breaking when placed under a given cyclic load. Fatigue limit or endurance limit is the cyclic load that a metal can endure indefinitely. Passivation is production of corrosion resistance by a surface of reaction products. This layer is normally an oxide layer, which is impervious to gas and water.

Which inhaled anesthetic has the following characteristics? - MAC = 1.7 - can potentiate seizures - nephrotoxic A. Enflurane B. Halothane C. Isoflurane D. Desflurane

Correct answer: (A) Enflurane. Explanation: Enflurane is a halogenated ether with a minimum alveolar concentration of 1.7. Enflurane is no longer in use. Enflurane has been shown to lower the seizure threshold making it contraindicated in epileptics. Enflurane has also been linked to malignant hyperthermia and renal failure. Enflurane is metabolized in the liver and ultimately its metabolite is the nephrotoxic agent. Enflurane may cause cardiac depression as well.

What is the position of the calcaneus in clubfeet? A. Equinus and varus B. Dorsiflexed and varus C. Vertical D. Equinus and valgus

Correct answer: (A) Equinus and varus. Explanation: The calcaneus is in severe equinus deformity with its anterior portion lying directly beneath the head of the talus. This displacement is responsible for the severe deformity of the heel. The calcaneus is severe equinus and in varus.

Pronation is made up which of the following movement combinations of the foot in open kinetic chain? A. Eversion of STJ, forefoot abduction, and dorsiflexion B. Inversion of STJ, forefoot abduction, and dorsiflexion C. Eversion of STJ, forefoot adduction, and dorsiflexion D. Eversion of STJ, forefoot adduction, and plantarflexion

Correct answer: (A) Eversion of STJ, forefoot abduction, and dorsiflexion. Explanation: During closed kinetic chain the calcaneus everts while the talus plantarflexes and adducts.

In a Steindler stripping, which group of muscles is sectioned? A. Flexor digitorum brevis, abductor digiti quinti, abductor hallucis B. Flexor digitorum brevis, adductor hallucis, abductor hallucis C. Abductor hallucis, adductor hallucis, quadratus plantae D. Quadratus plantae, abductor digiti quinti, flexor digitorum brevis

Correct answer: (A) Flexor digitorum brevis, abductor digiti quinti, abductor hallucis. Explanation: The procedure of Steindler stripping is done to decrease the pressure on the plantar fascia and plantar musculature. This soft tissue procedure is most commonly done in a cavus foot deformity to correct sagittal plane contracture. The muscles in the medial, lateral and calcaneal compartments are resected through a medial incision from the periosteum of the calcaneus. The quadratus plantae and adductor hallucis do not originate from the plantar calcaneus therefore any combination of answers with either of these two muscles is incorrect.

A ballet dancer has been diagnosed with a fractured tibial sesamoid. The decision has been made to resect the tibial sesamoid. What potential complication must the surgeon be worried about? A. Hallux valgus B. Hallux varus C. Hallux malleus D. Decreased push-off power

Correct answer: (A) Hallux valgus. Explanation: Removal of the tibial sesamoid may result in a hallux valgus deformity as the lateral slip of the short flexor has an increased mechanical advantage as compared to the medial slip of the short flexor. (B) Removal of the tibial sesamoid will not lead to a hallux valgus deformity, but rather a hallux varus deformity. (C) Removal of the tibial sesamoid will not create a hallux malleus deformity. Removal of both sesamoids could result in a hallux malleus deformity. (D) Removal of the tibial sesamoid will not significantly decrease push-off power. Resection of both sesamoids would have a far greater impact on push-off power.

The most probable etiology of the heloma molle pictured below is which of the following? A. Head of the third proximal phalanx and base of the fourth middle phalanx B. Head of the fourth proximal phalanx and base of the third middle phalanx C. Head of the third metatarsal and the base of the fourth proximal phalanx D. Base of the third middle phalanx and base of the fourth middle phalanx

Correct answer: (A) Head of the third proximal phalanx and base of the fourth middle phalanx. Explanation: A heloma molle or soft corn is a keratinized lesion that develops in the interdigital space. The lesion pictured above is a result of the interdigital skin being compressed between the lateral condyle of the head of the third proximal phalanx and the medial aspect of the base of the fourth middle phalanx. Conservatively they can be treated with debridement of the keratotic lesion and interdigital spacers. Surgically, removing the offending bony prominences with or without syndactlization is most often curative.

In following with the Ponsetti technique for treatment of talipes equinovarus, which aspect of the deformity is treated last? A. Hindfoot equinus B. Forefoot cavus C. Rearfoot varus D. Forefoot adductus

Correct answer: (A) Hindfoot equinus. Explanation: The Ponsetti method treated the condition with manipulation and serial casting starting with correction of the cavus deformity by dorsiflexing and supinating the first metatarsal. Once this was corrected, abducting the forefoot and applying counter-pressure at the talus to avoid a breach in the tibiofibular syndesmosis addressed the forefoot adductus. This also addresses the varus deformity of the rearfoot by dorsiflexing and anteriorly displacing the anterior calcaneus. Lastly, the equinus deformity was addressed through a percutaneous tenotomy of the Achilles tendon.

What anatomic structure is targeted by the X-ray beam of the lateral oblique view? A. inferior aspect of the calcaneus B. posterior ankle C. positioning block at posterior and inferior aspect D. calcaneocuboid articulation

Correct answer: (A) Inferior aspect of the calcaneus. Explanation: The lateral oblique view is a very common projection used in podiatry. The first metatarsal, first cuneiform, navicular, talus and calcaneus bones are well visualized. The film cassette is paced vertically, one-half of exposable portion in in the orthoposer slot. The medial aspect of the foot is next to the film and the opposite in angle and base of gait. The tube is angled 90o from vertical or perpendicular to the cassette. The tube is angled 90o from vertical or perpendicular to the cassette and targeted at the fourth metatarsal base.

What is the protocol for a patient with hypothyroidism on Levothyroxine, undergoing elective foot and ankle surgery? It is not imperative that patients take their levothyroxine dose the morning of surgery, because of the half-life of the drug which is approximately one week. Patient should take their levothyroxine the morning of surgery because of the negative effect of rebound hypothyroidism. The classic euthyroid is never incited by surgery. Hyperthyroidism has minimal effect on cardiovascular system, therefore no major organ risks.

Correct answer: (A) It is not imperative that patients take their levothyroxine dose the morning of surgery, because of the half-life of the drug which is approximately one week. Explanation: The preferred treatment of hypothyroidism is tetraiodothyronine (T4, Levothyroxine) replacement and patients should preferably be rendered euthyroid before surgery. The more active hormone T3 is less stable but is converted in vivo intracellularly. The half-life of levothyroxine is approximately one week, so it is not imperative for patients to take their dose the morning of foot and ankle elective surgery. In case intravenous dosing is necessary, one half of the enteral dose is equivalent.

Which of the following statements regarding ankle tourniquets is accurate? It should be applied for less duration than a thigh tourniquet. As a general rule the ankle tourniquet should be set approximately 150 mmHg higher than the systolic blood pressure. Is thought to have less chance of neuromuscular damage than a thigh tourniquet. It can be applied for longer duration than thigh tourniquet.

Correct answer: (A) It should be applied for less duration than a thigh tourniquet. Explanation: Ankle tourniquets placed just above the malleoli are thought to result in greater neurovascular damage due to the lack of adequate adipose tissue and muscle. It is recommended to place it slightly more proximal for increased safety. As a general rule an ankle tourniquet should be set 100 mmHg higher than the systolic blood pressure. Due to the proclivity of neurovascular damage potential an ankle tourniquet would be applied for a shorter duration than a thigh tourniquet.

Insertion of the flexor digitorum brevis: A. Middle phalanx of the lesser digits 2,3,4,5 B. Middle phalanx of the lesser digits 2,3,4 C. Distal phalanx of the lesser digits 2,3,4,5 D. Distal phalanx of the lesser digit 2,3,4

Correct answer: (A) Middle phalanx of the lesser digits 2,3,4,5. Explanation: The tendons on the plantar aspect on the base of the middle phalanges of digits 2 through 5 course with the flexor digitorum longus tendons, lying superficial or plantar to the flexor digitorum longus tendons;.The flexor digitorum brevis tendons, located near the middle of the proximal phalanges, divide, allowing passage of the flexor digitorum longus tendons, which are able to reunite deep to the flexor digitorum longus tendons before finally inserting on the middle phalangeal bases.

Which matricectomy procedure results in the least amount of post operative pain? A. Phenol matricectomy B. Scalpel matrix excision C. Sodium Hydroxide matricectomy D. Laser Matricectomy

Correct answer: (A) Phenol matricectomy. Explanation: Phenol has a local anesthetic effect which results in minimal post procedural pain. Scalpel excision will cut through distal nerve endings leaving exposed nerve tissue. Sodium hydroxide will chemically burn the nerve and laser will cause a thermal burn to the nerve endings. All three of these options will result in a greater amount of post procedural discomfort than a phenol matricectomy.

Plate application provides stability. However, installation of the plate can impair blood flow. This can lead to decreased bone healing. What is the term for plates with screw fixation leading to impaired periosteal blood flow? A. Plate associated osteoporosis B. Thermal necrosis C. Fat embolization D. Avascular necrosis

Correct answer: (A) Plate associated osteoporosis. Explanation: Plates provide stability, but can impair blood supply to the bone. This can decrease bone healing. A single screw inserted through a plate can generate 2000 N of compression. This force impedes periosteal blood flow in the plate region. This results in reduction in bone density under the plate. Loss of density occurs first at a distance from the plate and then slowly moves toward the plate surface. This is referred to as plate-associated osteoporosis. Other problems that can include: risk of bone refracture after the screws are removed, plate irritation, and sometimes an immunological reaction.

Which of the following first metatarsal osteotomies has the greatest amount of intrinsic stability? A. Scarf B. Ludloff C. Mau D. Closing base wedge

Correct answer: (A) Scarf. Explanation: The Scarf osteotomy (dividing the 1st metatarsal in a Z-shape fashion) is the most intrinsically stable osteotomy of all the bunionectomy procedures. The proximal dorsal and distal plantar angled cuts effectively "lock" the two bone segments together providing a significant amount of intrinsic stability. Both the Ludloff (oblique osteotomy of the first metatarsal in the transverse plane) and the Mau procedure (oblique diaphyseal osteotomy) are oblique osteotomies that, although share great surface area of cancellous bone contact lack much intrinsic stability. The closing base wedge osteotomy is powerful in its corrective abilities, but also do not possess intrinsic stability.

This structure helps maintain the orientation of the extensor digitorum brevis and the the extensor digitorum longus: A. The fibroaponeurotic extensor hood B. The capsule C. The bases of the distal phalanx 2,3,4,5 D. The dorsal interossei muscles

Correct answer: (A) The fibroaponeurotic extensor hood. Explanation: These two tendons maintain their orientation in part due to the fibroaponeurotic extensor hood. Its proximal segment, called the extensor sling, attaches to the plantar base of the proximal phalanx. It reattaches to the plantar base of the proximal phalanx. It receives contributions from the interossei muscles. Its distal segment or extensor wing receives the insertion of the lumbricales.

The following structures make the fifth ray: A. The fifth metatarsal only B. The fifth metatarsal and the cuboid C. The fifth metatarsal, the lateral cuneiform and the cuboid D. The fifth metatarsal and the lateral cuneiform

Correct answer: (A) The fifth metatarsal only. Explanation: The fifth ray consists of the fifth metatarsal only. Its motions occur at the articulation of the base of the fifth metatarsal and the cuboid. The fifth metatarsal base is connected to the cuboid by a ligament that lies dorsolateral to the base. In some individuals, a transverse band extends from this ligament to the dorsum of the third cuneiform. To be complete the second, third rays have the following structures that make their each individual ray: second and third rays consist of the second metatarsal with the intermediate cuneiform and the third metatarsal with the lateral cuneiform. These articulations have dense ligaments between them uniting them as a functional unit.

This is the weakest portion of the AO/ASIF partially threaded screw. A. The runout B. The shaft C. The core D. The pitch

Correct answer: (A) The runout. Explanation: The shaft of the screw is that region between the underside of the screw head and the point where the threads begin. The region where the shaft meets the core of the screw (starting point of the threads) is referred to as the runout. The runout is the weakest part of a partially threaded screw.

You are a 1st year Podiatric resident and it is your clinic day. The technician is not available and you would like to proceed with radiographic studies. This time you want a lateral projection, the patient is correctly positioned. What is the appropriate way to position the tubehead to take a lateral projection of the foot? The tubehead must be precisely positioned at 90 degrees from the vertical with the central direction at the lateral cuneiform. The tubehead must be precisely positioned at 90 degrees from the vertical with the central direction at the lateral malleolus. The tubehead must be precisely positioned at 45 degrees from the vertical with the central direction at the lateral cuneiform. The Tubehead is static, the patient should be able to mimic the angulations required.

Correct answer: (A) The tubehead must be precisely positioned at 90 degrees from the vertical with the central direction at the lateral cuneiform. Explanation: The tubehead must be precisely positioned at 90 degrees from the vertical with the central direction at the lateral cuneiform, if not the plantar aspect of the foot may be cut off.

This reactive phase of a host biologic response to implanted biomaterials is characterized by implant encapsulation, bone resorption, and sclerosis: A. Tissue remodeling B. Cellular response C. Immunologic response D. Carcinogenic phase

Correct answer: (A) Tissue remodeling. Explanation: Tissue remodeling response is a reactive phase of a host biologic response to implanted biomaterials. It is characterized by implant encapsulation, bone resorption, and sclerosis. Cellular response is the response of the body to implanted biomaterials, characterized by the inflammation, immunologic response, toxicity, and sometimes carcinogenic evolution. Immunologic response is part of the cellular response, and carcinogenic response as well.

Which of the following is the most devastating patient controlled health risk in terms of promoting non-union? A. Tobacco use B. Lack of Vitamin D consumption C. Excessive alcohol use D. Use of NSAIDs

Correct answer: (A) Tobacco use. Explanation: Tobacco use is the most devastating user-controlled health risk with respect to aiding in non-unions. One study determined that smoking was found to have a non-union rate 2.7 times higher than non-smokers for hindfoot fusions. Another study found that the smoking population took 1.7x longer to heal an Austin bunionectomy than non-smokers. Tobacco and nicotine have been shown to down-regulate and interfere with the formation of bone morphogenic proteins (BMPs), a group of cytokines and growth factors that induce bone and cartilage formation.

44 year old Caucasian Female admitted to the hospital status-post right BKA due to severe peripheral vascular disease. The patient spiked a fever of 101.2 Degrees Fahrenheit overnight. Which of the following is considered to be a typical etiology of post-operative fever in this setting? A. Urinary Tract Infection B. Viral Infection C. Fungal Infection D. Parasitic Infection

Correct answer: (A) Urinary Tract Infection. Explanation: Post-Operative Fever is considered fever >101.4oF and is most commonly seen in the few days after surgery. Some of the most common causes of infection in the Post-operative patient include urinary tract infections (A), surgical site infections, respiratory infections, and atelectasis. Viral infections, fungal infections and parasitic infections are typically not often seen in the post-operative period of a patient who had been previously immunocompetent.

What is the minimal ankle brachial index required for successful healing of a digital amputation? A. 0.35 B. 0.45 C. 0.55 D. 0.65

Correct answer: (B) 0.45. Explanation: An minimum ankle brachial index of 0.45 is required for a digital amputation. This is the perfusion threshold needed for successful tissue healing. (A) An ankle brachial index of 0.35 is too low to ensure successful healing of a digital amputation. (C) An ankle brachial index of 0.55 is sufficient for healing of a digital amputation, but is not the minimal threshold as asked in the question. (D) An ankle brachial index of 0.65 is sufficient for healing of a digital amputation, but is not the minimal threshold as asked in the question.

Which muscles are found in the third layer of the plantar foot? A. Plantar and dorsal interossei B. Adductor hallucis, flexor digiti minimi quinti brevis, flexor hallucis brevis C. Abductor digiti minimi, abductor hallucis, flexor digitorum brevis D. Quadratus plantae and lumbricales

Correct answer: (B) Adductor hallucis, flexor digiti minimi quinti brevis, flexor hallucis brevis. Explanation: The adductor hallucis muscle is the central layer of the 3rd layer, and has a transverse head and an oblique head. The transverse head originates from the plantar border of the plantar plate of the fifth, fourth and third metatarsal phalangeal joints, the proximal border of the deep transverse metatarsal ligament between toes 5 and 4, 4and 3, 3 and 2, the logitidunal septa of the plantar aponeurosis of the fifth, fourth and third toes and the medial crux of the plantar aponeurosis. The oblique head originates on the plantar aspect of the 2nd through the 4th metatarsals and fibularis longus tendon sheath also known as fibular canal. The flexor digiti minimi quinti brevis originates from the fibrous sheath of the peroneus longus, the crest of the cuboid, the base of the fifth metatarsal and the plantar aponeurosis and inserts into the plantar plate of the fifth metatarsal phalangeal joint and the base of the proximal phalanx of the fifth toe. The flexor hallucis brevis has a Y shaped fibrotendinous origin, the medial Y of the arm originates from the metatarsal component of the tibialis posterior tendon. The lateral arm of the Y originates from the lateral cuneiform and the cuboid. It inserts in the plantar medial and plantar lateral tubercles of the base of the hallucal phalanx.

Which of the following statements regarding digital amputations in diabetic individuals is correct? A. Should not be performed in patients with an ankle brachial index less than 0.65 B. Are likely to result in contracture of the adjacent digit C. Should always be accompanied by a metatarsal head resection D. Should not be performed in individuals with complete digital hair loss

Correct answer: (B) Are likely to result in contracture of the adjacent digit. Explanation: The abnormal mechanics after digital amputation causes a tendon imbalance in the adjacent digits leading to contracture. This is particularly troubling in diabetics where the buck- ling of the adjacent digit will cause a retrograde pressure and a new ulceration might develop un- der the adjacent metatarsal head. Digital amputations are very common among diabetic pa-tients. (A) The minimum threshold for an ankle brachial index to ensure adequate healing is 0.50 not 0.65. (C) The idea that a metatarsal head resection should always accompany the digital amputation is incorrect as it would cause even a greater biomechanical imbalance. (D) The notion that digital amputations in diabetics should not be performed in individuals with complete digital hair loss is inaccurate. Although the presence of digital hair is a general repre- sentation of adequate circulation, the absence of hair does not signify inadequate circulation to heal a digital amputation.

Which of the following potential complications of distal osteotomy of the first metatarsal for the treatment of hallux abducto valgus is the most serious? A. Recurrence of deformity B. Avascular necrosis of first metatarsal C. Hallux varus D. Transfer metatarsalgia E. Joint stiffness

Correct answer: (B) Avascular necrosis of first metatarsal. Explanation: All of the listed items are potential complications of first metatarsal head osteotomies for the treatment of hallux abducto valgus. Avascular necrosis of the first metatarsal head, although uncommon, is the most potentially serious complication due to the difficulty of its treatment and to its sequelae. It can result in residual deformity with associated incongruity and limitation of motion leading to development of degenerative joint disease in later life. Avascular necrosis, also known as aseptic necrosis, is a conditions that begins with initial loss of blood supply to the bone, followed by bone death and gradual replacement of dead bone by living bone. The incidence of aseptic necrosis following distal osteotomies of the first metatarsal has been reported at ranges from 1% to as great as 40%. Preservation of blood supply, preservation of adherent soft tissues on the lateral aspects of the first metatarsal head, and maintenance of good bone apposition can reduce the incidence of aseptic necrosis.

Which of the following tissues will absorb the largest amount of x-ray radiation in a diagnostic radiographic study? A. Lung B. Bone C. Muscle D. Liver

Correct answer: (B) Bone. Explanation: Bone has the greatest tissue mass density of the tissues listed, so will offer more resistance to the passage of x-radiation. Tissues with a higher tissue mass density absorb a larger portion of the radiation in the x-ray beam. Lung tissue is filled with air and has the lowest tissue mass density, with easier penetration and less absorption of radiation.

A 34 year old male presents to the ER with an open fracture to the left foot. He was chopping wood an his ax slipped and struck the left foot. Clinical and radiographic exam reveals highly comminuted metatarsal fractures of metatarsal 1, 2, 3. There is a 9 cm open wound adjacent to the fractures. The patient has no drug allergies. What would be the antibiotic would be recommend per the Gustilo classification for open fractures? A. Cefazolin B. Cefazolin and Gentamicin C. Cefazolin, Gentamicin, Penicillin D. Vancomycin and Gentamicin

Correct answer: (B) Cefazolin and Gentamicin. Explanation: The combination of Cefazolin and Gentamicin is the correct antibiotic regimen for the scenario above. The open fracture described above is a Gustilo grade III. A Gustilo grade III fracture is a highly contaminated open fracture with a wound > 5 cm. There is extensive soft tissue damage including skin, muscle, nerve and vasculature. Typically there is a large crush component with significant comminution. The recommended antibiotic treatment for a grade III fracture is Cefazolin and Gentamicin. The Gentamicin is added to provide coverage for nosocomial aerobic gram-negative rods like Pseudomonas and Enterobacter species. (A) This is the antibiotic recommendation for a Gustilo grade I fracture. (C) This is the recommended antibiotic regimen for a Grade II or III fracture with soil inoculation or occurring on a farm. (D) Vancomycin and Gentamicin in combination are not a recommended combination for any Gustilo grade fracture.

A 56 year-old male smoker on medication for hypertension which is controlled would be categorized by the American Society of Anesthesiology (ASA) as: A. Class I B. Class II C. Class III D. Class IV

Correct answer: (B) Class II. Explanation: Class II—a patient with mild systemic disease e.g. controlled hypertension or controlled diabetes without systemic effects, cigarette smoking without COPD, anemia, mild obesity, age younger than 1 year or older than 70 years, pregnancy.

This is a 23 year old presenting with an acute fracture (plain films are found below). After a thorough history and physical, and proper testing, he is diagnosed with an acute close fracture of the second metatarsal and incomplete fracture of the 3rd proximal phalanx, right. (See plain film radiographs below) What is your treatment plan? A. Open reduction internal fixation B. Closed reduction, with fiberglass cast application C. Cam boot, rice and elevation D. External fixation application

Correct answer: (B) Closed reduction, with fiberglass cast application. Explanation: There is displaced fracture of the mid-diaphysis of the 2nd metatarsal, with shortening, since this is a closed fracture without comminution, an attempt at closed reduction should be done. The goal is to restore the anatomy and immobilize the foot and start proper bone healing. (See picture below) (A) In this scenario, it is an acute fracture minimally displaced; closed reduction should be tried as a primary option. (C) This will not be the correct choice as the fracture is displaced, placing the patient in a Cam boot will immobilize the foot and will help heal the fracture, but improper correction will be the result, as there will be no attempt at anatomical reduction. (D) This should not be the primary attempt.

A surgical approach in a forefoot neurectomy procedure that most limits full access to the neuroma is: A. Dorsal longitudinal B. Dorsal transverse C. Plantar transverse D. Plantar longitudinal

Correct answer: (B) Dorsal transverse. Explanation: The author of this question has never seen this incision described for neuroma excision. It simply does not provide enough access, either distal or proximal, to adequately excise the neuroma. (A) Probably the most commonly utilized incision for forefoot neuroma excision. The length can vary, Easley uses 3cm, and terminates at the webspace. A longer incision terminating on the toe might be better for exposure. The incision is on the webspace, not over a metatarsal. (C) Easley places this incision over the affected interspace and proximal to the WB pad of the foot. It is parallel to the natural crease of the skin. The neuroma will be plantar to the FDB muscle or tendon and dorsal to the plantar fascia. (D) The recommended approach for a revisional neuroma surgery. Easley places it 4cm proximal to the web and extends it distally to within 1cm of the webspace. The incision is placed between the metatarsals. With either plantar approach, the DTML does not require transection, as the neuroma (or stump neuroma) is inferior to the ligament.

During the gait cycle, this phase is characterized by both feet being on the floor at the same time. A. Single limb support B. Double limb support C. Swing phase D. Single stance

Correct answer: (B) Double limb support. Explanation: During the gait cycle there are two periods when both feet are on the floor at the same time. Initial double support, or double stance, occurs from heel contact of one limb to toe- off of the opposite limb. This period has often been referred to as the period of reception, weight acceptance, or the braking period. During this phase, the body decelerates in preparation for stability and support by the limb that has just completed swing. Once this is accomplished, the opposite limb leaves the floor. Terminal double support, or double stance, occurs from opposite- limb heel-strike to support-limb toe- off. This period has often been referred to as thrusting double support, pre- swing, or weight release. Once again, there is a deceleration of the body as weight is transferred to the opposite limb. Once unloaded, it prepares for toe-off and the swing phase.

Which of the following statements is the exception regarding the proliferative or fibroblastic phase of wound healing? A. The proliferative or fibroblastic is marked by the massive influx of fibroblasts. B. Fibroblasts reach their maximum number approximately 2.5 weeks after injury. C. The fibroblasts are responsible for the formation of connective tissue matrix that provides a substrate for migrating macrophages, fibroblasts and new blood vessels. D. The fibronectin embedded in the connective tissue matrix enhances fibroblast movement and aids in wound contracture.

Correct answer: (B) Fibroblasts reach their maximum number approximately 2.5 weeks after injury. Explanation: As wounds enter the proliferative or fibroblastic phase, the fibroblasts start arriving in record numbers reaching their maximum at approximately 1 week after the injury. The fibroblasts come in response to the growth factors released by the macrophages in the first phase. The fibroblasts are responsible for the formation of the connective tissue component of the collagen and glycoaminoglycans that make up the connective tissue matrix that provides a substrate for the migrating macrophages, fibroblasts and new blood vessels. These cellular and vascular elements are embedded in the loose matrix of collagen, fibronectin, and hyaluronic acid. The fibronectin enhances fibroblasts movement and aids in would contracture.

Clinical features of congenital vertical talus include: A. Hindfoot calcaneus B. Forefoot abduction C. Concave medial longitudinal arch D. Prominent cavus midfoot

Correct answer: (B) Forefoot abduction. Explanation: The talus is dislocated medially and vertically with the head facing plantarly. This causes the navicular to rigidly dislocate dorsally on the talar neck. The pathologic changes cause abduction contractures of the forefoot. (A) The rearfoot is in equinus due to contractures of the triceps surae muscles. However, it is important to note that dorsal extension of the forefoot may mask the true equinus deformity. (C) The sole of the foot is convex due to the palpable dislocated talus rather than a normal concave medial longitudinal arch. (D) The midfoot is flattened, not high like a cavus foot. Therefore, it is important to distinguish between vertical talus and flexible flatfoot, both of which can be present at birth.

Which option listed below would be the best procedure as part of the surgical plan to correct the contracted toes 2,3,and 4? A. Flexor tendon transfer B. Hibbs procedure C. Plantar plate repair D. Lesser metatarsal osteotomy

Correct answer: (B) Hibbs procedure. Explanation: Extensor substitution hammertoes are commonly encountered in cavus and anterior cavus foot types. Due to the plantarflexed nature of the forefoot the long extensor tendons fire for a longer period of time in the gait cycle to facilitate the clearance of the toes to the ground. This causes an uneven tug-o-war phenomenon and the long extensors overpower the long flexors. This deforms the toes and leaves very prominent extensor tendons tenting the skin on the dorsum of the foot. The Hibbs procedure, or transfer of the long extensor tendons to either the peroneus tertius or base of the fourth and fifth metatarsals. This converts the long flexors from a deforming force on the digits to a corrective effect by dorsiflexing the forefoot as a whole.

Regarding antibiotic prophylaxis in the management of mammalian (non-human) bite injuries of the lower extremity, which statement is true? A. It is routinely indicated. B. It is indicated in wounds over tendon or bone. C. It is indicated in all dog bites. D. It is indicated in superficial skin wounds. E. The preferred regimen is erythromycin for 10 days.

Correct answer: (B) It is indicated in wounds over tendon or bone. Explanation: Antibiotic prophylaxis is indicated in the management of mammalian bites in the lower extremity when such bites are over bone, tendon, involve crushing of tissue, and in deep puncture wounds. (A) Antibiotic prophylaxis is not routinely indicated in these injuries. (C) It is not routinely indicated in dog bites, but is indicated in deep puncture wounds from cats. (D) It is not indicated in superficial skin wounds, except possibly in immunocompromised patients. (E) The preferred regimen is amoxicillin-clavulanic acid for 2-5 days.

Walking is studied in the gait cycle; the gait cycle has two phases, the stance phase and the swing phase. The following happens in the stance phase: A. It is the nonweightbearing portion of the gait cycle. B. It is initiated with heel contact. C. This is the period of time the foot is in swing phase. D. It is divided in 5 subphases.

Correct answer: (B) It is initiated with heel contact. Explanation: It is the weightbearing portion of each gait cycle. It is initiated with heel contact and ends with toe-off of the same foot. This period of time when the foot is in contact with the ground is divided in four phases, further subdividing the traditional three subphases of stance.

Which of the following statements about the abductor digiti minimi muscle flap is false? Its vascular supply is from the lateral plantar artery. Its main advantage is that there is excellent muscle bulk at the distal end of the graft where it is often needed. The flap is useful in covering lateral malleolar wounds, lateral midfoot wounds, calcaneus and heel. It has minimal donor site morbidity as the donor site can be closed primarily.

Correct answer: (B) Its main advantage is that there is excellent muscle bulk at the distal end of the graft where it is often needed. Explanation: In actuality the primary disadvantage of the abductor digiti minimi (ADM) is that it becomes quite small and thin distally where often bulk is needed. The ADM is a type II muscle with a dominant vascular pedicle and a few vessels. The lateral plantar artery is the dominant vessel. The ADM is very useful in covering lateral malleolar, heel, lateral midfoot and calcaneal wounds. The donor site can be closed primarily and therefore ever poses a problem post operatively.

What is the cause of the associated floating toe that accompanies the shortened metatarsal in brachymetatarsia? A. Spasticity of the long extensor tendon B. Lack of tension on the plantar fascia C. Underdevelopment of the long flexor tendon D. Altered pull on the plantar interossei

Correct answer: (B) Lack of tension on the plantar fascia. Explanation: In bracymetatarsia the metatarsal is short, but the muscle and tendon development is normal. This causes a lack of plantar fascial tension on the proximal phalanx which results in the floating toe deformity. (A) Spasticity of the long extensor tendon is not present in a brachymetatarsia deformity. (C) As stated above, the tendon and muscular development in cases of brachymetatarsia is normal and not underdeveloped. (D) The mechanical pull of the interossei is not altered in brachymetatarsia.

A 65 year-old diabetic female with hypertension would take all of the following medications on the morning of surgery except: A. Long-acting insulin 1⁄2 normal dose B. Metformin C. Beta-blocker D. Clonidine

Correct answer: (B) Metformin. Explanation: Diabetic medications: Insulin- give long-acting at 1⁄2 the normal dose, hold short- acting morning of surgery. Metformin- hold 2 days prior to surgery, due to risk of lactic acidosis if patient has renal problems preoperatively. Cardiovascular medications: Digoxin, beta-blockers, clonidine and calcium channel blockers are all continued up to and including day of surgery.

What is the most common indication for hardware removal in foot and ankle surgery? A. Infection B. Pain C. Carcinogenicity D. Joint irritation E. Implant breakage

Correct answer: (B) Pain. Explanation: Although all of the above answers are indications for hardware removal, which accounts for 5% of all orthopedic procedures performed, pain is the most common indication. Removal of hardware has been reported to provide improvement in pain in 50-75% of patients.

Local ester anesthetics have a higher incidence of allergic reactions than local amide anesthetics. Which of the following compounds is most responsible for the increased allergic potential of ester anesthetics? A. Methylparaben B. Para-aminobenzoic acid (PABA) C. GABA D. NMDA

Correct answer: (B) Para-aminobenzoic acid (PABA). Explanation: Ester local anesthetics have a higher rate of allergic reaction compared to their amide counterparts. This limits their routine use. A product of ester anesthetics is para- aminobenozic acid (PABA). This product is the compound that increases the allergic potential of ester anesthetics. Methylparaben is a preservative that was often added to local anesthetics to extend their shelf-life. It was identified as a potential allergen, and methylparaben-free formulations are available for use in those patients who have had a previous allergic reaction to this preservative.

Which of the following medications doesn't need to be discontinued 24 hours or more before surgery? A. Metformin B. Pioglitazone C. Incretins D. Chlorpropramide

Correct answer: (B) Pioglitazone. Explanation: Pioglitazone are thiazolidinedione, and can be held on the morning of surgery as they are given once daily with a duration of action of 24 plus hours. Secretagogues must be held preoperatively. The biguanide, metformin which has been associated with the development of lactic acidosis should be withheld 24 hours preoperatively, and restarted 48 to 72 hours postoperatively once normal renal function has been documented. Long acting sulfonylureas are best withheld 48 to 72 hours preoperatively to avoid potential hypoglycemia.

An anterior posterior projection radiograph of the ankle was ordered. What view is this? A. Anterior posterior view B. Posterior anterior view C. Mortise view D. Anterior view

Correct answer: (B) Posterior anterior view. Explanation: Radiographic positioning is usually described in terms of radiographic views or projections. These terms generally relate to either the trajectory of the X-ray beam or the relationship of the anatomic part being examined to the X-ray film. The term projection refers to the anatomic area of the foot or ankle into which the X-ray beam enters and the anatomic area from which the X-ray beam exits in the description of the projection. For example, a dorsoplantar projection of the foot is one which the X-ray beam enters the dorsal aspect of the foot and exits the plantar aspect of the foot. The term view refers primarily to the X-ray image and describes the portion of the foot closest to the film. For example, when a dorsoplantar projection is taken, the image produced is a plantar dorsal view because the plantar aspect of the foot is closest to the film.

The role of plates in bone and fracture stabilization is to maintain overall bone alignment and osseous contact that promotes the following: A. Indirect bone healing B. Primary bone healing C. Prevent pathologic fracture D. Prevent infection

Correct answer: (B) Primary bone healing. Explanation: The plate is a structure varying in size, shape, thickness and rigidity. Particular characteristics of the plate are based on site of application and biomechanical requirements of fracture segments. A plate is designed to work with bone to provide a stable construct. True stability can only be achieved when osseous segments are organized in a manner that allows them to transmit axial load through the full length of bone. Application of the bone plate maintains overall alignment and osseous contact that promotes primary bone healing.

Which of the following shoe modifications simulate plantarflexion at heel strike to decrease the ground reactive force? A. Thomas heel B. SACH heel C. Forefoot rocker D. External shoe lift

Correct answer: (B) SACH heel. Explanation: The solid ankle custom heel (SACH) heel replaces the posterior wedge of the heel (base plantar) with compressible foam rubber in order to simulate plantarflexion at heel strike. This decreases the ground reactive forces that flex the knee. (A) The Thomas heel flares the medial aspect of the standard heel approximately half an inch in order to reinforce the longitudinal medial arch. (C) A forefoot rocker is an additional platform on the bottom of the sole that sharply tapers from the metatarsophalangeal joints to the distal end of the shoe. This can relieve pressure from the MTPJs by facilitating midstance and toe-off, however does not influence plantarflexion at heel strike. (D) The external shoe lift is utilized in the case of pediatric limb length discrepancy. The lift is generally applied to the bottom of the sole in order to accommodate the inequality in length.

According to the Ponsetti method of clubfoot treatment, what is the recommended treatment for relapsing clubfoot in patients less than 2 years old? A. Repeat Achilles tenotomy only B. Same as the original treatment program C. Anterior tibialis tendon transfer D. Posterior tibialis tendon transfer

Correct answer: (B) Same as the original treatment program. Explanation: A study by Morcuende et al, found a relapse rate of 11% of those treated with the Ponsetti method. Of those 11%, he found that only 1% of families compliant with the abduction brace after casting experienced a relapse, whereas 89% of families non-compliant with the abduction brace experienced relapse in the deformity. Relapse of TEV is considered return of any one of the deformities that comprise the diagnosis of TEV. If the patient is less than two years old, the original treatment program of manipulation and casting may be done again with the one modification that the casts may be kept on for two weeks at a time rather than one week. This extra week allows for further time for the stronger and older connective tissue time to adapt. Typically, relapse management only requires two or three casts before reaching the goal of 70 degrees of abduction.

Simulated dorsal-plantar and lateral weight bearing foot projections are taken of an infant. Upon analysis, the following angular measurements were obtained: Lateral Projection: Talo-calcaneal angle: 0 degrees (below normal range) Calcaneal inclination angle: -5 degrees (below normal range) Simons angle: negative DP Projection: Talo-calcaneal angle: 4 degrees (below normal range) Engel's angle: 35 degrees (above normal range) Based on the measurements presented, what is the congenital deformity of this infant? A. Metatarsus adductus B. Talipes equino-varus C. Congenital pes valgus D. Calcaneovalgus

Correct answer: (B) Talipes equino-varus. Explanation: In order to correctly answer this question, you have to know what axes make up each angle and you have to remember what planar abnormalities occur with the listed congenital deformities. Provided is information comparing this infant's measurements to the normal range/values. The measurements obtained from the lateral projection tell us that the calcaneus is in equinus, which means that the posterior aspect of the calcaneus is positioned superior to the anterior process and the heel does not touch the ground. This does not occur in patients suffering from metatarsus adductus, which is primarily a transverse plane deformity. Metatarsus adductus does not alter the foot structure on a lateral projection. Therefore, metatarsus adductus can be crossed out. A decreased calcaneal inclination angle does occur in congenital pes valgus, calcaneovalgus and talipes equino-varus. However in calcaneovalgus deformities, the measurement is typically not a negative value. That information coupled with an increased Engel's angle, which means metatarsus adductus is present, allows us to cross out calcaneovalgus as a potential answer. Lastly, a talo-calcaneal angle value that is below the normal range on both the lateral projection and DP projection (as is the case here) suggests that there is a component of rearfoot varus and forefoot adduction. Therefore the correct answer is (B) talipes equino-varus.

What makes the fourth ray? A. Fourth metatarsal and cuboid B. The fourth metatarsal alone C. The fourth metatarsal and the lateral cuneiform D. The fourth metatarsal and the third cuneiform

Correct answer: (B) The fourth metatarsal alone. Explanation: The fourth ray is the fourth metatarsal alone. Root et al described the axes of these rays as most likely to be present in the transverse and frontal planes, just proximal to the tarsometatarsal articulations. Motion about this axis would be pure sagittal plane dorsiflexion/plantarflexion. Some plantarflexion movement of the rays is required for lesser metatarsophalangeal joint dorsiflexion to occur.

A 35 y/o patient presents with a painful tailor's bunion deformity. In addition to the painful bump, there is a callus both lateral and plantar, for which she has used OTC acid patches. She is an executive at a bank, and wears 3" heels daily, and refuses shoegear changes. She is interested in surgery. Radiographs demonstrate an IM angle of 7 degrees and a lateral deviation angle of 12 degrees. The BEST surgical procedure to reduce both the TB deformity and the calluses is a: A. Chevron B. Wilson C. Fifth metatarsal head resection D. Proximal CBWO

Correct answer: (B) Wilson. Explanation: The IM angle is normal (range 6.4-9.1); the LDA is high (range 0-7). Thus, a proximal osteotomy is not indicated. The patient is young, too young for a metatarsal head resection. Also, following metatarsal head resection, a complication can be pain and callus recurrence at the site of the resection. A chevron osteotomy can reduce the LDA, but its medial translation is limited. Additionally, the capital fragment cannot be elevated to resolve the plantar callus. The Wilson is an oblique metatarsal head slide which can be translated medially, and dorsiflexed at the same time to aid in resolution of the plantar callus.

This will cause palpitations, diaphoresis, angina, tremors, nervousness, and hypertension when using epinephrine with local anesthesia. Which of the following would possibly cause palpitations? A. Epinephrine in concentrations of 1:100,000 B. Epinephrine in concentrations of 1:200,000 C. A higher dose with intravascular injection D. Epinephrine with the following dose: 100 mL of 1:100,000

Correct answer: (C) A higher dose with intravascular injection. Explanation: Epinephrine in concentrations of 1:100,000 and 1:200,000, works well and safely for surgery. The most common side effect is transient tachycardia after inadvertent intravascular injection. At higher doses, and with intravascular injection, palpitations, diaphoresis, angina, tremors nervousness, and hypertension can occur. (A) Epinephrine in concentration of 1:100, 000 is safe without the following: pheochromocytoma, hyperthyroidism, severe hypertension, or severe peripheral vascular disease. (B) Epinephrine in concentration of 1:200, 000 is safe without the following: pheochromocytoma, hyperthyroidism, severe hypertension, or severe peripheral vascular disease. (D) Epinephrine in concentration of 100 mL of 1:100, 000 is the maximum dose with less risk of toxicity.

What is responsible for the extreme rise in body temperature during malignant hyperthermia? A. up regulation of the hypothalamus B. violent, excessive muscle contraction C. adenosine triphosphate D. creatine kinase

Correct answer: (C) Adenosine triphosphate. Explanation: Malignant hyperthermia is a genetic condition involving a mutation in the ryanodine receptor in skeletal muscle sarcoplasmic reticulum. The sarcoplasmic reticulum is the calcium-storing organelle within the skeletal muscle cell. Furthermore, the ryanodine receptor modulates the calcium release from the sarcoplasmic reticulum. When an individual with a genetic predisposition for malignant hyperthermia is exposed to a trigger such a volatile inhalation anesthetics or depolarizing muscle relaxants, a massive intracellular release of calcium into the skeletal muscle occurs via the outpouring of calcium from the sarcoplasmic reticulum into the myoplasmic area of the skeletal cell. This causes severe muscle contractions. As the skeletal muscle attempts to sequester the excess calcium it must use a tremendous amount of adenosine triphosphate (ATP) which generates a great deal of heat. The generation of excessive internal body temperature is the hallmark of malignant hyperthermia. As a result of these processes the cell is damaged to such an extent that it breaks down which leaks potassium, myoglobulin, creatine, phosphate and creatine kinase into the circulation. These fee-flowing compounds initiate a cascade of deleterious effects.

After stumbling off a curve, a 39 year old female felt a sudden, sharp pain on outer aspect of her left foot. Immediately the patient found it almost impossible to bear weight on the foot. She is a healthy female on no medication, negative past medical history, past surgical history of tonsillectomy and is a nonsmoker. Evaluation at the emergency department revealed point tenderness with localized edema left foot. Her skin is intact. The following image was obtained. Which would be the most appropriate initial treatment? Percutaneous intermedullary screw fixation with nonweightbearing below the knee cast for 4-6 weeks. Placement of small unilateral external fixator and nonweightbearing for four weeks. Application of a weightbearing below the knee cast for 4-6 weeks. Application of a nonweightbearing below knee cast for 4-6 weeks.

Correct answer: (C) Application of a weightbearing below the knee cast for 4-6 weeks. Explanation: The standard of care for a non displaced or minimally displaced distal fifth metatarsal fracture is a below the knee weight bearing cast worn for 4-6 weeks. If instability is of concern, then first 2-3 weeks can be nonweightbearing, but as can be seen on the radiograph above with the lateral cortex intact. The intact lateral cortex provides a degree of stability form the onset. Critical review of the other options reveals an unnecessary level of aggressiveness. Percutaneous intermediary screw fixation is only for Jones-type fractures not distal ones as shown above. The placement of a unilateral external fixture is not indicated and not the standard of care. Again, as mentioned previously, no need exists to nonweightbear this cast as the intact lateral cortex provides a degree of stability.

The first metatarsophalangeal joint incorporates the articular facets of four bones within a single synovial joint capsule. What are those four bones? Medial cuneiform, base of the 1st metatarsal base, intermediate cuneiform, base of the 2nd metatarsal Base of the 1st cuneiform, navicular, second cuneiform, base of the 1st proximal phalanx Base of the proximal phalanx, head of the 1st metatarsal, the superior surfaces of the medial and lateral sesamoid Base of the proximal phalanx, head of the proximal phalanx, base of the distal phalanx, head of the 1st metatarsal head

Correct answer: (C) Base of the proximal phalanx, head of the 1st metatarsal, the superior surfaces of the medial and lateral sesamoid. Explanation: The first metatarsophalangeal joint incorporates the articular facets of four bones within a single synovial joint capsule the first metatarsal head, the base of the proximal phalanx, and the superior surfaces of the medial and lateral sesamoid bones.

Methemoglobinemia is a disorder where there is higher than usual methemoglobin in the blood. Use of which of the following local anesthetics has the lowest risk of developing methemoglobinemia? A. Prilocaine B. Lidocaine C. Bupivacaine D. Benzocaine

Correct answer: (C) Bupivacaine. Explanation: Methemoglobinemia is a disorder where there is a higher than usual methemoglobin in the blood. Methemoglobin has a lower than usual ability to bind oxygen. It is not very common to occur but the three local anesthetics that have the highest probability of occurrence are lidocaine, prilocaine and benzocaine.

Which of the following is the best diagnostic imaging to assess the posterior facet in of the subtalar joint in a calcaneal fracture? A. MRI B. 3 phase bone scan C. CT D. Ultrasound

Correct answer: (C) CT. Explanation: Computerized tomography (CT) is the gold standard to assess the posterior subtalar facet in a calcaneal fracture. It is also the basis for the Sanders classification for calcaneal fractures. CT of the foot and ankle uses direct multiplanar image acquisition with an ultra-high resolution bone algorithm. An MRI is more appropriate for soft tissue and other non-osseous abnormalities. A 3 phase bone scan is best for inflammatory processes of the bone, but not actual imaging of the bone. Ultrasound is also used to assess soft tissue as MRI, but it is less expensive.

In a Bier block, what is the most serious adverse event that could occur due to a loss of proximal tourniquet inflation with a resultant sudden large intravascular release of local anesthetic? A. toxic reaction B. syncope C. cardiac arrest D. hypotension

Correct answer: (C) Cardiac arrest. Explanation: The key to this question is recognition that the questions asking for the worse adverse event. All options listed above are possible adverse events, but without a doubt a cardiac arrest would be the most serious. The Bier block is a form of regional anesthesia where local anesthetic is administered into a intravenous line in the foot/ankle while the extremity is under the influence of a proximally inflated calf tourniquet. As the arterial flow is severely slowed to stopped, the local anesthetic seeps out of the porous veins and perfuses into the surrounding tissues distal to the tourniquet.

Which antibiotic cannot be mixed with polymethylmethacrylate (PMMA) cement? A. Erythromycin B. Tobramycin C. Clindamycin D. Vancomycin

Correct answer: (C) Clindamycin. Explanation: Antibiotics are often mixed with PMMA to serve as a delivery system; the antibiotics must be available in powder form. Polymerization of PMMA is an exothermic process, so only heat-stable antibiotics can be used. These include tobramycin, erythromycin, vancomycin, and cephalosporins to name a few.

A laterally based closing wedge osteotomy of the calcaneus is being performed as part of a pes cavovarus deformity correction. The surgeon employs static staple fixation of the calcaneal osteotomy. In regards to staple fixation of this osteotomy which of the following statements is false? This osteotomy is a good indication for staple fixation as it is a transverse osteotomy. The intact medial hinge of the calcaneal osteotomy acts as a point of fixation. Compression across the osteotomy with staple fixation is comparable to axial cancellous screw fixation. Sural nerve irritation and fracture of the medial hinge with staple placement are potential drawbacks of staple fixation.

Correct answer: (C) Compression across the osteotomy with staple fixation is comparable to axial cancellous screw fixation. Explanation: Static staple fixation simply does not provide compression, they simply hold two bone segments together. Staple fixation is relatively ineffective in countering shear, rotational and bending forces. This is in stark contrast to screw fixation which indeed does provide compression. Staple fixation is indicated in transverse osteotomies and primarily in largely cancellous bone with thin cortical walls. In the above described osteotomy the medial hinge does act as a point of fixation and is an integral player in the stability. Also it is well documented that sural nerve interruption can come from the nerve rubbing against the staple. Lastly, the impacting nature of staple insertion can easily fracture the somewhat dedicated medial cortical hinge of the osteotomy.

A patient presents following acute trauma with pain, parasthesias and palor of the right lower extremity. The patient has decreased sensation to the plantar foot and decreased plantarflexion strength. You suspect elevation of which lower leg compartment? A. Anterior compartment B. Lateral compartment C. Deep posterior compartment D. Superficial posterior compartment

Correct answer: (C) Deep posterior compartment. Explanation: These clinical findings are consistent with acute compartment syndrome. A stryker catheter is used to measure intracompartmental pressures to determine whether emergent fasciotomy is necessary. Loss of plantarflexory strength and decreased sensation to the plantar foot would suggest damage to the tibial nerve, which is located within the deep posterior compartment of the leg. (A) The anterior compartment of the lower leg includes the extensor muscles, anterior tibial artery and deep peroneal nerve. The deep peroneal nerve provides sensory innervation to the first webspace. (B) The lateral compartment includes the peroneal muscles and superficial peroneal nerve. The superficial peroneal nerve provides sensory innervation to the dorsal foot except the first webspace and the anterolateral distal leg. (D) The superficial posterior compartment includes the gastrocnemius and soleus muscles.

Antibiotics for an elective surgical procedure should be administered: A. Resection of the abductor hallucis muscle B. Closing base wedge osteotomy of cuboid C. Evans calcaneal osteotomy D. Arthrodesis of the 1st metatarsal-medial cuneiform joint

Correct answer: (C) Evans calcaneal osteotomy. Explanation: Answer choice (A), (B) and (D) are all indicated as procedures of the surgical correction of metatarsus adductus, however answer choice (C) is not. An Evans calcaneal osteotomy would lengthen the lateral column which would increase the deformity rather than aiming to correct it. In some cases the lateral column may need to be shortened.

Which fixation modality is best utilized if fixation is required in an area of frank infection? A. Casting B. Internal plates and screws C. External fixation D. Percutaneous steimann pins or kirschner wires

Correct answer: (C) External fixation. Explanation: With external fixation there is no retained metal left behind after union has occurred. This is a particular advantage if the operative site has been contaminated or frankly infected. It is ideal in infected joints where arthrodesis is desirable and cannot be achieved with internal fixation. It can provide compression, distraction, or neutralization, and it allows for early patient mobilization and motion of the adjacent joints as desired. Casting is not the best choice here as it provides nothing more than marginal stabilization and limits monitoring of infection. Internal plates and screws, although can work in an infected site, the major negative is that metal is retained once bone healing has occurred and can harbor and sequester bacteria. In the situation where the internal fixation was placed and the site later became infected, then the metal can be left in place providing it is still providing adequate fixation. Percutaneous pinning really has no place in this situation. It provides substandard fixation and will provide a conduit for bacteria to travel from the outside into the surgical site.

Which of the following is not a function of the foot during gait? A. Pronating mobile adapter B. Shock Absorber C. Flexible Lever D. Torque converter

Correct answer: (C) Flexible Lever. Explanation: The foot does NOT act as a flexible lever during the propulsive period. It is important that the foot re-supinates and becomes a rigid lever to propulse off of. If the foot stay pronated too long into the gait cycle, there will not be a good propulsion. (A) The foot is a pronating mobile adapter during the contact phase of gait. The foot hits the ground inverted and through the contact phase, the foot pronates- this is one reason that we are able to walk on terrain that is uneven. (B) The foot is a shock absorber during the contact phase. It is important to remember that when your foot hits the ground, it has a force that is a bit above your body weight. It is important for the foot to act as a shock absorber. The heel fat pad absorbs shock as does pronation of the foot. (D) It is important for the foot to act as a torque converter changing motion in the leg into tri- planar motion in the foot.

A 39-year-old male presents with the crush injury present below. X-rays are negative for fracture and dislocation. Which of the following tests is the most helpful in determining skin flap viability? A. Digital plethysmography B. Indium scan C. Fluorescein dye study D. Segmental pressure gradients

Correct answer: (C) Fluorescein dye study. Explanation: Fluorescein dye test is a valuable tool in determining viability of skin flaps. Studies have shown that it is the most accurate predictor of viability occurs 18 hours after injection. An indium scan is a nuclear medicine study labeling white blood cells and does not help determine viability. Segmental pressures only assess macro circulation and in the case of skin flap viability the micro circulation is of more interest. Digital plethysmography will only assess the toes and not the flap.

A 44 year old female presents to the surgery center for a Tailor's bunionectomy. The patient is fairly healthy and only takes Aspirin 81mg daily which she stopped 7 days ago and metoprolol which she took this AM. The nurse comes to evaluate the patient in the surgery center and notices the medications the patient is taking. Which of the following is correct? A. Cancel the surgery because the patient stopped Aspirin. B. Cancel surgery because patient took metoprolol this AM. C. Go forward with surgery as planned. D. Give patient fresh frozen plasma and then go forward as planned.

Correct answer: (C) Go forward with surgery as planned. Explanation: In this question, the patient is only on 2 medications. Aspirin which is an irreversible platelet inhibitor is usually held 7-10 days before surgery. Metoprolol which is a beta blocker is generally taken up to and including the day of surgery. The answer here is (C). (D) is not the correct answer. Fresh frozen plasma can be given to reverse the effects of Coumadin- would not be needed in this case.

What specific property of the tertiary amine group of a local anesthetic allows it to bind to and block conduction in the sodium channel of a nerve cell? A. Hydrophilic group B. Non-ionic form of the hydrophobic group C. Ionic form of the hydrophilic group D. Hydrophobic group

Correct answer: (C) Ionic form of the hydrophilic group. Explanation: Each side of the local anesthetic molecule has a purpose. Once the local anesthetic molecule reaches the inside of the nerve cell an equilibrium dissociation occurs yielding an ionic and a nonionic form. The hydrophilic group, in its ionic form, is the end of the molecule that gains entry into the sodium channel providing a blockade to action potential conduction.

A patient with chronic lateral ankle instability with history of recurrent ankle sprains would benefit from which shoe modification? A. Medial flare B. Medial buttress C. Lateral flare D. External shoe lift

Correct answer: (C) Lateral flare. Explanation: A lateral flare is an additional protruding edge from the lateral aspect of the sole to provide lateral stabilization. The presence of the lateral flare would decrease the risk of inversion injury. (A) A medial flare would stabilize the medial column and would not be beneficial for prevention of recurrent lateral ankle injury. (B) A buttress is an external shoe modification to the midsole, outsole and upper to provide additional stability. A medial buttress would provide medial stabilization, which would not address the lateral ankle instability. In this case a lateral buttress would be more appropriate. (D) The external shoe lift is utilized in the case of limb length discrepancy. The lift is generally applied to the bottom of the sole in order to accommodate the inequality in length.

Which option listed below would provide elongation and plantarflexion top the first metatarsal? A. Mau osteotomy with distal translation B. Mau osteotomy with proximal translation C. Ludloff osteotomy with distal translation D. Ludloff osteotomy with proximal translation

Correct answer: (C) Ludloff osteotomy with distal translation. Explanation: If the capital fragment in a Ludloff osteotomy is translated distally subsequent elongation and plantarflexion will occur due to the dorsal proximal to plantar distal nature of the osteotomy. Conversely, in the Ludloff osteotomy if the capital is translated proximally, a resultant shortening and dorsiflexion will occur. In the case of the Mau osteotomy, which is angled from dorsal distal to plantar proximal parallel to the weight-bearing surface, there is little if any sagittal plane movement. So, in the Mau, when the capital fragment is translated distally then simply elongation is noted. Lastly, in the Mau, when the capital fragment is translated proximally then shortening of the first metatarsal is noted.

Which is the most poorly tolerated complication of a first metatarsophalangeal joint arthrodesis? A. nonunion B. arthritis of the interphalangeal joint C. malposition D. persistent edema

Correct answer: (C) Malposition. Explanation: The most poorly tolerated complication of first metatarsophalangeal joint arthrodesis is malposition of the hallux. This is particularly the case of sagittal plane malposition. Considerable patient dissatisfaction arises from a non-purchasing hallux. A overly plantar flexed hallux will shorten stride length and not allow full foot loading while walking. Frontal and transverse planes while not well tolerated, are better tolerated than sagittal plane malunions. In contrast, delayed and nonunions are often not symptomatic. Arthritis of the interphalangeal joint can be managed with orthotics, NSAIDs (topical and oral) and intraarticular corticosteroid injections. Furthermore, if needed, interphalangeal joint arthrodesis is a much less intensivee procedure than revision of a malunited first metatarsophalangeal joint arthrodesis. Persistent edema usually is tolerated well and can be managed by shoe changes, compression garments and short term steroid use.

What radiographic projection is especially helpful in the diagnosis of Congenital Vertical Talus? A. Ankle Mortise B. Harris Beath C. Maximum Dorsiflexion and Plantarflexion lateral views D. Stressed Inversion

Correct answer: (C) Maximum Dorsiflexion and Plantarflexion lateral views. Explanation: These views are necessary to confirm the talonavicular dislocation associated with congenital vertical talus. Furthermore the maximum dorsiflexion lateral view exhibits the fixed equinus deformity of the calcaneus and talus relative to the tibia as a result of the short Achilles tendon. The maximum plantarflexed lateral view is used to measure talar axis-first metatarsal angle and calcaneal axis-first metatarsal base angle to differentiate an oblique talus from navicular displacement and Achilles shortening. Furthermore, the maximum plantareflexed lateral view must show the first metatarsal, lateral cuneiform, or navicular remaining dislocated relative to the talus in order to confirm the diagnosis of congenital vertical talus.

Arthrodesis of the ankle in a position of equinus is a known complication of the ankle fusion procedure. What is the most common cause of the equinus malunion? A. Premature weight bearing in the post-operative period B. Failure to adequately posteriorly translate the talus relative to the tibia C. Mobility of the transverse tarsal joint D. Failure to perform equinus release

Correct answer: (C) Mobility of the transverse tarsal joint. Explanation: Mobility of the transverse tarsal joint can deceive the surgeon into thinking the ankle is at 90° when in actuality it is in equinus. Intra-operatively the surgeon loads the forefoot to dorsiflex the ankle and if there is excess mobility in the midtarsal joint some of the dorsiflexion will come from it as opposed to the ankle joint. (A) Although premature weight bearing could result in a delayed or a nonunion, it would have no influence in the formation of an equinus malunion. (B) Failure to adequately translate the talus posterior relative to the tibia would not create an equinus malunion, but rather could hyperextend the knee. (D) Failure to perform an equinus release, such as a gastrocnemius recession or tendo achilles lengthening, will not cause a equinus malunited ankle. In fact there is no need to perform an equinus release with an ankle arthrodesis.

Which of the following dysplasias is described as having a "bone within bone" appearance? A. Osteopoikilosis B. Melorheostosis C. Osteopetrosis D. Osteopatha striata

Correct answer: (C) Osteopetrosis. Explanation: Osteopoikilosis is described radiologically as having multiple well defined sclerotic lesions in bone. These tend to look like bone islands. Melorheostosis is frequently described as a "candle wax" appearance with sclerosis in the periphery. Osteopatha striata is radiographically seen as linear bands of sclerosis within the bone. Osteopetrosis also known as Albers- Schonberg disease, shows bone within bone appearance on x-ray. Most of these patients tend to be relatively asymptomatic.

Across an osteotomy site or fracture site of a long bone, optimal compression is achieved with a lag screw. What is the axis direction of the screw? A. Perpendicular to the plane of the foot B. Perpendicular to the axis of the bone C. Perpendicular of the plane of the fracture D. There is no axis needed as long as there are 2 points of fixation

Correct answer: (C) Perpendicular of the plane of the fracture. Explanation: Optimal compression is achieved with the axis of the screw perpendicular to the axis of the plane of the fracture. Inter-fragmentary compression is achievable with other fixation devices in addition to lag screws but less efficiently.

Prior to performing an ankle fusion on a 75 year old patient, they are placed on bridging therapy with low molecular weight heparin. Which of the following represents the optimal management of LMWH postoperatively? Discontinue lower molecular weight heparin and restart warfarin at patient's regular dose 24 hours post procedure Discontinue lower molecular weight heparin and restart warfarin at patient's original dose 12 hours postoperatively when there is no more risk of bleeding Restart LMWH approximately 24 hours post procedure Restart LMWH approximately 12 hours post procedure

Correct answer: (C) Restart LMWH approximately 24 hours post procedure. Explanation: The bridging instructions should be as follow: stop warfarin 5 days before surgery if INR is 2-3, stop warfarin 6 days before surgery if INR is 3-4.5, start LMWH 36 hours after the last dose of warfarin, check INR the morning of surgery to ensure a INR < 1.5 and in some cases <1.2. Restart the LMWH approximately 24 hours post procedure, or consider thromboprophylaxis dosing of LMWH on post op day # 1 if patient is at high risk for bleeding. Restart warfarin at patient's usual dose on the surgical day, check INR daily until patient is discharged and periodically thereafter until INR is therapeutic. Check CBC on post-op days 3 and 7 to monitor platelets, discontinue LMWH when INR is therapeutic for two consecutive days.

Regarding the timing of operative intervention after forefoot trauma, which clinical indicators signify the appropriate time to operate? A. Drainage of fracture blisters B. Erythrocyte sedimentation rate less than 40 mm/hr C. Return of skin lines D. 50% reduction of edema

Correct answer: (C) Return of skin lines. Explanation: When deciding on the appropriate timing for surgery in a trauma case, one should look for complete subsidence of edema and the return of skin lines. In trauma surgery, the fracture is subordinate to the overall soft tissue envelope. The concept of operating at "low tide"or when the soft tissue moves out of the acute inflammatory stage is critical. (A) Fracture blisters should be allowed to completely epithelialize before manipulated or violated by an incision. (B) There is not a consistent clinical indicator relative to the erythrocyte segmentation rate. (D) Although the reduction of edema plays a role in the timing of trauma surgery, the applicable state is complete edema reduction and not a 50% reduction of edema.

A 64 year old female presents complaining of a "knot" on the dorsum of her right foot near her ankle. Last week she was walking and felt a pop. That evening she saw some bruising, but never noticed any pain. She reports history of corticosteroid injection is the area for some tendonitis she was having. On exam there is a 1 cm non-mobile mass on the dorsomedial ankle. When the first met head is loaded the hallux plantarflexes. What is your diagnosis? A. Ganglion cyst B. Giant cell tumor of the tendon sheath C. Rupture of the extensor hallucis longus D. Rupture of the anterior tibial tendon

Correct answer: (C) Rupture of the extensor hallucis longus. Explanation: The clinical scenario described above is that of a rupture of the extensor hallucis longus tendon. The tendon was weakened by the repeated corticosteroid injections. The fact that when the first metatarsal head was loaded and that caused plantarflextion of the hallux indicated an intact FHL with no resistance from the EHL. In other words, the EHL would have to be ruptured. EHL ruptures, when they occur, can cause a popping sensation and often have bruising associated with them. Rarely are they painful.

A patient with cardiac history takes 325mg aspirin daily. The patient is scheduled to undergo bunion surgery with arthrodesis of digits 2, 3 and 4 of the right foot. When is the most appropriate time to discontinue aspirin pre-operatively? A. Four hours prior to surgery B. One to two day prior to surgery C. Seven to ten days prior to surgery D. Three to four weeks prior to surgery

Correct answer: (C) Seven to ten days prior to surgery. Explanation: Aspirin works by irreversibly inhibiting platelet function. The platelet pool is replaced every 7-10 days, therefore aspirin should be held for this amount of time pre- operatively to allow adequate platelet function. As long as hemostasis is adequate, the pre- operative dose may be resumed 24 hours after the operation or the following morning. (A) Discontinuing aspirin four hours prior to surgery will not have a significant impact on restoring platelet function. It takes 7-10 days for the platelet pool to be replenished once aspirin is discontinued. (B) Discontinuing aspirin 1-2 days prior to surgery will not have a significant impact on restoring platelet function. It takes 7-10 days for the platelet pool to be replenished once aspirin is discontinued. (D) It is generally not necessary to hold aspirin for 3-4 weeks prior to surgery as the platelet pool is replenished after 7-10 days of holding the medication.

A patient walks into the office complaining of pain on the outside of her foot. States that she was walking in the grocery store yesterday and upon leaving the store, twisted her foot stepping down the curb. She states that the area became swollen and painful immediately and has been icing and taking Advil since it occurred. X-rays taken in the office show an extra-articular fracture of the 5th metatarsal base at the metaphyseal- diaphyseal junction which is non displaced. Which of the following is the correct classification? A. Stewart Type 3 B. Jahss Type 1 C. Stewart Type 1 D. Stewart Type 2

Correct answer: (C) Stewart Type 1. Explanation: The correct classification system for 5th metatarsal base fractures is the Stewart classification system. A Stewart Type 1, also known as the Jone's fracture is at the metaphyseal diaphyseal junction of the bone and that is the correct answer to this question. Stewart type 2 is an intra-articular fracture of the 5th met base and Stewart Type 3 is a extra-articular avulsion fracture of the styloid process of 5th met base. The Jahss classification describes 1st MTPJ dislocations with a Type 1 being a dorsal dislocation of proximal phalanx on metatarsal head without rupture of inter-sesamoidal ligament and type 2 being a dorsal dislocation of the proximal phalanx on the metatarsal head with rupture of inter-sesamoidal ligament and thus wide separation of the sesamoids.

What constitutes the articulation of the subtalar joint? A. The distal articular surface of the fibula. B. The distal articular surface of the tibia. C. The articulation between the talus and the calcaneus. D. The trochlear surface of the talus.

Correct answer: (C) The articulation between the talus and the calcaneus. Explanation: The subtalar, or talocalcaneal, joint is located between the talus and the calcaneus. It has been described as being composed of three facets on the inferior surface of the talus articulating with three facets on the superior surface of the calcaneus. The posterior facet of the talus articulating with the broad, superior surface of the calcaneus is united by ligaments and a capsule so that the joint is anatomically distinct.

Which of the following bones is typically the most deformed in a clubfoot deformity? A. The calcaneus B. The navicular C. The talus D. The cuboid

Correct answer: (C) The talus. Explanation: The foot is rotated medially beneath the talus; the head of the talus is palpable on the lateral aspect of the dorsum of the foot, owing to the inward and backward displacement of the navicular. The head of the talus is adducted and plantarflexed. The defect is in the head of the talus which is the most deformed bone in the clubfoot deformity. (A) The calcaneus is in severe equinus deformity with its anterior portion lying directly beneath the head of the talus. This displacement is responsible for the severe deformity of the heel. (B) The navicular is medially displaced and its tuberosity is palpable just anterior to the medial malleolus. (D) The cuboid, cuneiforms, and metatarsals contribute in different degrees to the severe adduction deformity of the clubfoot.

Which of the below is true of a child with an accessory navicular? A. These patients tends to show a rigid flatfoot. B. The calcaneal inclination angle is increased when looking at a lateral x-ray. C. There is an increased talocalcaneal angle on AP and lateral x-ray. D. Patients never have pain in this area.

Correct answer: (C) There is an increased talocalcaneal angle on AP and lateral x-ray. Explanation: In a flatfoot condition, there is an increase in the talocalcaneal angle. With a flatfoot, there is a decrease in articulation between the talus and navicular. The talus is directed more medially thus leading to an increase in talocalcaneal angle. (A) Patients with an accessory navicular usually have a severe flexible flat foot condition. Rigid flatfeet are usually seen with tarsal coalition and vertical talus (B) As accessory navicular is a cause of a flatfoot, there would be a decrease in the calcaneal inclination angle. Cavus feet would show an increase in calcaneal inclination angle. (D) It is common for patients to have pain in this area.

In the treatment of nonunion and delayed union in the foot and ankle, which statement is true regarding pulsed electromagnetic fields and low- intensity pulsed ultrasound? A. They are invasive. B. They require short treatment sessions. C. They have the potential disadvantage of poor compliance. D. They have low success rates. E. They must be surgically implanted.

Correct answer: (C) They have the potential disadvantage of poor compliance. Explanation: These devices have the potential disadvantage of poor compliance. (A) These devices are noninvasive. (B) Although the low-intensity pulsed ultrasound has short treatment times (20 minutes per day), the pulsed electromagnetic fields must be used for at least 3 hours per day. (D) These devices have high success rates. (E) As these devices are noninvasive, they require no surgical implantation.

Which is not a major component of a common stainless steel used in surgical implants, 316L? A. Molybdenum B. Nickel C. Titanium D. Chromium

Correct answer: (C) Titanium. Explanation: 316L is one of the most common types of stainless steel used in surgical implants. It is an alloy of several metals. The 3 major components of this stainless steel are chromium at 17-20%, nickel at 12-14%, and molybdenum at 2-4%. Manganese is also present at around 2%. Other metals that compose less than 1% include silicon, carbon, phosphorus, and sulfur. The L in 316L stands for low carbon. In addition to the low carbon content, the nickel component allows for the implant to be very resistant to corrosion. This is one of several properties that make this an attractive option for surgical implants.

The limb is lengthened by callous distraction when using an external fixator. What is the recommended amount of lengthening that is allowed and required for proper callous distraction? A. 0.5 mm daily B. 0.25 mm twice daily C. 0.6 mm D. 1 mm daily

Correct answer: (D) 1 mm.daily Explanation: After extensive animal investigation, Ilizarov determined that 1 mm per day—0.25 mm lengthenings, four times a day—resulted in acceptable new bone formation. The new bone formation is much better than that which is created by longer incremental lengthenings at the same frequency or by using 0.5 mm twice daily. The threaded connecting rods have a pitch of 1 mm; each supporting nut on the rod is turned one-quarter turn every 6 hours to effect a 0.25 mm distraction (1 mm distraction per day).

During clinic, the radiographic technician is not available but you would like a dorsoplantar (project) xray of the foot. The patient recently underwent a Lapidus procedure and is experiencing post-operative pain. At what angle should the tubehead be positioned? A. 45 degrees from the vertical, directed posteriorly with the knee flexed B. 0 degrees from the vertical, directed posteriorly with the knee flexed C. 90 degrees from the vertical, directed anteriorly with the knee flexed D. 15 degrees from the vertical directed posteriorly with the knee flexed

Correct answer: (D) 15 degrees from the vertical directed posteriorly with the knee flexed. Explanation: 15 degrees from the vertical directed posteriorly with the knee flexed so that the foot fully purchases the cassette. A sandbag should be placed along the front edge of the cassette. There is a variation of this technique, in which the tubehead can be at 0 degrees or vertical, and the foot can either remain flat or a 15 degree radiolucent wedge can be placed beneath the forefoot so the metatarsals are parallel to the film.

The physiological effects on a interdigital nerve after a neurectomy is analogous to that of a nerve after an amputation. On the same day the nerve is cut it begins to spontaneously fire. During the post operative period the nerve will have two physiologic peaks of activity. The first occurs at about the third postoperative day. When does the second occur? A. the sixth postoperative day B. the tenth postoperative day C. 2 weeks postoperatively D. 3 weeks postoperatively

Correct answer: (D) 3 weeks postoperatively. Explanation: The reality is that what we commonly refer to as an interdigital neuroma is nothing as such. It is in fact more appropriately named a perineural fibrosis. However, a true neuroma is formed once the neurectomy is complete. This is no different than in amputation surgery and the nerve responds in the exact same manner. Once it is severed it begins spontaneous firing and exhibits two peaks of activity. The first peak of activity occurs on the third postoperative day with the second peak occurring within the third postoperative week. For some patients, these peaks can be more symptomatic than in others and this is believed to be enhanced by sympathetic fibers.

Redundant skin over the sinus tarsi in idiopathic Talipes Equinovarus is thought to be caused by: A. Atrophy of the extensor hallucis brevis B. Atrophy of the ligaments from tissue ischemia C. Atrophy of the distal calcaneus from persisting malrotation D. Atrophy of the skin from persisting pressure of the talar head from within

Correct answer: (D) Atrophy of the skin from persisting pressure of the talar head from within. Explanation: The atrophy over the sinus tarsi seen in idiopathic Talipes Equinuovarus is from internal pressure from the dorsolateral talar head. Talipes Equinovarus can be divided into four forms. The first, and most predominant at 95%, is idiopathic. This has a genetic component and its inheritance is likely polyallelic. Positional/postural is another form and stems from intra- uterine position. Teratologic is another form associated with conditions such as arthrogryphosis and vertebral anomalies. Syndromic is the last form and is part of skeletal-visceral syndrome.

After performing a percutaneous tendo-achilles lengthening procedure, you suspect the tendon may have been over-lengthened. A resultant finding may include: A. Tibial neuritis B. Sural neuritis C. Equinus gait D. Calcaneal gait

Correct answer: (D) Calcaneal gait. Explanation: A common complication of a TAL procedure is overlengthening, which may lead to calcaneal gait. (A) The tibial nerve is not typically encountered in a TAL as it is further medial than the medial border of the Achilles tendon. (B) Sural nerve damage or entrapment may be c omplication of TAL and gastrocnemius lengthening procedures, however this would not be due to over-lengthening of the tendon. (C) With overlengthening, there would be increased laxity of the Achilles tendon, causing calcaneal gait. The opposite would be true for equinus gait, which is the indication for performing such a procedure.

Which Tailor's bunionectomy procedure is indicated for a patient 10 years of age? A. Distal chevron osteotomy B. Hohmann type osteotomy C. Wilson type osteotomy D. Closing base wedge osteotomy

Correct answer: (D) Closing base wedge osteotomy. Explanation: The key to this question is to recognize the developmental stage of the fifth metatarsal and that at 10 years of age there is an open growth plate at the neck of the fifth metatarsal. The only procedure listed above that would not violate the open growth plate is the closing base wedge osteotomy. The chevron (Austin-type), Hohmann and Wilson osteotomies are all capital-based procedures and therefore would violate the open growth plate.

Which of the following would not be found on physical exam of a patient with metatarsus adductus deformity? A. Forefoot is adducted at the lis-franc joint B. Lateral foot border is convex instead of straight C. Medial soft-tissue crease D. Decreased hindfoot and subtalar motion

Correct answer: (D) Decreased hindfoot and subtalar motion. Explanation: Decreased hindfoot and subtalar motion would not be found on physical exam of a patient with metatarsus adductus deformity. (A) (B) (C) These are all expecting findings in a patient with metatarsus adducts deformity, however (D) is incorrect. In metatarsus adductus the hindfoot and subtalar joint motion is normal, where as in a skewfoot, as known as serpentine foot deformity the hindfoot/subtalar joint is in valgus.

Cryosurgery as a treatment option for forefoot neuroma: A. Destroys the neuroma permanently. B. Preserves the perineurium and endoneurium. C. Destroys an approximate 2.5cm portion of nerve. D. Decreases neural edema and fibrosis.

Correct answer: (D) Decreases neural edema and fibrosis. Explanation: It is thought that with the ice ball at -70 degreesC, neural edema is resolved, which is a pain generator, and fibrosis around and even within the nerve is decreased, effecting, in a sense an external, and maybe internal, neurolysis. (A) Although not considered a permanent solution, L. Fallat, DPM has seen patients years out without recurrence of pain. (B) Cryosurgery preserves the perineurium and epineurium of the nerve. Through this selective preservation, the incidence of stump neuroma is much diminished; some say stump neuroma formation will not occur at all. (C) An approximate 1cm portion of nerve is destroyed. Axons regenerate at a rate of 1-3mm/day, so regeneration should be complete within weeks.

In Charcot-Marie-Tooth disease, the triple arthrodesis: A. Is the best first line treatment option due to the progressive nature of CMT. B. Can be successful as an isolated procedure. C. Can be performed at any age. D. Despite reported complications, remains the cornerstone of surgical management in CMT.

Correct answer: (D) Despite reported complications, remains the cornerstone of surgical management in CMT. Explanation: The triple arthrodesis remains the cornerstone in the surgical management of CMT, but is considered as a first line option only in patients with rigid cavus, with rearfoot DJD, who are generally in adulthood, have failed non-surgical management, or have never had intervention earlier in life designed to slow progression at an early age. When performed in the appropriate patient, the triple arthrodesis can reduce forefoot equinus and calcaneal varus, provide a stable plantigrade foot, and reduce a forefoot valgus and ankle equinus. Complications include pain (frequently due to malpositioning), nonunion, loss of shock absorbing ability of the foot, DJD of the ankle, neuropathic ankle changes, and changes in foot structure secondary to the progressive nature of CMT, causing pain. Performing the triple fusion without adjunctive tendon transfers, especially with the progressive nature of CMT, can lead to failure. Even though the rearfoot is fused, muscle imbalance can lead to subsequent deformities; an example is a still functional tibialis posterior muscle causing an inverted/ supinated foot post-operatively despite total rearfoot fusion. The timing of when to perform this fusion is controversial, but after skeletal maturity would be preferable.

Which one of the following should be held the morning of surgery? A. Digoxin B. Acetaminophen C. Beta-blockers D. Diuretics

Correct answer: (D) Diuretics. Explanation: Diuretics, ACE inhibitors, Angiotensin II receptor blockers should be held on the morning of surgery, especially if indication is CHF as there is an increased risk of hypotension. (A) Digoxin, which is useful in controlling the resting ventricular rate in atrial fibrillation in the setting of left ventricular dysfunction and CHF, can be used as adjunct to control AF. This should be continued throughout the perioperative period. (B) Acetaminophen may be continued throughout the peri-operative period. (C) Beta blockers should be continued the day of surgery. They are recommended in patients with or at risk for CAD undergoing no cardiac surgeries. The most pronounced benefit has been in high-risk patients undergoing vascular surgery.

Which of the following tendon to bone anchoring methods provides the most secure fixation? A. Interference fit-type anchors B. Screw-type anchors C. Screw and spiked washer D. Drill hole with tendon passed through and tied to itself

Correct answer: (D) Drill hole with tendon passed through and tied to itself. Explanation: Regardless of the bone anchor design, whether screw-type, interference fit-type, suture or even spiked washer, they just tie down the tendon to the surface of the bone. The bone tunnel technique with the tendon passed through the tunnel and sutured back onto itself is tremendously stronger as the tendon heals within the bone and not just on the bone's surface.

Which one of the following radiographic findings is less likely to be associated with hyperparathyroidism? A. Resorption and lysis of the distal tufts of the phalanges B. Subperiosteal resorption at the medial aspect of the middle phalanx shaft C. Vertebral endplate sclerosis which creates an alternating dense-lucent appearance D. Frayed and cup-shaped metaphyses with associated growth plate widening

Correct answer: (D) Frayed and cup-shaped metaphyses with associated growth plate widening. Explanation: Hyperparathyroidism is a disease characterized by increased osteoclastic activity due to elevated parathyroid hormone levels. These osteoclasts resorb bone and create areas of radiolucency/decreased density on x-rays. The bone that is lost is then replaced by fibrous tissue. There are several radiographic features that, when present, can help support the diagnosis of hyperparathyroidism. Answers (A), (B) and (C) are all findings that can occur in individuals with this disease, therefore making answer (D) the correct one. With the up-regulation of osteoclast activity, the majority of the radiographic features are going to involve bone loss and lysis. Both (A) and (B) deal will resorption. In particular, bone loss at the phalangeal distal tufts in the hands and feet is termed acro-osteolysis. Additionally, resorption beneath the periosteum is one of the earliest x-ray findings seen with hyperparathyroidism. The medial aspect of the shaft of the middle phalanges is an area that is typically affected. The loss of bone along the shafts creates a smudged out or toothbrush bristle appearance. While answer (D) describes the metaphyses as being frayed, irregular with a paintbrush appearance, this feature is not associated with hyperparathyroidism but rather rickets. Rickets is disease in children resulting from abnormalities in calcium, phosphorus or vitamin D metabolism. Osteoid is produced but fails to mineralize. This leads to widening of the physes and absence of the sclerotic zone of provisional calcification. The frayed metaphyses are due to disorganized spongy bone. As growth and cell hypertrophy continues at the physis, the cells push/extend into the weakened metaphysis creating a cupped appearance. Answer (C) is describing what is known as a rugger jersey spine. The increased density at the superior and inferior endplates of the vertebrae is caused by the accumulation of osteoid. While the osteoid is not mineralized, the excessive amount/volume of osteoid tissue deposited at that location is what is really causing the sclerotic bands seen on x-ray. Since the vertebral body between the opaque bands has a density that is less than the endplates, it gives the appearance of stripes (like what is seen on a rugby jersey).

Toe-off is a subphase of the gait cycle that occurs in the stance phase. What initiates and ends this subphase? A. Heel contact to toe off of the opposite limb B. Heel contact to toe of the same limb C. Opposite side Toe off to heel lift of the support foot D. Heel off of the support limb to full strike of the opposite limb

Correct answer: (D) Heel off to full strike of the opposite limb. Explanation: Toe off is that period from heel off of the support limb to full strike of the opposite limb. It occupies 20% of the gait cycle and 30% of the stance phase.

The formation of hammertoes after an intermetatarsal neurectomy is not a common complication, but it has been reported to occur. What is the explained anatomical structure that is altered during surgery is responsible for the formation of the digital deformity? A. plantar interossei B. dorsal interossei C. lumbricale D. inter metatarsal ligament

Correct answer: (D) Inter metatarsal ligament. Explanation: Dorsal contracture of the adjacent digits can occasionally be seen after neuroma surgery. This is due to the transection of the deep transverse intermetatarsal ligament. The rationale behind this association is rooted in the fact that the deep transverse intermetatarsal ligament functions as the fulcrum for the lumbricale tendon that stabilizes the toe. Of particular note, is that the question asks for the altered anatomical structure which, although involved, eliminates the lumbricale tendon.

When performing the Evan's procedure, placement of the osteotomy is critical in order to not violate either the anterior of middle facets. What anatomical landmark ensures bisection of the anterior and middle facets? A. 1 cm proximal to the calcaneocuboid joint B. 1.5 cm proximal to the calcaneocuboid joint C. Apex of the floor of the sinus tarsi D. Junction of the anterior superior calcaneus and the floor of the sinus tarsi

Correct answer: (D) Junction of the anterior superior calcaneus and the floor of the sinus tarsi. Explanation: The anatomical landmark for the Evan's procedure is the junction of the anterior superior calcaneus and the floor of the sinus tarsi. This allows success bisection of the anterior and middle facets. The actual distance of the osteotomy placement is 1-1.5 cm proximal to the calcaneocuboid joint, however the question asks for the anatomic landmark. (A) (B) The actual distance of the osteotomy placement is 1-1.5 cm proximal to the calcaneocuboid joint, however the question asks for the anatomic landmark. (C) If the osteotomy was placed at the apex of the floor of the sinus tarsi the the proximal location would result in the osteotomy being placed through the middle facet. This would result in post operative degenerative changes of the joint.

Which of the following is the best projection for visualization of the interspaces between the first and second cuneiform bones? A. Lateral foot B. Medial foot C. Medial oblique foot D. Lateral oblique foot E. Weight bearing lateral projection.

Correct answer: (D) Lateral oblique foot. Explanation: The interspaces between the first and second cuneiforms and first and second metatarsals can best be visualized using the lateral oblique projection. The medial oblique projection is used to demonstrate most tarsal bones and intertarsal spaces. Weight-bearing lateral projection is used to demonstrate the longitudinal arches of the foot.

Which of the following is true regarding the gait cycle? Stance phase lasts for approximately 38% of the entire gait cycle. Contact, midstance and swing are the three components of the stance phase of gait. The gastrocnemius/soleus complex is the primary initiator of heel off. Midstance begins with toe off of opposite foot and continues until heel off of the weight-bearing foot.

Correct answer: (D) Midstance begins with toe off of opposite foot and continues until heel off of the weight-bearing foot. Explanation: The midstance phase begins with toe off of the opposite foot and continues until heel off of the weight-bearing foot. (A) The stance phase of the gait cycle lasts for approxaimtely 62% of the entire gait cycle. The swing phase lasts for approximately 38% of the entire gait cycle. (B) The three phases of the stance phase of gait are contact, midstance and propulsion. Contact phase begins with heel strike and ends with toe off of the opposite limb. The midstance phase begins with toe off of opposite limb and ends with heel off of weight-bearing limb. Propulsion begins with heel off of weight-bearing limb and continues until toe off of weight-bearing limb which is when the swing phase begins. (C) The gastrocnemius/soleus complex make up the Achilles tendon. This is not the PRIMARY initiator of heel off. The main initiator of heel lift is momentum once the foot is resupinated.

During a work up of a 35 year old male with the complaint of forefoot discomfort, an irregularity in the medial cuneiform is noted. The patient does not complain of pain in the area of the medial cuneiform nor is there pain upon palpation to the medial cuneiform. Range of motion of the mid tarsal joint and tarsometatarsal joint is not painful. No swelling, warmth or color changes are noted around the area. He denies trauma or previous surgery. An MRI of the affected foot is ordered to further assess the irregularity. Pictured below is a sagittal image from the MRI study. What is the recommended treatment? A. 4-6 week immobilization in a CAM walker B. Bone stimulator C. Fixation of the fracture D. No treatment needed

Correct answer: (D) No treatment needed. Explanation: Key points of this question lie in the fact that there are no symptoms related to the irregularity nor is there history of trauma or surgery. This eliminates the possibility of an atrophic nonunion of the medial cuneiform. Likewise if the irregularity was caused by a stress fracture, pain and swelling would be present. Avascular necrosis would also cause pain as the bone started to crumble on itself. This is a classic example of a bipartite medial cuneiform which typically presents as a horizontal segmentation. As can also be seen on the MRI, articular cartilage is present on the adjacent sides of each segment. In this scenario no treatment is needed at this time and would only need to be revisited if the patient began to complain of pain in that area.

A 43 year old female presents 5 days after a neurectomy surgery and her entire third toe is completely purple. Closer examination of the toe and reveals a deeply purple, warm, blanch able third digit. They patient is obviously concerned. From the options below, what would be the best choice of initial treatment with her toe? A. amputation B. digital plethysmography C. hyperbaric oxygen D. observation

Correct answer: (D) Observation. Explanation: Arterial embarrassment after an inter metatarsal neurectomy is an uncommon, but very serious complication. The key to this question is the realization that the above scenario is not arterial compromise, but rather venous congestion of the toe. This is a benign, self-resolving condition that develops from tissue disruption from dissection. Sometimes it is associated with a tight dressing as well. The first three options all hinge on the misdiagnosis of an ischemic digit.

A 43 year old female presents to clinic complaining of pain in the ball of her right foot. Physical exam reveals a moderate hallux valgus deformity with a hypermobile first ray. Additionally, sagittal plane contracture is noted of the second and third digits and an adductovarus deformity is noted on the fourth and fifth digits. There is a medial deviation of the second digit, however the second digit does not contact nor underlap or overlap the hallux. There is a somewhat diffuse painful area under the third and fourth met head and in the 3rd interspace. Additionally there is pain with side to side compression of the right forefoot. Below is an ultrasound image of the patient's right foot. With the information provided and the clinical image below, what would be the provisional diagnosis to explain the patient's pain in the ball of her right foot: A. Metatarsal stress fracture B. Predislocation syndrome C. Capsulitis third tmp joint D. Neuroma

Correct answer: (D) Neuroma. Explanation: The foot described above is a foot prone to many problems. It is overpronated with a hypermobile first ray. It has lead to a hallux valgus deformity and pathology related to lateral overload of pedal pressure due to the insufficiency of the first ray to bear its portion of weight. All the conditions listed above are possible with this foot type. However, the key component in arriving to the correct answer lies in the diagnostic ultrasound picture. Highlighted by the yellow arrow is a sizable neuroma in the third innerspace (Morton's neuroma). It can be seen protruding plantarly beyond the metatarsal heads and occupying the entirety of the innerspace. In contrast, a metatarsal stress fracture cannot be diagnosed through this modality. Pre dislocation syndrome would be highly unlikely in this part of the foot. It typically affects the second tmp joint. Capsulitis of the third tmp joint is not visualized above. Thickening of the plantar capsule and/or accumulation of fluid under the metatarsal head is not present.

Please refer to the radiographic image below. What factors noted in the x-ray increase the probability of a hallux varus deformity? A. intermetatarsal angle B. hypermobility of the first ray C. proximal articular set angle D. over resection of the medial eminence

Correct answer: (D) Over resection of the medial eminence. Explanation: Pictured above is an opening wedge osteotomy. The correct answer to the question is that the surgeon performed an overly aggressive of the eminence. The archaic practice of eminence resection at the sagittal groove tends to remove too much of the metatarsal head. In the example above this is clearly evident. Furthermore even though the tibial sesamoid position is at best a 2, the medial aspect of the tibial sesamoid is peaking out from under the medial aspect of the first metatarsal head. The IM angle is acceptable. However it should be noted that a negative IM angle can lead to a hallux virus deformity. First real hyper mobility and PASA have little influence in the various deformity.

Out of the options below, which is the best description for the Gauthier's test for an intermetatarsal neuroma? A. Pain is produced when axial pressure is applied to the intermetatarsal space while the toes are held in extension. B. Pain is produced when dorsoplantar pressure of the suspected interspace and simultaneous side-to-side pressure of the forefoot is applied. C. Pain is produced with complete plantarflexion of the metatarsophalangeal and interphalangeal joints of the adjacent toes of the interspace suspected. D. Pain is produced when the metatarsophalangeal and the interphalangeal joints of the adjacent toes of the suspected interspace are completely extended.

Correct answer: (D) Pain is produced when the metatarsophalangeal and the interphalangeal joints of the adjacent toes of the suspected interspace are completely extended. Explanation: Gauthier's test was presented by G. Gauthier in 1979. Gauthier's test is best described as a test that produces pain when the metatarsophalangeal and the interphalangeal joints of the adjacent toes of the suspected interspace are completely extended. With complete plantarflexion of the metatarsophalangeal and interphalangeal joints of the adjacent toes of the interspace suspected neuroma pain is typically relieved. Pain is produced when axial pressure is applied to the intermetatarsal space while the toes are held in extension can also be found with an intermetatarsal neuroma, but also can be found with arthritic conditions. A positive Mulder's sign is when pain is produced when dorsoplantar pressure of the suspected interspace and simultaneous side-to-side pressure of the forefoot is applied.

A patient with history of diabetes mellitus and end stage renal disease is undergoing a dorsiflexory osteotomy of the first metatarsal due to recurrent plantar ulcerations. Which of the following anesthetics should be used for a local anesthetic block? A. Lidocaine B. Marcaine C. Prilocaine D. Procaine

Correct answer: (D) Procaine. Explanation: Procaine is an ester anesthetic. This group of anesthetics including procaine undergoes hydrolysis by plasma esterase in the blood. The other options are amide anesthetics which are metabolized by the liver and excreted by the renal system. (A) Lidocaine (xylocaine) is an amide anesthetic which is metabolized by the liver. It is fast acting and has a half-life of 1.5 to 2 hours. (B) Marcaine (bupivacaine) is longer acting than lidocaine but is also an amide anesthetic and metabolized by the liver. (C) Prilocaine is also within the amide group and is often used in conjunction with lidocaine for topical anesthesia.

Which of the following choices is an indication for a Jones suspension procedure? A. Rigid cavus foot B. Rigid plantarflexed first ray C. Hypermobile first ray D. Prophylaxis when both hallucal sesamoid bones are removed

Correct answer: (D) Prophylaxis when both hallucal sesamoid bones are removed. Explanation: The Jones suspension procedure is the transfer of the extensor hallucis longus through the head of the first metatarsal from medial to lateral. The extensor hallucis longus is transected at the interphalangeal joint and the distal remnant is sutured to the extensor hallucis brevis to maintain some active dorsiflexion of the hallux. Indications for the Jones suspension are: Flexible cavus foot; Flexible plantar flexed first ray; Prophylaxis when both hallucal sesamoids are removed. The procedure will not work if utilized on a rigid cavus foot or rigid plantar flexed fist ray. Additionally, a Jones Suspension would have no effect on stabilizing a hypermobile first ray.

Which agent listed below is safe to use on a patient who has known susceptibility to malignant hyperthermia? A. sevoflurane B. enflurane C. dacamethonium D. propofol

Correct answer: (D) Propofol. Explanation: Malignant hyperthermia is a genetic disorder involving the ryanodine receptor. Most individuals have little to know symptoms of the condition prior to exposure of a trigger. The most common triggers are the volatile inhalation anesthetics. These include sevoflurane, halothane, enflurane, desflurane, and isoflurane. Also included are the depolarizing muscle relaxants decamethonium and suxamethonium. More recently it has been suspected that environmental heat, as well as stress caused by physical exercise might also be triggers. Local anesthetics, opiates, and non-depolarizing muscle relaxants are all safe for those susceptible to malignant hyperthermia. In addition propofol, ketamine, nitrous oxide, barbiturates, benzodiazepines, and etomidate are also safe.

A 52 year old female has a unilateral flatfoot of 2 months duration. The patient has a history of multiple corticosteroid injections by a previous podiatrist for the diagnosis of "Os Tibiale Externum." What is the most likely cause of the patient's unilateral flatfoot? A. Rupture of the plantar fascia B. Rupture of the peroneus brevis tendon C. Rupture of the spring ligament D. Rupture of the posterior tibial tendon

Correct answer: (D) Rupture of the posterior tibial tendon. Explanation: The key to this question is the recognition that the clinical entity describe above is a tendinopathy of the posterior tibial tendon irrespective of the presence or absence of an os tibiale externum. The initial symptomatology would stem from the tendon itself and not the accessory ossicle. The presence of the accessory ossicle alters the insertion orientation and the mechanical pull of the posterior tibial tendon. This in turn decreases the efficiency of the tendon's ability to support the arch. The resultant instability of the arch puts an excessive strain on the disadvantaged tendon and creates inflammation and pain. The corticosteroid injection likely further weakened the already compromised posterior tibial tendon. With the continued strain on the tendon from over-pronated ambulation, the likely cause of the recent unilateral flatfoot is rupture of the posterior tibial tendon.

Which of the following techniques is the most useful for the surgical treatment of long or irregular hypertrophic scars? A. Z-plasty B. W-plasty C. V-Y plasty D. Short wavy incision method E. Geometric broken line closure

Correct answer: (D) Short wavy incision method. Explanation: The W-plasty, and its variation, the short wavy incision method (SWIM), along with the geometric broken line closure are utilized for the resection and subsequent repair of irregular or long hypertrophic scars. The SWIM allows resection of irregular scar material while allowing more lengthening than the W-plasty. The Z-plasty and V-Y plasty techniques are utilized for release of skin contractures.

A 45 y/o patient presents with a painful bunion. Clinically, there is full ROM at the first MPJ, but there is tracking. There is greater than ten degrees of ankle DF with the knee flexed and two degrees of ankle DF with the knee extended. Radiographically, the HI angle is 8 degrees; DASA 6 degrees; PASA 15 degrees; HAA 25 degrees; IM angle 16 degrees; and MAA of 15 degrees. The best choice of surgical procedures is: A. Silver; Reverdin-Green; TAL B. Modified McBride; proximal Akin; Gerbert-Massad; gastrocnemius recession C. McBride; Juvara A; Lepird; gastrocnemius recession D. Silver; Volger Offset-V; Gastrocnemius recession

Correct answer: (D) Silver; Volger Offset-V; Gastrocnemius recession. Explanation: The HAA is abnormal (10-15 degrees); a capsule-tendon balance procedure will correct this. The Silver is not just removal of the bump, as most think. Silver's classic article referred to a lateral release. Gerbert refers to tight lateral structures as a cause for tracking; thus the need for a lateral release. The Vogler shaft osteotomy can reduce both the intermediate increase in the IM angle (0-8 in a rectus foot) and the high PASA (0-8 degrees). Vogler states his osteoplasty can reduce an IM angle up to 17-18 degrees, and by swiveling the fragment, reduce a PASA as high as 30 degrees. The Silfverskiold test indicates a tight gastrocnemius, not tendo- Achilles, thus the indication for a gastrocnemius recession, not TAL. (A) The Silver capsule-tendon balance procedure will correct the high HAA. The Reverdin- Green has a plantar shelf to protect the sesamoids, but the lateral hinge is intact, reducing the PASA, but not the IM angle. A TAL would be indicated if, via the Silverskiold test, there is lack of adequate ankle DF with both the knee flexed and extended. (B) A modified McBride procedure is pretty much synonymous with a true Silver. It is indicated for a high HAA, normal being 10-15 degrees. A proximal Akin addresses a high DASA; in this case, the DASA is within normal values of 0-8 degrees; a proximal Akin is not necessary. The Gerbert-Massad is a bi-correctional Austin modification, so that could be used for the high PASA and intermediate increase in the IM angle. The gastrocnemius recession is indicated. (C) The true McBride and gastrocnemius recession are indicated. The Juvara A is a closing base wedge osteotomy; it could be used, but a more stable distal osteotomy would be preferred. The Lepird is an osseous procedure for metatarsus adductus. To evaluate the "true" IM angle with a metatarsus adductus component, use the formula IMtrue=IMmeasured+(MAA-15). In this case, the formula is not necessary; a MAA of 15 degrees or less will not influence procedure selection. Boards are looking to see if you appreciate any metatarsus adductus; if present, consider a more proximal osteotomy.

Which of the following nerves are not blocked during a typical 1st ray MAYO block before bunion surgery? A. Deep peroneal nerve B. Medial dorsal cutaneous nerve C. Medial plantar nerve D. Sural nerve

Correct answer: (D) Sural nerve. Explanation: The Mayo block is the most commonly used block before a distal bunion procedure. The mayo block hits the deep peroneal nerve in the 1st interspace (always aspirate as the artery runs in close proximity to it), the medial dorsal cutaneous nerve (1st digital branch), the medial plantar nerve as well as the saphenous nerve. The sural nerve is on the lateral aspect of the foot and is not blocked in a typical 1st ray Mayo block.

Which of the following deformities is the most common cause of peroneal spastic flatfoot? A. Convex pes valgus B. Tarsal tunnel syndrome C. Charcot marie tooth D. Tarsal coalition

Correct answer: (D) Tarsal coalition. Explanation: Any condition that causes restriction of the subtalar joint, can cause peroneal spastic flatfoot, most commonly this is a tarsal coalition. Convex pes valgus is a congenital disorder that can restrict subtalar joint motion, however it is rare. Charcot marie tooth and tarsal tunnel syndrome do not block subtalar joint motion.

What radiographic projection is conducted when the patient is required to go into a "ski jump" position, with the central ray direction on the posterior aspect of the foot between the Achilles tendon insertion and the ankle joint? A. The posterior anterior sesamoid axial projection B. The Lewis method C. The Isherwood method D. The calcaneal axial projection

Correct answer: (D) The calcaneal axial projection. Explanation: With the tube head angulation at 25 degrees from the vertical, the heel placed so that its posterior aspect is about 1 to 1 and 1⁄2 inches away from the edge of the cassette closest to the x-ray tube, the central ray should be on the posterior aspect of the foot between the Achilles tendon and the ankle joint, also known as the dorsoplantar calcaneal axial projection.

All of the following statements about the remodeling or maturation phase of wound healing are accurate except which one? It is the final stage of wound healing. It begins at approximately 3 weeks after the injury and can last up to 1 year. This phase is characterized by the transformation of the previously highly cellular matrix tissue to a relatively acellular mass of collagen bundles which provide tensile strength to the residual scar. The collagen bundles consist primarily of type IV collagen.

Correct answer: (D) The collagen bundles consist primarily of type IV collagen. Explanation: The maturation or remodeling phase of wound healing begins approximately 3 weeks after injury and can last up to 1 year. It is characterized by the formation of a lattice-like network of collagen that makes up scar tissue. The previously highly cellular granular tissue of fibroblasts and capillaries transforms into a relatively acellular mass of type I collagen bundles. These bundles provide increased tensile strength to the residual scar.

What is the characteristic radiographic feature of the bipartite medial cuneiform on radiographs? When the bipartition is incompletely divided into two segments at its anterior margin, but is fused centrally. When indentations are seen along the articular surfaces of the bipartite medial cuneiform from posterior to anterior. When the bipartition is not superimposed to the other bone shadows. The transverse joint space has to be identified in the center of the medial cuneiform, with the joint space incomplete from anterior to posterior.

Correct answer: (D) The transverse joint space has to be identified in the center of the medial cuneiform, with the joint space incomplete from anterior to posterior. Explanation: The bipartition of the medial cuneiform classically divides the bone into upper and lower halves. Best seen in lateral view, there could be a rare variation dividing the medial cuneiform into small posterosuperior to a large anteriorinferior one. Even not readily identified, it is fully superimposed to the remaining cuneiform bones and easily mistaken for other bone shadows. However, transverse joint space identified in the center of the medical cuneiform is its characteristic radiographic feature. This joint space is complete from anterior to posterior. A variation can be encountered when the bipartition is incomplete. The medial cuneiform may be incompletely divided into two segments at its anterior and/or posterior margins but is fused centrally.

How many tendons are found with the extensor digitorum brevis? A. Five tendon slips each one for the 2nd, 3rd, 4th and 5th digits B. Four tendon slips, each one for the 1st, 2nd, 3rd, 4th digits C. Three tendon slips, each one for the 2nd, 3rd, 5th D. Three tendon slips, each one or the 2nd, 3rd, 4th digits

Correct answer: (D) Three tendon slips, each one or the 2nd, 3rd, 4th digits. Explanation: The tendons of the extensor digitorum brevis are found on the lateral aspect of the extensor digitorum longus tendons 2, 3, 4, within the extensor expansion, the muscle belly passes in a distomedial direction and is relatively short; the tendons then pass to the dorsolateral aspect of the respective toe.

A 27 year old male involved in a motorcycle injury presents to the emergency department with the injury pictured below. There appears to be adequate soft tissue coverage for closure. Posterior tibial pulse is palpable and dorsalis pedis pulse is nonpalpable. What Gustilo classification would the injury below be best described? A. Type II B. Type IIIA C. Type IIIB D. Type IIIC

Correct answer: (D) Type IIIC. Explanation: The injury pictured above is a clear-cut example of a Gustilo IIIC fracture. Regardless of any other aspect, the fact that the dorsalis pedis pulse is nonpalpable means this is a clear-cut Gustilo IIIC open fracture. Gustilo III fractures are best characterized as an open fracture, highly contaminated, with a wound > 5 cm. There is extensive soft tissue damage including skin, muscle, nerve, and vasculature. A large crush component is seen. Gustilo III injuries can further be subdivided into three variants. IIIA is when there is adequate soft tissue coverage and it is a high-energy trauma. IIIB is when soft tissue coverage is not adequate for coverage and there is extensive periosteal stripping of exposed bone. IIIC is an open fracture with arterial injury regardless of bone or soft tissue injury. Furthermore type I is an open fracture with a clean wound < 1 cm, with little soft tissue involvement and no crush component. The wound is usually from bone protrusion inside to out. The fracture is usually short, short oblique, or transverse. Type II is an open fracture moderately contaminated, with a wound > 1 cm. There is no extensive soft tissue damage, skin flaps, or avulsions. A minimal crush component and moderate fracture comminution only.

When performing an osteotomy to correct tailor's bunion deformity, which of the following are complications of entering the metatarsophalangeal joint? A. Nonunion or malunion B. Aseptic necrosis C. Staking of metatarsal head D. Instability of metatarsophalangeal joint E. All of the above

Correct answer: (E) All of the above. Explanation: All of the above complications, as well as postoperative neuritis, are potential complications of entering the fifth metatarsophalangeal joint when performing osteotomy for surgical treatment of tailor's bunion. Unless associated pathology of the joint or of the fifth digit is also to be corrected, it is preferable to avoid entering the metatarsophalangeal joint.

Which diagnostic maneuver strongly suggests a neurogenic origin to a patient's heel pain? (Select all that apply) A. Plantarflexion and inversion B. Plantarflexion and eversion C. Dorsiflexion and inversion D. Dorsiflexion and eversion

D - Dorsiflexion and eversion,A - Plantarflexion and inversion. Correct answer: (A) (D) Plantarflexion and inversion and dorsiflexion and eversion. Explanation: Plantarflexion and inversion of the foot increases the pressure within the ports pedis. This in turn can place pressure on the posterior tibial nerve and its branches and is highly suggestive of a neurogenic origin to a patient's heel pain. This maneuver is known as Phalen's test. Dorsiflexion and eversion of the foot increases the pressure within the ports pedis. This in turn can place pressure on the posterior tibial nerve and its branches and is highly suggestive of a neurogenic origin to a patient's heel pain. (B) Plantarflexion and eversion of the foot does not have any diagnostic effect in regards to heel pain. (C) Dorsiflexion and inversion of the foot does not have any diagnostic effect in regards to heel pain.

In type 2 DM managed with insulin, what should be the regimen peri-operatively? (Select all that apply) If patient will eat postoperatively, basal insulin should be given. If on long acting insulin, if the patient usually takes it in the morning, it should be withheld. If on long acting insulin, if the patient usually takes it in the morning 50 to 100% of the usual dose can be given. If using NPH (intermediate-acting) 1⁄2 to 2/3 of the usual morning dose can be given to avoid hyperglycemia. The usual insulin treatment has to be withheld postoperatively until renal function has returned to normal.

D - If using NPH (intermediate-acting) 1⁄2 to 2/3 of the usual morning dose can be given to avoid hyperglycemia.,C - If on long acting insulin, if the patient usually takes it in the morning 50 to 100% of the usual dose can be given.,A - If patient will eat postoperatively, basal insulin should be given.. Correct answer: (A) (C) (D) Explanation: If it is anticipated the patient will be able to eat postoperatively, basal insulin is still given on the morning of surgery, if the patient is on long acting insulin such as glargine insulin, and the patient usually takes the dose in the morning, 50 to 100% of the usual dose can be given. If the patient is on intermediate-acting insulin like NPH, 1⁄2 to 2/3rd of the usual morning dose is given to avoid peri-procedural hyperglycemia. For fluid management, dextrose-containing IV fluids may be required to avoid hypoglycemia. If the patient is undergoing a major procedure, it will typically require an insulin drip preoperatively. And finally the usual insulin treatment can be reintroduced once oral intake is established postoperatively.

Which of the following diabetic medications should be withheld the morning of surgery? (Select all that apply) A. Incretins B. Pioglitazones C. Chlopropamide D. Metformin

D - Metformin,C - Chlopropamide. Correct answer: (C) (D) Chlopropamide and Metformin. Explanation: The biguanide, metformin, which has been associated with the development of lactic acidosis, should be withheld 24 hours preoperatively and restarted 48 to 72 hours postoperatively once normal renal function has been documented. Long acting sulfonylureas are best withheld 48 to 72 hours preoperatively to avoid potential hypoglycemia. Pioglitazone is a thiazolidinedione and can be held on the morning of surgery as it is given once daily with a duration of action of 24 plus hours. Secretagogues must be held preoperatively.

Reliable indicators for the diagnosis of compartment syndrome include: (Select all that apply) A. Any compartment pressure greater than 20mm Hg B. A compartment pressure 10-30mm Hg below the patient's diastolic blood pressure C. Absent pedal pulses D. Pain with passive stretch of the toes

D - Pain with passive stretch of the toes,B - A compartment pressure 10-30mm Hg below the patient's diastolic blood pressure. Correct answer: (B) (D) A compartment pressure 10-30mm Hg below the patient's diastolic blood pressure and pain with passive stretch of the toes. Explanation: A compartment pressure greater than 30mm Hg indicates compartment syndrome. Factoring in the patient's BP accounts for hypotension, which may be seen with polytrauma patients. Absent pedal pulses are a poor indicator of compartment syndrome. Pain with passive stretch of the toes is secondary to evolving ischemia to the intrinsic muscles.

From the options listed, which statements are correct regarding the medial heel skive technique? (Select all that apply) A. Requires an extrinsic forefoot post B. Cannot be used with a shallow heel cup C. Helps control excessive supination D. Shifts the ground reactive force medially

D - Shifts the ground reactive force medially,B - Cannot be used with a shallow heel cup. Correct answer: (B) (D) Cannot be used with a shallow heel cup and shifts the ground reactive force medially. Explanation: A standard or preferably deep heel cup is required for the medial heel skive technique. If the heel cup is left too shallow that the heel will slip out of the device and not only make in ineffective, but uncomfortable as well. The medial heel skive is a technique used on the positive cast where a 2,4, or 6 mm build up is performed on the medial heel. This "shallows" out the medial portion of the heel cup in the orthotics and shifts the ground reactive force more medially in order to resistant calcanea eversion and unlocking of the midtarsal joint. The medial heel skive technique, developed by Dr. Kevin Kirby, does indeed shift the ground reactive force more medially in order to resistant calcanea eversion and unlocking of the midtarsal joint. (A) There is no prerequisite posting requirement of the forefoot as it pertains to the medial heel skive modification. (C) The medial heel skive technique is a tremendously useful modification to help control excessive pronation. The skive can be ordered in 2,4,or 6 mm increments. The larger the skive the greater the pronator control.

What is the distinct difference between a Jahss type II-A and type II-B first metatarsophalangeal joint dislocation? (Select all that apply) Type II-A injuries are reducible and Type II-B injuries are nonreducible. Type II-A injuries do not have joint interposition of the sesamoids while type II-B do have joint interposition by the sesamoids. Type II-A injuries do not have a sesamoid fracture while type II-B does have a sesamoid fracture. Type II-A injuries have a ruptured intersesamoidal ligament while Type II-B have an intact intersesamoidal ligament.

D - Type II-A injuries have a ruptured intersesamoidal ligament while Type II-B have an intact intersesamoidal ligament.,C - Type II-A injuries do not have a sesamoid fracture while type II-B does have a sesamoid fracture.. Correct answer: (C) (D) Explanation: In the Jahss classification for first metatarsophalangeal joint dislocations a type II- A injury does not have a sesamoid fracture while type II-B does have a sesamoid fracture. Type II-B injuries, which have a sesamoid fracture, do not have a ruptured intersesamoidal ligament. Instead of the ligament rupturing, the sesamoid fractured. The fracture is an avulsion type from further hyperextension of the joint. The Jahss classification is as follows: Type I - dislocation without sesamoid fracture or intersesamoidal ligament rupture; Type II A - dislocation with intersesamoidal ligament rupture and without a fractured sesamoid; Type II-B dislocation with sesamoid fracture and intact intersesamoidal ligament. (A) The Jahss classification does not relate to the reducibility of first metatarsophalangeal joint dislocations. (B) The Jahss classification does not relate to joint interposition of the sesamoid apparatus in the first metatarsophalangeal joint.

You would like to proceed with a subtalar joint fusion using an external fixation of one of your patients below. Which one of them would require anticoagulation with bridging therapy? (Select all that apply) A. A patient with a single venous thromboembolic event 7 months ago B. Protein C and S deficiency C. Diabetes without complications D. Factor V Leiden mutation E. Antiphospholipid antibody syndrome

E - Antiphospholipid antibody syndrome,D - Factor V Leiden mutation,B - Protein C and S deficiency. Correct answer: (B) (D) (E) Protein C and S deficiency, factor V Leiden mutation and antiphospholipid antibody syndrome. Explanation: Perioperative heparin bridging is recommended for patients with a history of VTE within the preceding 3 months and can be considered for VTE events within the past 3 to 6 months. A single VTE event that occurred more than 6 to 12 months ago is unlikely to justify bridging therapy. Patients with a prior VTE event and a known thrombophilia (protein C and S deficiency, antiphospholipid syndrome, and homozygous factor V Leiden mutation) are also candidates for heparin bridging. Even without a diagnosed defect, if a patient has a clinically apparent thrombophilia based on recurrent or unprovoked VTE events, perioperative bridging should be employed.

A pre-operative EKG can be stratified according to "recommended" vs. "reasonable." In which scenario is a pre-operative EKG considered "reasonable"? A. A patient older than 65 y/o. B. A patient with one or more clinical risk factors undergoing intermediate-risk surgery. C. An asymptomatic patient undergoing low-risk surgery. D. A patient with one or more clinical risk factors undergoing vascular surgery.

Explanation: The recommendation of the American College of Cardiology states an EKG is reasonable but not recommended with a patient undergoing cardiac endarterectomy, prostate surgery, head and neck surgery, or orthopedic surgery; examples of intermediate risk surgeries where the risk of a cardiac event is 1-5%. Clinical risk factors include CAD, CHF, diabetes, or renal insufficiency. (A) An EKG is recommended by the Institute for Clinical Systems Improvement, for patients older than 65. (C) Unless the H & P reveals a high risk patient with a risk greater than 5%, the Institute for Clinical Systems Improvement does not recommend an EKG in this patient subset. (D) The American College of Cardiology recommends a preoperative EKG in a patient with risk factors mentioned above, and undergoing vascular, noncardiac, surgery.

A 9 year old girl presents to your office with a chief complaint of heel pain worst after long periods of activity. The child is on her town's travel soccer team and plays sports year round. On examination, the pain that the girl is experiencing is around the growth plate of the heel which is still open. The doctor decides he would like to attempt physical therapy in addition to heel lifts. Which of the following modalities would be contraindicated in this patient? A. Whirlpool B. Paraffin Wax C. Ultrasound D. Moist Heat Packs

Correct answer: (C) Ultrasound. Explanation: What needs to be picked up on in this questions is the child's age and the fact that she still has open growth plate in the area of chief complaint. Ultrasound is contraindicated over open growth plates in children as it can cause a premature closure of the growth plate. Thus this is the answer choice for this question. Paraffin wax, moist heat packs and whirlpool can all be utilized in children as a whole as long as there is supervision by an adult.

A physician purchases a new computer for his practice and he would like to allow his son to use the old computer. Which of the following is true? Computers used for storage of electronic protected health information cannot be reused. The computer can only be reused by employees of the practice who are participating in patient care. The computer hard drive must be replaced and the old hard drive should be physically destroyed. The physician can allow his son to reuse the computer after he deletes any protected health information manually. The computer may only be reused after using software or hardware to overwrite the media or until it is purged.

Correct answer: (E) The computer may only be reused after using software or hardware to overwrite the media or until it is purged. Explanation: Reuse or disposal of computers or other electronic media used to store protected health information can only occur after the covered entity has removed protected health information according to policies and procedures established in the HIPAA Privacy and Security Rules. This includes purging or using software or hardware to overwrite the media. (A) Reuse is permitted after adequate steps have been taken to remove any protected information stored on the device. (B) Reuse by anyone is permitted after adequate removal of protected health information. (C) Overwriting media or purging are two options for removal of protected health information that are acceptable. (D) Simple deletion is not considered adequate protection of patient health information stored on electronic media.

Which of the following is consistent with radiographic appearance of chondrosarcoma? A. Well-demarcated shallow cortical defect B. Saucerization of underlying bone C. Central medullary lesions that calcify over time D. Cortical thickening and destruction E. Minimal endosteal erosion (<50% width of cortex)

Answer:D. Cortical thickening and destruction Explanation: Enchondromas appear as well-defined, lucent, central medullary lesions that calcify over time, from 1 to 10 cm in size. They appear first in the metaphyseal location, then become more diaphyseal as the long bone grows. There is minimal endosteal erosion, compared to endosteal erosions and scalloping over greater than 50% of the width of the cortex in chondrosarcomas. Chondrosarcomas display cortical thickening and destruction. Endosarcomas have cortical expansion. Thinning of the cortex may be present or the lesions may have a purely lytic appearance. Osteochondromas display well-demarcated shallow cortical defects and may display saucerization of the underlying bone.

A 50 year male patient with a long standing history of diabetes that he claims is well controlled is scheduled for fusion of the 1st hallux interphalangeal joint of the right foot, secondary to hallux malleus, and pre-ulcerative lesion of the dorsal interphalangeal. You would like to know the severity of the disease before proceeding, what are the laboratory values that should be ordered? (Select all that apply) A. Hemoglobin A1C B. Measurement of renal function C. CBC with differential D. Albumin levels E. PT and INR

B - Measurement of renal function,A - Hemoglobin A1C. Correct answer: (A) (B) Hemoglobin A1C and measurement of renal function. Explanation: It is imperative to do careful assessment for all patients. The diabetic patient requires a systematic approach as the disease affects numerous end organs. Furthermore to discern the degree of severity, there are several laboratory values that can help to do just that. 1) Glycosylated hemoglobin (HbA1C) values can reflect the degree of hyperglycemia to which red blood cells have been exposed. Because the average lifespan of red blood cells is 120 days, the HbA1c level is an indicator of glycemic levels over that period of time (or more strongly related to 8-12 weeks). A normal value is up to 6%, but some values greater than 5.5% may have impaired glucose tolerance. The goal of the American Diabetes Association for control of diabetes is an Hb1Ac less than 7%, which can be considered as adequate control. Values more than 8% correspond to average blood glucose levels greater than 180 mg/dl and indicate poor glycemic control. Renal function is also the other important laboratory value. Because diabetes can lead to renal failure, measurement of renal function can give insight into the severity of the disease. Diabetic patients with renal insufficiency are at greater risk for hypoglycemia given the prolonged life of insulin and sulfonylureas. Identifying these patients will help make a plan for glucose monitoring sometimes as frequent as every 30-60 minutes.

It is well accepted that extra-neural scarring is a known complication following tarsal tunnel surgery. By what means does extra-neural scarring cause a negative outcome? (Select all that apply) A. Facilitation of nerve gliding B. Nerve ischemia C. Mechanical circumferential constriction D. Vascular embarrassment

C - Mechanical circumferential constriction,B - Nerve ischemia. Correct answer: (B) (C) Nerve ischemia and mechanical circumferential constriction. Explanation: Post-operative extra-neural scarring can cause ischemia of the nerve by disturbing the delicate vasovasorum thereby compromising the vascular supply to the nerve. Post-operative extra-neural scarring can cause mechanical circumferential constriction on the nerve causing an entrapment neuropathy. (A) In contrast, extra-neural scarring impairs nerve gliding. This impairment of the gliding action of never adds to the negative symptomatology of recurrent tarsal tunnel syndrome. (D) Extra-neural scarring does not create vascular embarrassment.

What would be the appropriate rigidity for an orthotic made of the treatment of metatarsalgia? (Select all that apply) A. Flexible B. Semi-rigid C. Rigid D. Accommodative

C - Rigid,B - Semi-rigid. Correct answer: (B) (C) Semi-rigid and rigid. Explanation: A semi-rigid device and rigid device provides basically an "intrinsic offloading" quality as the foot is fully supported by the orthotic. (A) A flexible device is not appropriate as it will compress with loading. This will cause loss of contact of the device with the arch of the foot increasing the force at the metatarsal heads. (D) Accommodative devices do not particularly correlate with a specific rigidity as within the class of accommodative devices have varying rigidities as well.

When evaluating for hallux limitus on a lateral radiograph, which of the following would likely not be present? A. Loose body B. Dorsal metatarsal osteophyte C. Periarticular osteopenia D. Elevated first metatarsal

Correct answer: (C) Periarticular osteopenia. Explanation: On the lateral radiograph, dorsal first metatarsal metaphyseal shaft osteophytes and loose bodies may be present. This view is also used to evaluate the presence of an elevated first metatarsal in relationship to the lesser metatarsals. The central diaphyseal axes of the first and second metatarsals are marked and it has been noted up to 5 mm of elevation is normal.

Which of the following pediatric deformities would NOT show an increase in the talocalcaneal angle? A. Calcaneovalgus B. Global cavus C. Congenital pes valgus D. Accessory Navicular

Correct answer: (B) Global cavus. Explanation: A cavus foot is a high arch foot that shows a decrease in the talocalcaneal angle and thus an increase in articulation between the talus and navicular as well. (A) Calcaneovalgus is one of the most common abnormalities in foot in children. It is a form of flatfoot that tends to be seen at birth. Since it is a type of flatfoot, it does show an increase in talocalcaneal angle. (C) Congenital pes valgus is another name for vertical talus. This is considered a rigid flat foot and thus will show an increase in the talocalcaneal angle. (D) Accessory navicular causes a flexible flatfoot and will thus show an increase in the talocalcaneal angle.

What is a clinical distinction that can be made between vertical talus and oblique talus? The oblique talus deformity is reducible and the vertical talus deformity is not reducible. The vertical talus deformity is reducible and the oblique talus deformity is not reducible. The oblique talus deformity has a less everted hindfoot than the vertical talus deformity. Resupination during swing phase of gait is observed in the oblique talus deformity whereas in the vertical talus deformity there is no resupination during swing phase.

Correct answer: (A) The oblique talus deformity is reducible and the vertical talus deformity is not reducible. Explanation: A critical clinical distinction to make is that the oblique talus deformity is reducible and the vertical talus deformity is not reducible. This can and should be confirmed radiographically. (B) The vertical talus deformity is reducible and the oblique talus deformity is not reducible is not correct. In fact, the converse statement is true. (C) Both the oblique talus deformity and the vertical talus deformity possess a valgus hindfoot. (D) Neither deformity undergoes supination during the swing phase of gait.

During the contact period the intrinsic muscles are inactive. However, the extrinsic muscles which include the posterior tibial, peroneus longus and brevis, flexor digitorum longus and brevis, and posterior tibial due to its insertion has a prime effect on decelerating which joint during the contact period? A. Subtalar joint B. Midtarsal joint C. Talonavicular joint D. Ankle joint

Correct answer: (A) Subtalar joint. Explanation: The function of the posterior tibial muscle is: 1. Decelerates pronation at the subtalar joint 2. Decelerates anterior movement of the tibia 3. Decelerates internal rotation of the tibia. The posterior tibial has a prime effect on decelerating subtalar joint pronation owing to its perpendicular angle relative to the subtalar joint axis combined with its long lever arm. The posterior tibial has approximately twice the muscle leverage to supinate the subtalar joint as does the soleus. Electromyographic studies show that this muscle exhibits two peaks, one during the contact phase of gait and the other during active propulsion. Since the muscle inserts proximally on the leg, it has a substantial effect on decelerating internal rotation and forward movement of the tibia.

What muscle is the primary stabilizer of the first ray during midstance? A. Anterior tibialis B. Peroneus Longus C. Posterior Tibialis D. Extensor Hallucis Longus

Correct answer: (B) Peroneus Longus. Explanation: The peroneus longus is the primary stabilizer of the first ray during midstance in the posterior, lateral, and plantar directions. In addition, it maintains the stability of the midfoot along with the posterior tibialis by compressing the tarsus. It initiates its contraction during the middle of midstance phase. It has a significant lever arm for pronating the midtarsal joint and therefore stabilizes the entire medial forefoot against the ground.

A 68 year old male presents with a history of left foot pain. He is diagnosed with a flexibly contracted left 4th digit. A heloma molle is noted at the proximal interphalangeal joint. The patient has tried shoe modifications with no resolution of his symptoms. He has elected to undergo a 4th digit arthroplasty with an implant. His past medical history is significant for a 40 pack-year history of tobacco use, COPD, and no other major medical history. Which of the following should be ordered as part of the pre-operative testing? A. Chemistry profile, ECG B. Complete blood count, chest radiographs C. Coagulation profile, ECG D. Coagulation profile only

Correct answer: (A) Chemistry profile, ECG. Explanation: If the patient has a history of hypertension, diuretic use, chronic obstructive pulmonary disease or obstructive lung disease, blood loss, renal disease, or chemotherapy, a chemistry profile should be ordered. An ECG should be completed as well, and is recommended in men over 40 years of age. (B) Complete blood count should be performed in patients with a recent history of blood loss, a history of fatigue, dyspnea on exertion, liver disease, and signs of coagulopathy. A chest radiograph should be obtained if there is a history of pulmonary disease. In this case, the patient does have a history of pulmonary disease, but the patient does not require a complete blood count. (C) (D) Indicated if the patient is receiving anticoagulant therapy, has a family or personal history that suggests bleeding disorders, or has evidence of liver disease.

Which of the following options below is an indication for a peroneus longus to peroneus brevis transfer? A. unstable lateral ankle B. hypermobile first ray C. forefoot varus D. first metatarsal overload

Correct Answer: D. First metatarsal overload. Explanation: The peroneus longus to peroneus brevis transfer involves transecting the peroneus longus at the level of the cuboid and tenodesing it into the peroneus brevis. The distal stump of the longus is anchored into the cuboid to provide static stabilization of the first metatarsal. The goal is to offload the first ray. Indications for the procedure are first metatarsal overload with painful callus or ulceration, peroneus brevis weakness in cases such as Charcot-Marie-Tooth, peroneus brevis tears and when a peroneus brevis tendon was ill previously used for a lateral ankle stabilization. The transfer of the longus will provide limited benefit in a foot with a rigid hindfoot varus or forefoot valgus deformity if the transfer is not accompanied by an osseous procedure.

During routine pre-operative evaluation for elective hammertoe repair, a chest x-ray is recommended for which of the following patients? 72 yo M with a history of well-compensated congestive heart failure whose last chest x-ray was 8 months ago 45 yo F with a history of uncontrolled T2DM whose pre-operative blood sugar measures 185 18 yo F with history of GERD, which is controlled with medication 68 yo M with no significant medical history

Correct answer: (A) 72 yo M with a history of well-compensated congestive heart failure whose last chest x-ray was 8 months ago. Explanation: A pre-operative chest x-ray is not routinely required for many podiatric surgical patients. Pre-operative chest x-rays should be obtained in patients where acute cardiopulmonary disease is suspected based on clinical findings during pre-operative physical examination. Chest x-rays should also be obtained pre-operatively on patients with a chronic cardiopulmonary condition, such as congestive heart failure, who are elderly (greater than age 70), and whose most recent chest x-ray is more than 6 months ago.

What is the role of pretibial muscles (anterior tibial, extensor digitorum longus, extensor hallucis longus) at contact period of the gait cycle in normal gait? A. Decelerate the plantarflexion of the ankle B. Accelerate plantarflexion of the ankle C. Provide a stable midtarsal joint for shock absorption D. Accelerate pronation and provide even weightbearing from the medial to the lateral side of the foot

Correct answer: (A) Decelerate the plantarflexion of the ankle. Explanation: Role of pretibial muscles (anterior tibial, extensor digitorum longus, extensor hallucis longus) at contact period of the gait cycle in normal gait: 1. Decelerate plantarflexion of the ankle 2. Decelerate pronation and provide even weightbearing from the lateral to the medial side of the foot 3. Provide an unstable midtarsal joint for shock absorption and adaptation 4. Absorb impact loads from the floor At contact, all the muscles are active and reach their peak activity prior to foot flat. The anterior tibial generates the largest torque around the ankle joint, followed by the extensor digitorum longus and the extensor hallucis longus. At heel contact, the subtalar joint is supinated. The anterior tibial assists in the deceleration of subtalar joint pronation.

When dissecting for a Lapidus procedure, what structure must be avoided when exposing the first metatarsocuneiform articulation? A. deep perforating artery B. deep peroneal nerve C. anterior tibial tendon D. saphenous nerve

Correct answer: (A) Deep perforating artery. Explanation: This question focuses on the understanding of the surgical anatomy of the first metatarsocuneiform joint. Several significant anatomic structures are in the nearby vicinity. The saphenous nerve runs in the subcutaneous tissue dorsal medial to the joint. The anterior tibial tendon is just deep to the subcutaneous tissue. While it does cross the joint in a proximal lateral to distal medial orientation, it is easily retracted medially and out of the way of the joint. The deep peroneal nerve also travels in the subcutaneous tissues just lateral to the joint and is easily retracted in the soft tissue envelope. On the other hand the deep perforating artery travels from dorsal to plantar intimately associated with the lateral aspect of the first metatarsocuneiform articulation. If not careful it would be easy for the surgeon to dissect too aggressively laterally and lacerate the artery. Not only would this causes a significant bleeding situation but the tight osseous confines in which the artery travels would make accessing it very difficult.

Which of the following is seen with Talipes Equinovarus? A. Equinus position of the entire foot to the leg B. Valgus component of entire foot to leg C. Talipes component of entire foot to leg D. Abduction of forefoot to rearfoot

Correct answer: (A) Equinus position of the entire foot to the leg. Explanation: In club foot, there is an equinus deformity- decrease in ankle dorsiflexion is frequently seen. (B) There is no valgus component in club foot- the foot is in varus hence the name talipes equinovarus. (C) The word talipes comes from a combination of talus and pes (foot) but is not an appropriate answer choice. (D) There is an adduction of the forefoot to the rearfoot.

A 56 year old male patient presents to your office with a chief complaint of a painful, hot swollen first MPJ. When you obtain an x-ray you note opacification and densities, bone representing urate deposits and osteophytes of tophi. Which of the following is true of your patient? A. He is having an acute gout attack but has gout for over 5 years. B. He is having his first acute gout attack ever. C. He has an infection and needs antibiotics. D. He has cellulitis and should be referred to the emergency room.

Correct answer: (A) He is having an acute gout attack but has gout for over 5 years. Explanation: The clinical picture is consistent with chronic gouty changes with an acute attack. An acute gout attack without a history of gout would not show osseous changes on xray immediately. Due to the gouty changes on xray, it is presumed it is not an infection requiring antibiotics or cellulitis requiring a visit to the emergency room.

This raised soft tissue lesion is often associated with an underlapping fifth toe and an adductus deformity of the fifth metatarsophalangeal joint. The head of the proximal phalanx of the fifth toe is typically abutting the lateral condyle of the base of the fourth proximal phalanx. A. Heloma molle B. Helloma durum C. Ganglion cyst D. Verruca plantaris

Correct answer: (A) Heloma molle. Explanation: A corn is a circumscribed, sharply demarcated area of hyperkeratotic plaque over the bony prominences of the feet and toes. It has a visible translucent central core (i.e., a thickening of the stratum corneum due to shearing or compressive forces), which presses deeply into the dermis, causing pain and sometimes inflammation. The conical core in a corn is a protective response to the mechanical trauma. This central core distinguishes the corn from the callus. Like a callus, a corn forms as a response to friction and pressure. Unlike a callus, a corn has no papillary ridges, and the surface is burnished smooth because the skin responds to pinpoint pressure against a bony prominence by a rapid increase in cell production. This causes the development of layers of immature cells that never form a competent dead skin layer as in a callus. The term heloma is sometimes used to denote a corn. Corns, which are tender on direct pressure, are divided into five subtypes—the hard corn (heloma durum), the soft corn (heloma molle), the vascular corns (heloma vasculare), the fibrous corn (heloma fascia) and the seed corn (heloma milliare).

The most sensitive and specific modality to diagnose a forefoot neuroma is: A. History and physical B. MRI C. Ultrasound D. Diagnostic injection

Correct answer: (A) History and physical. Explanation: The history and physical is the most reliable indicator in the diagnosing of a forefoot neuroma. Patient complaints of "a rolled up sock", "a rock", "shooting pain to two toes", "have to take my shoe off to rub my foot", "cramping", and "numbness" are examples. Physical findings of pain in the affected interspace, a palpable mass such as a bursa, or most importantly, a positive Mulder's sign, which Coughlin's text states is diagnostic. (B) The use of MRI is controversial. It may be indicated in atypical cases. Specificity and sensitivity results are quite variable. Differences in sensitivity can be due to the equipment, lesion size, or foot positioning. The highest sensitivities are seen when both MRI and US re performed together. (C) The use of US is controversial. It may be indicated in atypical cases. It is more operator dependent than MRI, leading to more errors in diagnosis than MRI. If one had to choose between the two modalities, MRI would be preferred, despite the increased expense. (D) A diagnostic injection of 1-3cc lidocaine into the affected interspace may be helpful in leading one toward the diagnosis of neuroma; it is not diagnostic. Its sensitivity and specificity is low. McGlamry states "Diagnostic injections are of little value in the accurate diagnosing of intermetatarsal neuromas."

A 43 year old male attempted to commit suicide by jumping off the roof of his house. He was unsuccessful and was brought to the emergency room and trauma work-up started. After being cleared in terms of neck/back injury, x-rays were taken of his feet and heels. The podiatry resident noted a calcaneal fracture. Which of the following would be expected to be seen on the x-ray of the heel? A. Increase in Critical Angle of Gissane B. Increase in Bohler's angle C. Narrowing of the heel D. Posterior facet never involved

Correct answer: (A) Increase in Critical Angle of Gissane. Explanation: The critical angle of Gissane is an angle formed between a line at the subchondral bone on the posterior facet and that on the anterior and middle facet. The normal value of this angle is 125 degrees to 140 degrees. This increases in an intra-articular calcaneal fracture. Bohler's angle is an angle formed by two lines. The first is that between the highest point on anterior tubercle of heel to highest point on posterior articular facet, the second is between the highest point on posterior articular facet and most superior part of calcaneal tuberosity. The normal for this angle is 25- 40 degrees and this angle decreases in a calcaneal fracture. Other things that are commonly seen include a large space in between the posterior facet of the subtalar joint (sometimes this can be the only radiographic sign) and widening of the heel- especially on calcaneal axial image.

Which of the following ligaments does not provide stability to the midtarsal joint? A. Lateral talocalcaneal ligament (Cervical ligament) B. Plantar calcaneonavicular ligament C. Bifurcate Ligament D. Long plantar ligament

Correct answer: (A) Lateral talocalcaneal ligament (Cervical ligament). Explanation: The lateral talocalcaneal ligament, also known as the cervical ligament, runs from the lateral tubercle of the talus obliquely and inserts on the lateral surface of the calcaneus. This ligament limits inversion of the subtalar joint. The plantar calcaneonavicular, bifurcate, long and short plantar ligaments provide stability to the midtarsal joint, limiting pronation and supination of the midtarsal joint. The plantar calcaneocuboid (short plantar ligament) ligament and the calcaneocuboid component of the bifurcate ligament are tightened with pronation of the rearfoot and supination of the midtarsal joint.

Which one of the following is an absolute contraindication for reaming? A. Long bones with segmental bone loss B. History of osteomyelitis C. Surgeon's inexperience with the technique of nailing with reaming D. Smoking

Correct answer: (A) Long bones with segmental bone loss. Explanation: Nailing with reaming is clearly indicated for a patient who has had a non-union of the tibial (long bone) diaphysis in which there is no evidence of infection and for which other forms of treatment have failed. Absolute contraindications are fractures with segmental bone loss, those that are outside the level of the tibia (long bone) that are amenable to fixation with nails, and those in which there are soft tissue defects at the site of the fracture. It should never be performed in patients with severe peripheral vascular disease. Relative contraindications include active drainage from osteomyelitis and surgeon inexperience with the techniques of nailing with reaming.

immediately pre-operatively, the patient is having severe anxiety in the holding area. Which is the best medication to order for the patient? A. Lorazepam 3mg PO B. Celecoxib 200mg PO C. Morphine 1mg IV D. Gabapentin 300mg PO

Correct answer: (A) Lorazepam 3mg PO. Explanation: Lorazepam is a benzodiazapem used to treat anxiety disorders. It may be used in the peri-operative setting for anxiolysis. (B) Celecoxib is a non-opioid analgesic and is not beneficial for perioperative anxiety. (C) Morphine is an opioid used for analgesia, not routinely used for anxiolysis. (D) Gabapentin is a calcium channel blocker that can be used to stop the mechanisms of pain processing.

A calcaneonavicular coalition is best visualized using which x-ray view? A. Medial oblique view B. Medial oblique projection C. Harris-beath projection D. Lateral projection

Correct answer: (A) Medial oblique view. Explanation: The medial oblique view, also the lateral oblique projection is best able to visualize the relationship between the anterior process of the calcaneus and the navicular. If there is an osseous coalition, there may be no visible joint space between the structures. If there is a cartilaginous or fibrous coalition, there is continuity in shape of the calcaneus with the navicular with visible joint space. (B) The medial oblique projection would not provide an ideal view of the relationship between the anterior process of the calcaneus and the lateral aspect of the navicular. (C) The Harris-beath view is utilized to visualize the middle and posterior facets of the subtalar joint when talocalcaneal coalition is suspected. (D) A lateral projection can provide information that may increase suspicion of a calcaneonavicular coalition. An "anteater sign" of the anterior process of the calcaneus protruding upwards and overlapping with the navicular is highly suggestive of a calcaneonavicular coalition. However, the medial oblique view provides a clearer visualization of the union.

A 44 year old male presents complaining of pain to the ball of his right foot. He denies any trauma and states the pain has been present for over 2 years. He states that he is just too busy to come to the doctor, but today his wife made him make the appointment. Below is a clinical picture from his exam. Based on the photograph below, what is the most likely diagnosis? A. Morton's neuroma B. Hauser's neuroma C. Iselin's neuroma D. Heuter's neuroma

Correct answer: (A) Morton's neuroma. Explanation: The clinical photograph above is an example of Sullivan's sign. Sullivan's sign is a divergence of the toes created by the neuroma physically spreading the toes apart. The photograph above shows affection of the third interspace. A neuroma of the third interspace is formally known as a Morton's neuroma. A Heuter's neuroma is a neuroma of the first interspace. A Hauser's neuroma is that of the second interspace. An Iselin's neuroma is a neuroma of the fourth interspace. Lastly, a Joplin's neuroma is a neuroma of the medial digital nerve of the hallux.

What is the main ligament of the subtalar joint? A. The interosseous talocalcaneal ligament. B. The posterior talocalcaneal ligament. C. The inferior extensor retinaculum. D. The bifurcate ligament.

Correct answer: (A) The interosseous talocalcaneal ligament. Explanation: The ligaments of the subtalar joint are considered short and powerful structures well-adapted to the severe stresses applied to them during weightbearing activities. The main ligament is the interosseous talocalcaneal ligament which consists of two bands, occupying the sinus tarsi. The anterior band runs from the calcaneus superiorly, anteriorly, laterally to insert on the inferior surface of the talar neck, just behind the edge of the articular facet of the head. The posterior band originates just superior to the anterior band obliquely, superiorly, posteriorly and laterally to insert just anterior facet of the talus. The ligament has been said to limit inversion and eversion of the subtalar joint.

Which of the following statements regarding phenol is correct? A. Must be stored in a dark bottle as it is photosensitive. B. It has low toxicity. C. It is insoluble in alcohol and soluble in water. D. Alcohol neutralizes phenol.

Correct answer: (A) Must be stored in a dark bottle as it is photosensitive. Explanation: Phenol is quite photosensitive and therefore must be stored in a dark glass non- reactive bottle in a closed cabinet. Exposure to light must be minimal and light will denature the phenol rendering it ineffective. It is a common misconception that alcohol neutralizes phenol. This in fact is not the case. The reason alcohol is is used in a phenol matrixectomy procedure is that like phenol, alcohol is hydrophobic. This is in stark contrast to saline which is hydrophilic. In other words phenol is soluble in alcohol and insoluble in water. Therefore, alcohol can effectively dilute and wash away the phenol from the surgical site. That is also why it is recommended to use copious amounts of alcohol. If in fact alcohol neutralized phenol, then only enough alcohol to equal the amount of phenol used would be necessary. The most commonly used concentration of phenol is 89%. In contrary, it is highly toxic and must be handled with care.

Where can authorized health entities go to view any payments made for physicians in association with medical malpractice settlements or judgments as well as adverse peer review actions taken against the physician? A. National Practitioner Data Bank B. National Bank of Troubled Physicians C. Any Local Courthouse D. No such records are kept in an attempt to protect the people involved.

Correct answer: (A) National Practitioner Data Bank. Explanation: The National Practitioner Data Bank (NPDB) is an electronic bank of payments made for physicians in association with medical malpractice settlements or judgments as well as adverse peer review actions taken against the physician. By federal law, all medical malpractice payments and certain adverse actions must be reported to the NPDB. The NPDB is required to share this information with authorized health entities such as hospitals, certain professional societies, and state licensure boards.

Which of the following is the primary determinant of the patient radiation dose? A. Photoelectric effect B. Compton scatter C. Modified scatter D. Thompson scatter

Correct answer: (A) Photoelectric effect. Explanation: The photoelectric effect results in the transfer of all of the energy from a photon to the tissue and disappearance of the photon, causing the greatest amount of harm to the patient. Compton scatter, also known as modified scatter, results in partial transfer of energy from a photon to an electron that is loosely bound in an outer atomic shell. The photon is scattered in another direction with reduced energy, causing the greatest hazard to the radiographer and other personnel. Compton scatter also contributes to image fog. Thompson scatter, known as classic scatter, results in disappearance of the x-ray photon during a non-ionizing interaction with an atom. No ionization results, but a photon of equal energy is released to travel in a different direction.

Concerning the depth of anesthesia, stage 3 is further subdivided into 4 planes. Which planes is best described by the following characteristics? - slight somatic relaxation - regular periodic breathing - active ocular muscles - swallowing reflex disappears - eyelid reflex is lost A. Plane 1 B. Plane 2 C. Plane 3 D. Plane 4

Correct answer: (A) Plane 1. Explanation: When discussing the depth of anesthesia there are four separated stages. Stage 1 is amnesia/analgesia, stage 2 is delirium/excitement, stage 3 is surgical anesthesia and stage 4 is overdosage. Stage 3 lasts from the onset of regular breathing to respiratory arrest. Stage 3 is further subdivided into four planes. Plane 1 is best characterized by a slight somatic relaxation and regular periodic breathing. The patient will still have active ocular muscles in plane 1. The eyelid reflex is lost, the swallowing reflex disappears and toward the end of the plane the conjunctival reflex is lost.

Which incisional approach for neuroma resections providers the best exposure? A. Plantar longitudinal B. Plantar transverse C. Dorsal longitudinal D. Web-splitting

Correct answer: (A) Plantar longitudinal. Explanation: The plantar longitudinal incision is unsurpassed as it pertains to surgical exposure for neuroma excision. Lying between the skin and the neuroma from a plantar perspective is only subcutaneous fat. Painful plantar scarring is the reason why this incision isn't more commonly used. (B) The plantar transverse incision for the most part avoids the painful plantar scar. However, exposure, especially to the proximal portion of the nerve is limited. (C) The most commonly used incision, the dorsal longitudinal, requires extensive dissection to gain exposure of the neuroma. Dissection must be performed through subcutaneous fat, the transverse metatarsal ligament, then to only have to manually operated the adjacent metatarsals to gain exposure. (D) The web-splitting incisional approach for neuroma excision makes visualization of the proximal portion of the neuroma difficult. The resulting scar is probably the most desirable of all the incisional approaches.

Which of the following is a potentially limb-threatening complication of external fixation use? A. Potential for pin tract deep osteomyelitis B. Bulkiness C. Inconvenience to the patient D. Manipulation by the patient

Correct answer: (A) Potential for pin tract deep osteomyelitis. Explanation: Advantages of external fixators include the accessibility of the frame for manipulation or adjustment after initial fixation, easy access to the injury site, minimal interference with adjacent joints, and ease of patient mobilization. Disadvantages of external fixators include damage to structures (nerves and vessels) during frame application. Because the pins are continuous through the skin surface, a potential route for infection exists, either locally(pin tract infection) or deep (osteomyelitis). Pins can loosen and cause bony sequestra to form. External fixators can be bulky and inconvenient to the patient.

What is the role of pretibial muscles (anterior tibial, extensor digitorum longus, extensor hallucis longus) at contact period of the gait cycle in normal gait? A. Provide an unstable midtarsal joint for shock absorption and adaptation B. Accelerate plantarflexion at the ankle C. Provide a stable midtarsal joint for shock absorption D. Transform the foot into a rigid lever at heel strike

Correct answer: (A) Provide an unstable midtarsal joint for shock absorption and adaptation. Explanation: Role of pretibial muscles (anterior tibial, extensor digitorum longus, extensor hallucis longus) at contact period of the gait cycle in normal gait: 1. Decelerate plantarflexion of the ankle 2. Decelerate pronation and provide even weightbearing from the lateral to the medial side of the foot 3. Provide an unstable midtarsal joint for shock absorption and adaptation 4. Absorb impact loads from the floor At contact, all the muscles are active and reach their peak activity prior to foot flat. Perry states that the anterior tibial generates the largest torque around the ankle joint, followed by the extensor digitorum longus and the extensor hallucis longus. At heel contact, the subtalar joint is supinated. The anterior tibial assists in the deceleration of subtalar joint pronation.

This is an excellent technique to produce intramedullary grafting: A. Reaming B. Insertion of the nail C. Insertion of locking screws D. Use of comminuted fragments at the fracture site

Correct answer: (A) Reaming. Explanation: Reaming is an excellent technique for producing intramedullary autografting. The debris which is created usually is collected at the fracture gap and that facilitates the local fracture healing response. Besides the different experimental trials that have been done to prove these facts, there is also plenty of evidence about the beneficial effect of the reaming being considered as a bone stimulating procedure and that's why it is used as a means of treating nonunions of long bones. When we ream, the byproduct also has an increased concentration of growth factors. The reaming debris is also a source of multiple releases of stem cells which are able to grow and proliferate in vitro. That means that not all the cellular elements which are present in the marrow are actually killed when we ream. A lot of them remain alive and this is why we get this beneficial effect when we ream, because we stimulate them to release some of these growth factors and they also have the capacity because of this release to proliferate.

A 12 y/o girl has a painful flatfoot, with no STJ motion. A CT scan finds a middle facet TC coalition, involving 25% of the facet. There is no DJD on x-ray. There is talar beaking at the TNJ, on the lateral x-ray. When non-surgical bracing and casting failed, the best surgical course is: A. Resection of the coalition with split FHL interposition B. STJ fusion C. Arthroereisis D. Coalition resection; TNJ fusion

Correct answer: (A) Resection of the coalition with split FHL interposition. Explanation: The preponderance of the literature states a middle facet coalition involving less than 50% of the facet should be excised. Interposition with tendon or a fat graft may prevent reapproximation. Of interest, Myerson resects the entire coalition regardless of size or percent involvement. Davis, in ADVANCED RECONSTRUCTION: FOOT AND ANKLE states "a subtalar arthrodesis is performed in adolescents with more than 50% involvement." (B) The indication for an STJ fusion may be more extensive involvement of the coalition, or hindfoot DJD, which is not present in this scenario. If DJD is present in the hindfoot, consideration may have to be given to a triple arthrodesis, or at least a double arthrodesis, excluding the CCJ. (C) Arthroereisis will not correct this rigid flatfoot caused by the coalition, but may be a nice adjunct post resection if there is heel valgus but no forefoot abduction. In addition to inverting the heel, the posterior facet will be opened. (D) It is incorrect to assume that talar beaking is indicative of TNJ arthritis, requiring fusion. Talar beaking occurs from traction and pulling of the dorsal talonavicular ligament; traction on the anterior aspect of the ankle on the talar neck and in no way is indicative of DJD. However, it has been stated that the presence of talar beaking is pathognomonic of a coalition.

Which inhaled anesthetic has the following characteristics? - MAC = 2.2 - non-pungent - excellent induction agent - expensive - little cardiac depression A. Sevoflurane B. Halothane C. Isoflurane D. Desflurane

Correct answer: (A) Sevoflurane. Explanation: Sevoflurane is a volatile inhalation anesthetic used for both induction and maintenance. It has a minimum alveolar concentration of 2.2. It is very commonly used for induction as it does not irritate mucous membranes and has a sweet smell. Second only to desflurane, it has both a rapid onset and offset. Generally considered to be a very safe agent for all age groups. Sevoflurane shows minimal cardiac depression. Some question has recently arisen whether or not Sevoflurane might potentiate neurotoxicity. Another significant negative of Sevoflurane is the cost.

Which of the following is a relative contraindication to tourniquet use? A. Sickle cell disease B. History of closed fracture C. Hypertension D. COPD E. History of stroke

Correct answer: (A) Sickle cell disease. Explanation: Tourniquet use is sometimes dangerous in a patient who carries the sickle cell gene. This is because tourniquet use can lead to circulatory stasis, acidosis, and hypoxia which all can induce sickling. Tourniquet use may be safe in these patients if optimum acid-base status and oxygenation are maintained throughout the procedure and the patient's PO2 and pH are carefully monitored as well.

What is the most ideal implantable alloy for metallic implants such as screws, plates, and pins? A. Stainless steel B. Iron C. Aluminum D. Cobalt

Correct answer: (A) Stainless steel. Explanation: This form is steel is designated 316LVM in manufacturing. It offers great resistance to corrosion, as well as malleability and ductility when compared to harder steels like those found in the 400 series. Its ability to withstand corrosion stems from its passivation of a thin surface layer of oxide. Austenitic alloys, such as 316LVM, are also used for the manufacturing of certain surgical instruments, while martensitic alloys are typically used for cutting instruments as they are typically harder materials. (B) (C) (D) These are not used for implants such as screws, plates, and pins for the body. Their properties are not ideal for implantation within the human body.

Which of the following is the most common cause of iatrogenic hallux varus? A. Staking the head B. Underzealous medial capsulorraphy C. Undercorrection of the intermetatarsal angle via osteotomy D. Aggressive post-op bandaging and splinting

Correct answer: (A) Staking the head. Explanation: Staking the head or excessive resection of the medial eminence with disruption of the sagittal groove is the most common cause of iatrogenic hallux varus. By doing so, the 1st MPJ is destabilized and alters the pull of the flexor hallucis brevis and abductor hallucis muscles. Other causes of iatrogenic hallux varus include overzealous medial capsulorraphy, overcorrection of the intermetarsal angle via osteotomy, and aggressive post-op bandaging and splinting (although this isn't the most common cause). Another very common cause of iatrogenic hallux varus is overagrressive lateral release with or without lateral sesamoidectomy, which has been cited as the most often cause of iatrogenic hallux varus (although this was not one of the answer choices).

You are evaluating a patient with a chief complaint of first metatarsophalangeal joint pain when walking for long periods of time. While performing your physical exam you find the patient to have 50 degrees dorsiflexion range of motion at the 1st MPJ unloaded, and 12 degrees of dorsiflexion when loaded. What is the best term to describe her biomechanical diagnosis? A. Hallux rigidus B. Functional Hallux Limitus C. Plantar plate tear D. Mallet toe

Correct answer: (B) Functional Hallux Limitus. Explanation: Functional hallux limitus is defined by several authors as 12° or less of restricted hallux dorsiflexion in closed kinetic chain and 50° or greater motion in open kinetic chain examination. Grady et al demonstrated that 47% of patients with symptomatic hallux limitus were treated with custom orthoses alone.

You are planning a cheilectomy procedure to decompress the joint of a 40 year diabetic patient. During the pre-operative office visit, you are going over the necessity for glycemic control and the risks involved with the procedure. You explain a stress response to the patient, which is best defined as which of the following? Surgery and anesthesia may cause diabetic ketoacidosis in type 1 DM and hyperosmolar hyperglycemic nonketosis in type 2 Diabetes Mellitus Surgery may cause diabetic ketoacidosis in type 2 Diabetes Mellitus and hyperosmolar hyperglycemic nonketosis in type 2 Diabetes Mellitus Surgery and anesthesia will increased peripheral affinity to insulin Surgery and anesthesia will cause low levels of insulin and hyperglycemia

Correct answer: (A) Surgery and anesthesia may cause diabetic ketoacidosis in type 1 DM and hyperosmolar hyperglycemic nonketosis in type 2 Diabetes Mellitus. Explanation: Surgery and anesthesia invoke a "stress response" in patients that is characterized by hypersecretion of counterregulatory hormones like glucagon, norepinephrine, cortisol, growth hormone. This response will lead to increased gluconeogenesis, glycogenolysis, and peripheral insulin resistance. Endogenous insulin levels are dramatically increased in the face of hyperglycemia. This will cause altered hormonal milieu which may culminate in diabetic ketoacidosis in Type 1 DM, hyperosmolar hyperglycemic nonketosis in patients with Type 2 DM. Diabetic ketoacidosis is a catabolic condition that results in severe insulin deficiency, often in association with stress and activation of counterregulatory hormones such as glucagon, norepinephrine, cortisol, growth hormone. Nonketotic hyperosmolor syndrome is one of the most life threatening complications of type 2 DM with precipitating factors such as stress dehydration, infection, stroke and non -compliance with medications. Understanding this hormonal imbalance is fundamental to appreciating the fine endocrine balance the patients withstand in the perioperative period.

What is the the first joint fixated when performing a triple arthrodesis? A. Talonavicular B. Calcaneocuboid C. Subtalar D. It doesn't matter which joint is fixated first.

Correct answer: (A) Talonavicular. Explanation: The talonavicular joint is the first joint fixated when performing a triple arthrodesis. The justification for this is that often there is a masked compensatory forefoot deformity that must be addressed before the hindfoot is fixated. The technique utilized is that the hindfoot is held in 5° valgus , the forefoot is reduced into a rectus position while simultaneously provisionally fixating the talonavicular joint. (B) The calcaneocuboid joint is not the first joint fixated in a triple arthrodesis. (C) The subtalar joint is not the first joint fixated in a triple arthrodesis. (D) It does, indeed, matter which joint is fixated first. The talonavicular joint is the first joint fixated when performing a triple arthrodesis.

Which radiographic sign is consistent with congenital convex pes planovalgus (vertical talus)? A. Talus parallel with the longitudinal axis of the tibia B. Foot adducted relative to the leg C. Navicular/metatarsals plantar to the talus D. Talocalcaneal angle less than 40 degrees on the anteroposterior view

Correct answer: (A) Talus parallel with the longitudinal axis of the tibia. Explanation: Congenital convex pes planovalgus is a severe rigid congenital deformity. The radiographic signs include increased talocalcaneal angle, navicular/metatarsals dorsal to the talus, and foot abducted relative to the leg. answers B-D are opposite of what you would expect with pes planovalgus

Kirschner wires can be used to provide compression with this method: A. Tension band wiring B. Cerclage wiring C. Percutaneous pinning D. Adding threads

Correct answer: (A) Tension band wiring. Explanation: Kirschner wires can be combined in a unique fashion with interosseous wire to provide the technique of tension band wiring. The technique requires that a displaced fracture have a stable soft tissue attachment, such as a ligament or tendon. Two Kirschner wires are placed in parallel fashion across the fracture site for provisional stability. Interosseous wire is placed in a figure-of-eight fashion on the fracture side opposite to the tendon's anatomic pull or in this case, the deltoid ligament. The Kirschner wire with figure-of-eight interosseous wire loop exerts a force opposite in direction, yet it is equal in magnitude to distraction forces generated by the deltoid ligament. Tension band wiring is especially useful for reattachment of small fracture fragments that would be irreversibly damaged with passage of a screw (i.e., intraarticular avulsion fractures of the styloid process of the fifth metatarsal).

A 26 year old female patients presents with burns on the anterior surface of both legs and dorsal surfaces of both feet. She states she was cooking dinner and spilled a pot of boiling water causing her injuries. On exam, the anterior surface of both legs and dorsal surfaces of both feet are erythematous with large ruptured blisters with intact skin on the feet and ruptured, deroofed blisters on the legs with pink tissue within the burn base. There is a small intact blister on the hallux of the left foot. The burns are very painful to exam and hypersensitive to air exposure. What would be the best treatment? Tetanus immunization, bedside cleansing, debridement of any ruptured blisters, silver sulfadiazine cream and non adherent dressing Culture blister fluid to target antibiotic therapy, bedside cleansing, silver sulfadiazine cream, non adherent dressing Debridement of ruptured blisters, bedside cleansing, vaseline, non adherent dressing Sterile dry dressing, admit the patient and plan for OR for split thickness skin grafting when the burns are stable

Correct answer: (A) Tetanus immunization, bedside cleansing, debridement of any ruptured blisters, silver sulfadiazine cream and non adherent dressing. Explanation: Tetanus immunization should be verified for anything more serious than a superficial burn. Cleansing of the skin will help reduce skin flora and infection risk. While some controversy exists surrounding the treatment of intact burn blisters, it's generally accepted that already ruptured blisters should be deroofed to reduce the risk of infection. The use of Silvadene cream and other topical antibacterial agents are advised for the treatment of most partial thickness burns. (B) The aspiration of blister fluid may increase the risk of infection. Moreover, antibiotic therapy is not typically part of the treatment of partial thickness burns. (C) This answer does not include tetanus update or the use of an antibacterial topical, both are advised. (D) Split thickness skin grafting is not usually needed for partial thickness burns. Reepithelization usually occurs within 14 days with conservative treatment.

What provides medial and lateral stability of the first metatarsophalangeal joint? The collateral ligaments, the suspensory sesamoid ligaments, the plantar sesamoid ligaments The adductor muscle, the transverse head of the abductor muscle The extensor hallucis capsularis The medial and collateral ligaments

Correct answer: (A) The collateral ligaments, the suspensory sesamoid ligaments, the plantar sesamoid ligaments. Explanation: The medial and lateral stability of the first metatarsophalangeal joint is provided by a triangular arrangement of ligaments reinforcing the capsule. The collateral ligaments, the suspensory sesamoid, and the plantar sesamoid ligaments make up the three arms of the triangle on either side of the joint. In addition, a ligament is transversely attached between the two sesamoids and the deep transverse metatarsal ligament (medial segment), which with the other intermetatarsal segments forms a "strap", that resists separation of the joints under loading stress.

A 68 year old male with a history of severe osteoarthritis of the right ankle has undergone a total ankle replacement surgery. For optimal pain control, the anesthesiologist has left the epidural catheter in place. What precautions need to be taken while using pharmacologic VTE prophylaxis, when removing the indwelling catheter? Wait at least 2 hours after removal of the catheter to restart the lower molecular weight heparin. Wait at least 2 hours after the last dose of lower molecular weight heparin to remove the catheter. Remove the catheter before the last dose of lower molecular weight heparin. Wait at least 2 hours after the first dose of warfarin to remove the catheter.

Correct answer: (A) Wait at least 2 hours after removal of the catheter to restart the lower molecular weight heparin. Explanation: The timing of prophylactic anticoagulation is further complicated when neuraxial anesthesia is employed. The delivery of the anesthetic to the neuraxis requires insertion of a needle or a catheter into the epidural space. In the scenario, the concomitant use of prophylactic anticoagulation may increase the risk of paraspinal hematomas. The American College of chest physicians and the American Society of Regional Anethesiology agree that pharmacologic VTE prophylaxis can still be used in parallel with neuraxial anesthesia, as long as precautions are taken to reduce the risk of bleeding. Specific recommendations include waiting 10 -to 12 hours after a prophylactic dose of lower molecular weight heparin before inserting the epidural catheter, waiting at least 10 to 12 hours after the last dose before removing the catheter, and waiting at least 2 hours after the last dose before restarting the LMWH. These precautions include Lower dose unfractionated heparin, Fondapirinux, warfarin.

A 23 year old male presents for surgical consultation for treatment of tibial and fibular sesamoid fractures of the left first metatarsophalangeal joint. This injury was the result of a hard landing while sky diving. The surgical plan is surgical excision of both sesamoids. What adjunctive procedure(s) might the surgeon consider to eliminate the chance for later biomechanical sequela? (Select all that apply) A. Keller procedure B. Hallux interphalangeal joint arthrodesis C. First metatarsophalangeal joint arthrodesis D. Jones tenosuspension

Correct answer: (B) (D) Hallux interphalangeal joint arthrodesis and jones tenosuspension. Explanation: Removal of both sesamoids creates the scenario where the flexor apparatus may be overpowered and a hallux malleus deformity frequently develops. The hallux interphalangeal joint arthrodesis prevents the formation of hallux malleus and therefore would be a sound adjunctive procedure. Removal of both sesamoids creates the scenario where the flexor apparatus may be overpowered and a hallux malleus deformity frequently develops. The Jones procedure would balance the first metatarsophalangeal joint and prevents excessive dorsiflexion of the joint. (A) The Keller procedure is a viable adjunctive procedure, however is best indicated for individuals with arthritic developments in the first metatarsophalangeal joint. (C) First metatarsophalangeal joint arthrodesis, although would stabilize the ray at this joint love, the interphalangeal joint could still develop a flexion deformity.

Neutral position of the subtalar joint as described by Root is when the joint in which the foot is neither supinated nor pronated. Full supination of the normal subtalar joint inverts the calcaneus twice as many degrees as full pronation everts it. What is the average subtalar joint neutral position? A. 0 to 5 degree inversion of the calcaneus to the lower 1/3 of the leg B. 0 to 3 degree inversion of the calcaneus to the lower 1/3 of the leg C. 0 to 5 degree eversion of the calcaneus to the lower 1/3 of the leg D. 0 to 3 degree eversion of the calcaneus to the lower 1/3 of the leg

Correct answer: (B) 0 to 3 degree inversion of the calcaneus to the lower 1/3 of the leg. Explanation: Root et al state that "the neutral position of the subtalar joint is that position of the joint in which the foot is neither pronated nor supinated." From this position, full supination of the normal subtalar joint inverts the calcaneus twice as many degrees as full pronation everts it. The average subtalar joint neutral position is a 0- to 3-degree inverted attitude of the calcaneus as measured relative to the lower one third of the leg. Although it is common to find a 2:1 ratio of supination to pronation at the subtalar joint, it is not unusual for individuals to exhibit 3:1 or 4:1 ratios.

AO/ASIF screws are available in three sets: mini-fragment, small fragment, and large fragment sets. Which of the following options lists the screw diameters found in the mini-fragment set? A. 2.7 and 3.5 mm B. 1.5, 2.0 and 2.7 mm C. 1.5 and 3.5 mm D. 3.5 and 4.0 mm E. 4.5 and 6.5 mm

Correct answer: (B) 1.5, 2.0, and 2.7. Explanation: AO/ASIF screws are available in three basic sets (other sets are available): minifragment, small fragment, and basic or large fragment. The minifragment set contains screws with an outer diameter of 1.5, 2.0, and 2.7 mm. The small fragment set contains screws with an outer diameter of 3.5 and 4.0 mm. The large fragment set contains both 4.5- and 6.5-mm screws. All AO/ASIF screw sets come complete with a full complement of equipment to insert each type of screw. Screws are available in stainless steel or titanium.

An 80kg patient who is NPO from midnight to their 7am or 8am surgery time has an approximate 1L fluid deficit. In the peri-operative period, if the patient is admitted, how much IV fluid replacement does this patient require daily? A. 2500ml IV fluid daily. B. 2700ml IV fluid daily. C. 2900ml IV fluid daily. D. 3100 IV fluid daily.

Correct answer: (B) 2700ml IV fluid daily. Explanation: The Holliday-Segar Method can be utilized to calculate fluid replacement based upon caloric needs. Water loss is a function of caloric expenditure. Total daily water requirement to replace insensible and urinary water loss in the hospitalized patient is approximately 100ml/100kcal/day. The caloric expenditure is approximated based upon body weight. For each 1-10kg BW, there is a daily caloric cost/kg of 100kcal/kg/day. For each kg BW between 11- 20kg, there is a daily caloric cost/kg of 50kcal/kg/day. For greater than 20kg BW, there is a daily caloric cost/kg of 20kcal/kg/day. For the above patient, the calculation is as follows: First 10kg BW x 100ml=1000ml Second 10kg BW x 50ml=500ml Remaining 60kg BW x 20ml=1,200ml Therefore, this patient requires 2700ml IV fluid/day.

How long should clopidogrel be discontinued preoperatively? A. 24 hours B. 7-10 days C. 2 weeks D. 48 hours E. None of the above

Correct answer: (B) 7-10 days. Explanation: Clopidogrel, as well as the other commonly used thenopyridine, prasugrel, should be discontinued 7-10 days preoperatively.

Patient who is currently on the operating table undergoing a bunion surgery on his right foot starts to bleed profusely upon the first incision. The surgical team applies pressure to the area but is having a very hard time getting the blood to clot. The nurse re-evaluates the chart and realizes there was a mis-communication among the team. The patient's last warfarin dose was this morning and his PT was significantly elevated. Which of the following is the most effective measure to help stop the bleeding? A. Packed Red Blood Cells Transfusion B. Administer Fresh Frozen Plasma C. Vitamin K Infusion D. Platelet Transfusion

Correct answer: (B) Administer Fresh Frozen Plasma. Explanation: The correct answer is fresh frozen plasma. This is a blood product that has the clotting factors necessary to reverse the effects of warfarin. Warfarin is a drug that blocks the synthesis of Vitamin K dependent factors including Factor II, VII, IX, X as well as protein C and S. By giving FFP these factors can work immediately in the patient to stop the bleeding. Vitamin K Infusion (C) does reverse the effect of warfarin but it takes some time to kick in. In a patient who is actively bleeding like the one above, fresh frozen plasma is always a better choice. Packed Red Blood Cell Transfusion (A) may be used if the patient became symptomatic from their hemoglobin dropping too low, but it will not act to reverse the effect of warfarin. Platelet infusion is not indicated at this time because warfarin does not have an effect on the platelet count.

What is the term given for a theoretical line described as a point line about which motion occurs? A. Plane of motion B. Axis C. Atlas D. Center of Gravity

Correct answer: (B) Axis. Explanation: Motion is described as occurring about an axis. It is the motion of a joint, which defines the location of the joint axis. An axis is a point line lying 90 degrees, or perpendicular, to the direction of motion of a joint. The direction in which motion occurs at a given joint is called the plane of motion. In the body, these axes are not fixed in place as the motion at a given joint determines the axis position. The position of a specific axis of a joint can only be determined from understanding the amount of motion at a given joint.

Which of the following techniques for internal osseous fixation will leave comminuted fragments out of the mechanical construct while preserving their blood supply? A. Extramedulllary fixation technique B. Biologic fixation C. Tranfixation D. Percutaneous pinning technique

Correct answer: (B) Biologic fixation. Explanation: The biologic method of fixation can be used to maintain length and alignment. A plate fixation may sometimes be required to bridge a severely comminuted segment of bone. Biologic fixation is recommended when a large screw would result in devitalization of comminuted fragments, or when there is a gap in the fracture and a plate is used to maintain the relative position of the fragments. This fixation is not stable and requires adjunct methods to maintain all the soft tissue attachments and blood supply of the intervening comminuted fragments, since union will depend on the formation of a bridging callus rather than primary bone union. In these situations, techniques of indirect reduction are particularly useful.

Which description below best describes the mechanism of action by which local anesthetics prevent depolarization of the nerve membrane? A. Blockade of the calcium channel B. Blockade of the sodium channel C. Blockade of both the sodium and calcium channels D. Neither blockade of the sodium channel nor the calcium channel

Correct answer: (B) Blockade of the sodium channel. Explanation: All local anesthetics are membrane stabilizing drugs; they reversibly decrease the rate of depolarization and repolarization of excitable membranes. Local anesthetic drugs act mainly by inhibiting sodium influx through sodium-specific ion channels in the neuronal cell membrane, in particular the voltage-gated sodium channels. When the influx of sodium is interrupted, an action potential cannot arise and signal conduction is inhibited.

Which of the following is true regarding radiological findings of Freiberg disease? A. Most commonly affects 3rd metatarsal head B. Commonly see a flattening of the metatarsal head on x-ray C. Rarely see fragmentation of the metatarsal head D. Rarely see secondary osteoarthritis once Friedberg's infarction resolves

Correct answer: (B) Commonly see a flattening of the metatarsal head on x-ray. Explanation: The answer choice that best fits this question is B. Very commonly with this disorder you see a flattening of the metatarsal head on x-ray. It is also very common to see a widening of the joint space early on as well. Another key radiographical feature is the "Crescent Sign". This is a crescent shaped radiodensity. It is most commonly seen in the 2nd met head and shows fragmentation as well. One big complication of Friedberg's disease is the osteoarthritis that occurs in the joint once the problematic area resolves. Treatment during healing consists of immobilization of the part. Surgical intervention is sometimes warranted as well.

Oligohydramnios during pregnancy has been linked to this congenital pediatric foot and ankle deformity. A. Metatarsus adductus B. Congenital Talipes equinovarus C. Tibial torsion D. Flexible pes planus

Correct answer: (B) Congenital Talipes equinovarus. Explanation: Among the causes of congenital talipes equinus varus are intrinsic and extrinsic factors. Among intrinsic factors, we have neurologic conditions such as spina bifida, cerebral palsy, arthrogryposis, connective tissue disease dystrophic dysplasia. Mechanical or extrinsic factors are the following: oligohydramnios, congenital constriction bands, increased intrauterine pressure, uterine fibroids, and multiple gestation pregnancy. Other causes maybe Arrest of early development (Bohm), primary germ plasm defect in the talus, defective signals that provide positional information for the limbs, multifactorial inheritance system.

What is the role of pretibial muscles (anterior tibial, extensor digitorum longus, extensor hallucis longus) at contact period of the gait cycle in normal gait? A. Accelerate plantarflexion of the ankle B. Decelerate pronation and provide even weightbearing from the lateral to the medial side of the foot C. Provide a stable midtarsal joint for shock absorption and adaptation D. Accelerate pronation and provide even weightbearing from the medial to the lateral side of the foot

Correct answer: (B) Decelerate pronation and provide even weightbearing from the lateral to the medial side of the foot. Explanation: Role of pretibial muscles (anterior tibial, extensor digitorum longus, extensor hallucis longus) at contact period of the gait cycle in normal gait: 1. Decelerate plantarflexion of the ankle 2. Decelerate pronation and provide even weightbearing from the lateral to the medial side of the foot 3. Provide an unstable midtarsal joint for shock absorption and adaptation 4. Absorb impact loads from the floor At contact, all the muscles are active and reach their peak activity prior to foot flat. Perry52 states that the anterior tibial generates the largest torque around the ankle joint, followed by the extensor digitorum longus and the extensor hallucis longus. At heel contact, the subtalar joint is supinated. The anterior tibial assists in the deceleration of subtalar joint pronation.

You are caring for an 8-year-old female. Her parents are divorced, and the noncustodial parent lives in another state. At an office visit, the child's custodial parent (who is of the opposite sex) confides in you about the loneliness inherent as a single parent and states that your relationship has "been a lifesaver" because you give support that is no longer available from the other parent/previous spouse, and invites you to dinner. What is the appropriate response? Tell the parent that you are flattered by the invitation, but you must ask the medical board for advice. Decline the invitation, as there is potential for this relationship to affect your professional judgment and/or the parent's behavior relating to the child's health. Since the child's parent is not your patient, accept this invitation if you have never treated the parent in the past. Avoid a relationship with the patient's parent because dependent behaviors could result.

Correct answer: (B) Decline the invitation, as there is potential for this relationship to affect your professional judgment and/or the parent's behavior relating to the child's health. Explanation: A pediatrician or other physician should maintain professional boundaries with the families of his or her patients. Because patients and family members depend on the knowledge and authority of the physician, there is an inherent risk of exploitation. Patients and their family members should be able to trust the physician completely and should feel both physically and emotionally safe in professional relationships with physicians. The clinical judgment of physicians who are involved in an intimate relationship with a patient or family may be clouded, resulting in a breach of professional responsibility.

A patient presents complaining of numbness along the adjacent sides of the hallux and second digits of the left foot. A history of previous ganglion resection from the dorsum of the same foot six months prior was discovered. The most likely diagnosis is iatrogenic severance of which nerve? A. saphenous B. deep peroneal C. Intermediate dorsal cutaneous D. medial dorsal cutaneous

Correct answer: (B) Deep peroneal. Explanation: The key to this question is recognition of surgical anatomy. The innervation to the lateral aspect of the hallux and medial aspect of the second digit is supplied by sensory branches of the deep peroneal nerve. The saphenous nerve provides sensation to the dorsal medial aspect of the foot. The medial dorsal cuntaneous supplies sensory innervation to the hallux, second digit and medial aspect of the third digit with the exception of the adjacent surfaces of the hallux and second toe. The intermediate dorsal cutaneous nerve supplies innervation to the lateral aspect of the third digit and the fourth and fifth digits.

What is the most common complication of surgery performed on patients with rheumatoid arthritis? A. Infection B. Delayed wound healing C. Delayed union D. Vasculitis E. Tendon pathology

Correct answer: (B) Delayed wound healing. Explanation: Complication rates following foot and ankle surgery in rheumatoid patients are as high as 32%, with wound healing complications being the most frequent. (A) Postoperative infection rates are also high, occurring in 12-14% of patients. (C) Delayed union is an infrequent complication. (D) Vasculitis is a potential but uncommon complication. (E) Tendon pathology is an uncommon complication.

The major reason for countersinking a screw is? A. reduce screw prominence B. disperse compression force of the screw head evenly C. allow for shorter screw utilization D. provide proper angulation of the screw

Correct answer: (B) Disperse compression force of the screw head evenly. Explanation: Countersinking is a common practice in AO fixation. The primary reason for doing so is to provide even dispersion of the compressive forces of the screw head on the proximal aspect of the bone. This not only provides even compression across the osteotomy, arthrodesis or fracture site, but greatly reduces the occurrence of a stress riser. Although countersinking does reduce screw head prominence that is not a more important aspect then the dispersion of force mentioned earlier. The proper angulation is best provided by precise drilling prior to screw placement. The aspect of countersinking allowing the utilization of a shorter screw is insignificant.

Which of the following statements regarding external fixation is false? It can provide compression, distraction, or neutralization. External fixation is well tolerated by patients and compliance is seldom an issue. The fixator can be adjusted during the postoperative period in response to changes noted clinically or radiographically. Intraoperative nerve and vessel injury is a potential complication.

Correct answer: (B) External fixation is well tolerated by patients and compliance is seldom an issue. Explanation: Being the most versatile of all the fixation options, external fixation can provide compression, distraction and neutralization. In fact, through circular fixation, combinations of these functions can be provided within the same construct. As an example, compression can be achieved through a subtalar joint arthrodesis while distraction can be applied at the ankle joint for an arthrodiastasis. Due to sometimes lack of social acceptance, size and bulkiness and the need for daily pin and wire care, compliance very often can be an issue that the surgeon must address. A quality that external fixation has to offer is that it can be adjusted during the postoperative period to increase or decrease compression or distraction, provide angular adjustment or move bone segments. Furthermore due to the percutaneous nature of the wire and pin, insertion may in fact inadvertently lead to injury of the neurovascular structures.

You are performing an open reduction internal fixation of the right ankle on a 45 year old male with a past history of ankylosing spondylitis. In addition to obtaining pulmonary tests for residual functional capacity and vital capacity, what other test should be ordered pre-operatively? A. The patch test B. Flexion radiographs C. INR D. Cardiac stress test

Correct answer: (B) Flexion radiographs. Explanation: Patients with spine disease should be adequately assessed for problem with the neck and jaw, which interfere with intubation. Flexion of the neck may show atlantoaxial subluxation, especially in patients with RA and ankylosing spondylitis. (A) This should be ordered if there is suspicion of nickel allergy, regardless of spine disease or not. (C) Recommended in patients on anticoagulation or with a coagulation factor deficiency. (D) This may be ordered in patients with a history of cardiac disease, often in addition to an EKG.

The response of the body to an implanted biomaterial is characterized by inflammation, immunologic response, toxicity, and sometimes carcinogenic evolution. What is this response known as? A. Tissue remodeling response B. Encapsulation C. Cellular response D. Infection response

Correct answer: (C) Cellular response. Explanation: Cellular response is the response of the body to implanted biomaterials, characterized by the inflammation, immunologic response, toxicity, and sometimes carcinogenic evolution. Tissue remodeling is a reactive phase of a host biologic response to implanted biomaterials. It is characterized by implant encapsulation, bone resorption, and sclerosis. Encapsulation is part of the tissue remodeling response.

Which inhaled anesthetic has the following characteristics? - MAC = 0.75 - myocardial depressant - high lipid solubility - liver toxicity A. sevoflurane B. halothane C. isoflurane D. desflurane

Correct answer: (B) Halothane. Explanation: Halothane is the only inhalation anesthetic which contains bromine. Its minimum alveolar concentration is 0.75. It is pleasant smelling so it is well tolerated as an induction agent. It is photosensitive and must be stored in a dark bottle. It causes cardiovascular suppression, lowering both heart rate and blood pressure. In the United States its use has been superseded by Sevoflurane, Isoflurane and Desflurane. Has associated "Halothane Hepatitis" which in realty is very uncommon. Occurrence of halothane-induced hepatitis is between 1:10,000 and 1:35000.

Which of the following is not a potential side effect of spinal anesthesia? A. Headache B. Hypertension C. Urinary retention D. Palsies

Correct answer: (B) Hypertension. Explanation: Post-dural puncture headaches have been a common side effect of spinal anesthesia, which have decreased in frequency due to the introduction of pencil point spinal needles. Other side effects of spinal anesthesia include hypotension, urinary retention, infection and possible palsies. Hypotension and bradycardia are common effects when the sympathetic nerve fibers are anesthetized. Hypertension would occur with a normal sympathetic nervous system response. Urinary retention is common after any type of anesthesia due to anesthesia to the nerves innervating the bladder and opioids can contribute to retention. Nerve palsies may linger after spinal anesthesia if there has been nerve damage.

What is the condition in the synovium of individuals with rheumatoid arthritis when imaged in magnetic resonance? A. Atrophied B. Hypertrophied C. Frayed D. Attenuated

Correct answer: (B) Hypertrophied. Explanation: Synovial hypertrophy is one of the most widely investigated MR imaging manifestations of rheumatoid arthritis. The capsules of diseased joints are characterized by a combination of effusion and synovial hypertrophy. Synovial hypertrophy is intermediate in signal on T1-weighted images and is typically higher in signal than adjacent effusion depending on imaging parameters. Synovial hypertrophy is best appreciated on T2-weighted images, particularly fat suppressed fast spin echo T2-weighted sequences. Abnormal synovium is thickened and may demonstrate frond-like projections that are lower in signal than adjacent effusion.

The number of new cases of a disease over a specific time period is known as what? A. Prevalence B. Incidence C. Frequency D. Inoculation

Correct answer: (B) Incidence. Explanation: Incidence is defined as the number of new cases of a disease over a specific time period. There are two measures of incidence with the only difference between the two measures being the denominator used in the formula. The incidence proportion is the number of new cases divided by the population without the disease at baseline. The incidence rate is the number of new cases of the disease divided by the person - time at risk. The incidence rate is often reported as the number of new cases per some rounded measurement of time at risk, such as 1,000 or 100,000 person-years. This method gives a more precedence description of the incidence than the incidence proportion.

During the onset of malignant hyperthermia, which clinical sign is the last to appear? A. increased end tidal carbon dioxide B. increased body temperature C. increased heart rate D. arrhythmias

Correct answer: (B) Increased body temperature. Explanation: All of the above options are clinical signs of malignant hyperthermia. However, the elevation in body temperatures is that last of the clinical signs above to appear. In other words, by the time that the clinical team detects an elevated body temperature, the malignant hyperthermia is well underway. The signs of malignant hyperthermia are due to the hypermetabolic state of the condition. These signs include increased heart rate, rapid breathing, increased carbon dioxide production, increased oxygen consumption, muscle rigidity, extreme body temperature increase, mixed acidosis and rhabdomyolysis.

Which of the following is correct in terms of four point pattern of gait training? A. Used in patients who need to be non weightbearing B. Indicated for full weightbearing status C. Can be used with a wheelchair D. Most commonly used in patients with no abnormality in balance

Correct answer: (B) Indicated for full weightbearing status. Explanation: A four point pattern of gait training does not allow for complete non weightbearing to either lower extremity. It is usually indicated if there is a problem with balance or coordination. (It is sometimes seen with patients with MS) The order of crutch and foot is "right crutch, left foot, left crutch, right foot" or if starting with the left crutch it is "left crutch, right foot, right crutch, left foot". The three point gait pattern is usually the gait pattern used for non weightbearing of one of the lower extremities.

Dislocations of the first interphalangeal joint in children are almost always treated by closed reduction. From the options below, what is a possible indication for open reduction? A. Medial dislocation B. Interposition of the interphalangeal sesamoid C. Plantar dislocation D. Lateral disclocation

Correct answer: (B) Interposition of the interphalangeal sesamoid. Explanation: Soft tissue interposition of the interphalangeal sesamoid can force the surgeon to convert the reduction from closed to open. (A) Medial dislocation of the distal phalanx of the proximal phalanx is not an indication for open reduction. (C) Plantar dislocation of the distal phalanx relative to the proximal phalanx is not an indication for open reduction. (D) Lateral dislocation of the distal phalanx relative to the proximal phalanx is not an indication for open reduction.

Which of the following statements regarding the plantar transverse incisional approach for neuroma excision is true? A. It is difficult to dissect the nerve trunk proximal to the metatarsal heads. B. It is superior for adjacent interspace neuroma excisions. C. Often it results in a painful plantar scar. D. A period of nonweightbearing is required.

Correct answer: (B) It is superior for adjacent interspace neuroma excisions. Explanation: The plantar transverse incision for neuroma resection has numerous advantages. It is superior in removing adjacent neuromas. The incision is made on a nonweightbearing portion of the foot so immediate weight bearing is afforded. It is not difficult to dissect the nerve trunk proximal to the metatarsal heads with this exposure. It requires much less dissection than the dorsal approach. The incision is made on a nonweightbearing portion of the foot between the metatarsal heads and interdigital folds of the toes. The resultant scar is almost always minimal and obviously there are no cosmetic issues as there is on the dorsum of the foot.

A frail 69 year old female patient presents to the office with a chief complaint of hallux pain. She has had multiple falls recently and is unsteady on her feet. Patient states that she has tried injections and change in shoe gear with limited improvement. States that she is interested in surgical intervention. Patient does not do much athletic activity but does not want a procedure that she has to be non weight-bearing for long periods of time. On x-ray there is narrowing of the 1st MTP joint with osteophytes present. Proximal phalanx length is within normal limits. On exam, 5 degrees of dorsiflexion noted at the MTPJ. Which of the following is the best choice of procedure? A. 1st MTPJ Fusion B. Keller Bunionectomy C. Austin Bunionectomy D. Yougswick Bunionectomy

Correct answer: (B) Keller Bunionectomy. Explanation: The patient in this case has hallux rigidus of her 1st MTPJ. The considerations that you should evaluate for a case like this is age of patient, medical co-morbidities and patient's understanding of weight bearing status. A 1st MTPJ fusion would keep the patient non weight bearing for 6-8 weeks with the use of crutches. In this case, the patient is frail and unstable on her feet and does not want to be non weight bearing so this is not the best choice for her. A Keller bunionectomy is a procedure where the base of the proximal phalanx is removed to increase joint space. Complications of this procedure include an extension toe deformity and is usually reserved for older patients who are not very active and thus would be the procedure of choice in this question. The Austin bunionectomy is not a procedure completed for hallux rigidus and the Youngswick bunionectomy can be used for hallux limitus to decompress the joint but not in severe hallux rigidus.

Which of the following is true of malignant hyperthermia? A. A blood test can be used to determine if you will get this during anesthesia. B. Local anesthesia is the safest for these patients. C. Inhaled anesthetics such as sevoflurane/halothane should be used in these patients with a history. D. Dantrolene is avoided in patients with malignant hyperthermia.

Correct answer: (B) Local anesthesia is the safest for these patients. Explanation: Malignant hyperthermia can not be tested for before surgery with the use of a simple blood test. Most of the time to test for it, you take a skeletal muscle biopsy and expose it to halothane in vitro and look for reaction. In patients who have had it previously, you should avoid all inhaled general anesthetics because they increase the chance of it occurring. Local anesthesia is the safest if you have to do surgical intervention on these patients. If a patient has this disorder while in OR, the drug of choice is dantrolene. (This drug is not to be avoided in malignant hyperthermia).

For a patient who requires discontinuation of warfarin before surgery, this is most common option used for perioperative bridging. A. Unfractionated heparin B. Lower molecular weight heparin C. Fondapirinux D. Intermittent pneumatic compression devices

Correct answer: (B) Lower molecular weight heparin. Explanation: For patients who require discontinuation of warfarin before surgery, the two main options for perioperative bridging include unfractionated heparin (UFH) or LMWH. The use of therapeutic-dose intravenous UFH for perioperative bridging has significantly declined significantly over the years due to the benefit of LMWH. Compared with UFH, LMWH has improved bioavailability, a more predictable dose response, a longer plasma life, and less interaction with platelets, endothelial cells, macrophages, and plasma proteins. Another advantage of the LMWH is that it can be administered outpatient. Factor Xa should always be monitored, as well as creatinine clearance.

Which of the following features is NOT characteristic of the transverse plane type of flexible flatfoot? A. Uncovering of the talar head medially at the talonavicular joint B. Normal lateral radiographs C. Absence of calcaneal eversion D. Abduction of the lateral forefoot E. No failure of medial column on the sagittal plane

Correct answer: (B) Normal lateral radiographs. Explanation: Normal radiographs are NOT characteristic of the transverse plane type of flexible flatfoot. Of the four types of flexible flatfoot based on planar dominance, the transverse type is characterized by uncovering of the talar head medially at the talonavicular joint in the absence of calcaneal eversion. Unlike the sagittal plane type, there is no medial column failure. Abduction of the lateral forefoot is also characteristic of this type of deformity.

Which of the following terms fits this definition: The ability of substances in the graft to induce nonosseous tissue to become osteogenic in the presence of a favorable environment. A. Osteogenesis B. Osteoinduction C. Osteoconduction D. Creeping Substitution

Correct answer: (B) Osteoinduction. Explanation: This question focuses on the biology of bone grafts and their incorporation into host tissues. The description above most accurately describes osteoinduction. In contrast, osteogenesis refers to the synthesis of new bone by cells residing within the graft itself or cells of the host. Osteoconduction is best described as the graft providing a lattice or scaffold that allows surviving host tissue to invade and facilitate new bone formation. Additionally, creeping substitution is the process by which the cutting cone of the new osteoclasts, followed by the osteoblasts, will invade the graft from the host bone.

A 57 year old with IDDM presents to your office with a chief complaint of a swollen toe with a draining ulceration at its tip. The patient states that it has been going on for a few weeks, that he has been feeling generally fatigued and sick the last few days. The patient states that his temperature was 100.9 F last night. His blood sugars have been in the mid 200's which are elevated for him. X-rays done in the office showed erosions in the proximal phalanx of the third toe with fracture through the distal phalanx. Based on the above scenario and x-ray interpretation, which of the following diagnosis is most likely? A. Rheumatoid Arthritis B. Osteomyelitis C. Psoriatic Arthritis D. Gout

Correct answer: (B) Osteomyelitis. Explanation: The clinical scenario points to an infectious etiology based on the temperature of the patient and the fact that the patient is feeling sick. Blood sugars also tend to increase in the presence of infection which is what is described here. Radiologically, osteomyelitis shows erosions of the bone as well as pathological fractures. Rheumatoid arthritis tends to show joint space narrowing that is uniform and symmetrical as well as lateral deviation of the digits. Psoriatic arthritis can show erosions but primarily at the distal interphalangeal joints. With gout, it is common to see the "rat bite erosion sign" but generally the patient would not be sick and febrile; therefore osteomyelitis (bone infection) is the best answer choice here.

Which factor listed below makes a unilateral external fixator less stable? A. Larger diameter pins B. Presence of a locking hinge in the fixator C. Shorter distance between the fixator rail and the bone D. Bicortical purchase of the pins

Correct answer: (B) Presence of a locking hinge in the fixator. Explanation: The stability of unilateral external fixators is multifactorial. The larger the pin diameter the more stable the construct. There obviously is an upper limit that the pins can go based on the size of the bone. In the forefoot most pins are no larger than 3.5 mm in diameter. In the rearfoot they can reach 5-6 mm in diameter. Likewise, the shorter the distance the fixator bar is from the bone the more stable the construct. The pins themselves are more stable when they achieve bicortical purchase which in turn stabilizes the entire external fixator construct. The addition of hinges of any type, be it uniplanar or universal, will destabilize the fixator construct.

During contact phase, what is the position and motion of the oblique midtarsal joint axis? A. Supinated and supination B. Pronated and pronation C. Pronated and supination D. Supinated and pronation

Correct answer: (B) Pronated and pronation. Explanation: Pronated and pronation, the oblique midtarsal joint pronation occurs as a result of contraction of the pretibial muscles, which insert lateral to the axis. In addition, the dorsiflexory force of the ground against the metatarsal heads pronates the joint to the end of its range. As this increases, a mild degree of arch collapse occurs. This action assists in absorbing impact load from the floor.

Which of the following actions will contribute to unlocking of the knee during the gait cycle? A. Supination at the subtalar joint B. Pronation at the subtalar joint C. Lateral rotation of the femur D. Extension of the knee joint

Correct answer: (B) Pronation at the subtalar joint. Explanation: The quadriceps restrains rapid flexion as the hamstrings resist hyperextension. Together they dampen impact load while maintaining stability of the joint. In addition, pronation at the subtalar joint permits faster internal rotation of the tibia to continue, resulting in medial rotation of the femur and further unlocking of the knee.

A patient that sustained polytrauma involving one or more long bones is at risk of developing a "second hit" phenomenon. Which of the following is the most likely cause? A. Thermal necrosis B. Release of inflammatory response during the remaining procedure C. Increase intramedullary pressure D. Fat embolization

Correct answer: (B) Release of inflammatory response during the remaining procedure. Explanation: Numerous studies have demonstrated that stimulation of a variety of inflammatory mediators takes place in the immediate aftermath of trauma. This response initially corresponds to the first-hit phenomenon. Hoch et al. reported elevation in plasma concentrations of IL-6 and IL-8 in patients with an injury severity score of ≥25 points. An immediate increase in expression of neutrophil L-selectin was reported in patients with an injury severity score of ≥16 points. Similarly, a significant (p < 0.05) increase in the expression of the integrin CD11b was noted in more severely injured patients. The development of multiple organ dysfunction syndrome has also been associated with a persistent elevation of CD11b expression on both neutrophils and lymphocytes for 120 hours, a finding that is suggestive of neutrophil activation in the early development of leukocyte mediated end-organ injury. Several other studies have clearly demonstrated the effect of injury severity on the degree. The biological profile of the first hit in trauma patients is being defined. Obertacke et al. demonstrated the importance of the first hit by using bronchopulmonary lavage to assess changes in pulmonary microvascular permeability in patients who had sustained multiple trauma. The permeability of the pulmonary capillaries increased following multiple trauma, and patients in whom adult respiratory distress syndrome later developed had a high correlation (r = 0.81) with increased permeability within just six hours after admission than did those who had had an uneventful recovery. The development of a massive immune reaction in a patient with bilateral femoral fracture who showed a massive inflammatory reaction, which was subsequently hyperstimulated by the surgical procedure itself (bilateral reamed femoral nailing), further supports the importance of the first-hit phenomenon. Although there was no obvious additional risk factor present (i.e., no chest injury), the patient died from full-blown adult respiratory distress syndrome three days after the injury. This case not only clearly illustrates the existence of biological variation in the inflammatory response to injury, but also confirms the importance of the degree of the response to the first hit and the response to the second (surgical) hit that created the final fatal event. The above studies suggest that the degree of the initial injury is important in determining a patient's susceptibility to posttraumatic complications.

Which of the following situations warrants a second dose of peri-operative antibiotics? A. Surgery lasting over one hour B. Significant intra-operative blood loss C. Lapidus with plate and screws D. Diabetic patient with an A1c of 6.5mg/dl

Correct answer: (B) Significant intra-operative blood loss. Explanation: A second dose of antibiotics is warranted with significant intra-operative blood loss, surgery lasting over 2 hours, and a severely immunocompromised patient. (A) A second dose of antibiotics is indicate when surgery is prolonged, generally over 2 hours. (C) Internal fixation is not an indication for additional intra-operative antibiotics. (D) A severely immunocompromised patient may require a second dose of antibiotics intra- operatively. A diabetic patient with moderately controlled A1c is not severely immunocompromised.

A Mayo block affects branches of all of the following nerves except? A. Tibial nerve B. Sural nerve C. Deep peroneal nerve D. Saphenous nerve

Correct answer: (B) Sural nerve. Explanation: The Mayo block is a local block used commonly in surgeries involving the distal first metatarsal, specifically hallux valgus repair. A Mayo block is performed beginning in the space between the first and second metatarsals where the deep peroneal nerve and medial plantar nerve which is a branch of the tibial nerve are blocked. The saphenous nerve is then blocked. The sural nerve is not blocked as it provides sensory innervation to the lateral aspect of the lower leg and foot. Anesthesia on the lateral aspect of the foot and ankle is not needed for hallux valgus repair.

A patient tells her long-time healthcare provider that she recently signed a new will and instructed her estate to deliver a valuable coin collection to the healthcare provider after her death. Which of the following statements is correct? Since the proposed gift is not monetary, the healthcare provider may express gratitude. The healthcare provider must decline this valuable gift. The healthcare provider can accept the gift, but he/she should provide this patient with free medical care as compensation. The healthcare provider should not accept any gift of value from a patient due to risk of noncompliance with tax laws. The healthcare provider cannot accept a gift of value from this patient during her lifetime but may acknowledge a bequest, as it will occur after the patient is deceased.

Correct answer: (B) The healthcare provider must decline this valuable gift. Explanation: A healthcare provider should consider the potential implications of accepting a gift in light of the provider-patient relationship, including any patient expectations of the healthcare provider as a result of the gift. To demonstrate favorable treatment to a patient due to a patient gift may undermine the healthcare provider's objectivity when caring for the patient and can undermine the professionalism expected of the provider. In general, a healthcare provider ethically can accept a small gift as a token of appreciation, but acceptance of a valuable gift is problematic. The promise of a bequest may cause the healthcare provider consciously or unconsciously to alter treatment decisions in the care of a patient and may tarnish the healthcare provider's reputation by the suggestion of impropriety.

Which of the following procedures is best indicated for an underlapping fifth toe? A. Ruiz-Mora Procedure B. Thompson Procedure C. Post Procedure D. Lapidus Procedure

Correct answer: (B) Thompson Procedure. Explanation: The Thompson procedure is the only procedure listed above indicated for an underlapping fifth toe. It consists of a Z-type incision dorsally over the proximal phalanx, the resection of the fifth proximal phalanx in its entirety, reefing of the capsule to fill the dead space and corrective closure of the Z incision. (A) The Ruiz-More procedure is indicated for a cock-up fifth toe. It consists of an elliptical plantar incision, resection of the proximal phalanx and suturing of the incision to medial deviate and plantarflex the toe with closure. (C) The Post procedure is for correction of the rectus hammertoe. It consists of resection of the head of the proximal phalanx through either a linear or transverse elliptical incision and reefing of the capsule with closure. (D) The Lapidus procedure is indicated for severely overlapping fifth toes. It consists of a curvilinear incision over the dorm of the fifth tmp joint and proximal phalanx with a secondary more proximal incision over the extensor tendon. The extensor tendon is transected proximally and bought through the distal incision. The dorsal, medial and lateral metatarsophalangeal capsule are released. The extensor tendon is wrapped around the phalanx from dorsal medial to plantar lateral and attached to periosteum of the head of the fifth metatarsal, soft tissue or muscle proximal to the fifth metatarsal head.

A 61-year-old female patient presents 2 weeks after she was in a motor vehicle collision. At the time of the MVC, she did not seek medical attention. Today, she complains of weakness in her legs. On exam, the patient has no palpable contraction of ankle plantarflexors against gravity, but you observe fasciculations in the muscles tested. Before sending the patient to the emergency room for evaluation you want to finish your progress note. Which of the following would be the correct muscle strength for this patient? A. 3/5 B. 0/5 C. 1/5 D. 2/5

Correct answer: (C) 1/5. Explanation: The most commonly used scale for grading muscle strength is the Medical Research Council system. It tests the ability of the patient to move the muscle against resistance provided by the examiner. The patient's effort is graded on a scale of 0 to 5: 5 - Muscle contracts normally against full resistance. 4 - Muscle strength is reduced, but muscle contraction can still move joint against resistance. 3 - Muscle strength is further reduced, such that the joint can be moved only against gravity with the examiner's resistance completely removed. As an example, the elbow can be moved from full extension to full flexion starting with the arm hanging down at the side. 2 - Muscle can move only if the resistance of gravity is removed. As an example, the elbow can be fully flexed only if the arm is maintained in a horizontal plane. 1 - Only a trace or flicker of movement is seen or felt in the muscle, or fasciculations are observed in the muscle. 0 - No movement is observed. The patient in this case has fasciculations in the tested muscle but cannot actively move against gravity, giving a score of 1 on the five point scale, or 1/5.

A 43 year old male patient presents to your office with a chief complaint of feeling like toes are dragging when ambulating on the right foot. Upon further questioning, no trauma that patient can remember and it has been slowly getting worse over a period of a few years. Upon manual muscle testing, in dorsiflexion of the ankle, the patient has movement against gravity but not against your resistance. Which of the following would be the correct muscle strength for this patient? A. 5/5 B. 4/5 C. 3/5 D. 2/5

Correct answer: (C) 3/5. Explanation: This question is looking for you to know the manual muscle testing scale that is commonly employed in the physical examination. 5/5: Is normal strength. 4/5 is full range of motion with less than normal resistance. 3/5 is movement possible against gravity. 2/5 is movement possible with gravity eliminated. 1/5 is evidence of contraction but no joint motion. In this case since the patient has movement against gravity, the answer is 3/5.

An 81 year old male patient presents to emergency department with a an ulceration with purulent drainage right foot. The patient states that he has been feeling sick recently. He has been on oral antibiotics for the last week with little improvement. He states that he is currently on the list for both kidney and liver transplant due to severe disease. The patient is supposed to be going to dialysis three times per week but just hasn't been going. He states that he has both kidney and liver failure. On exam in ED temperature 103.8, BP 90/75, Pulse 151. In the ED, saline boluses for resuscitation elevated blood pressure minimally. Blood work drawn with WBC 31.2, Creatinine 9.5 and liver function tests are very elevated. X-rays taken show air like radiolucencies in the soft tissues. It is decided upon that patient needs to go for incision and drainage in operating room. What ASA classification status would this patient be? A. ASA PS 3 B. ASA PS 4 C. ASA PS 5 D. ASA PS 6

Correct answer: (C) ASA PS 5. Explanation: The American Society of Anesthesiologists (ASA) Physical Status classification system is the main system used by anesthesiologists to rate the degree of "sickness" a patient has before surgery. ASA 1 is a normal healthy patient excluding the very old and very young. ASA 2 is a patient with mild systemic disease (well controlled DM), pregnancy, mild obesity. ASA 3 is a patient with severe systemic disease. ASA 4 is a patient with severe systemic disease that is a constant threat to life. ASA 5 is a patient not expected to survive without the operation and ASA 6 is a brain dead patient whose organs are being harvested for donor purposes. In this case, the patient has sepsis syndrome with hemodynamic instability. He would thus be considered an ASA 5.

Your 62 year old patient has rheumatoid arthritis and is on multiple drug therapy including oral steroids and methotrexate. You are planning a pan metatarsal head resection and antibiotic prophylaxis pre-operatively. The best therapy for this patient is: A. Ciprofloxacin 1500 mg IV pre-operatively B. Ceftriaxone 1 gm IV pre-operatively C. Cefazolin 2 gm IV pre-operatively D. Vancomycin IV pre-operatively

Correct answer: (C) Cefazolin 2 gm IV pre-operatively. Explanation: Surgical prophylaxis is indicated in this patient due the immunocompromised state due to the steroid use and the methotrexate. Cefazolin is the drug of choice here. It has excellent anti-staphylococcal activity and also some gram negative coverage as well. It has the longest half life of the first generation cephalosporins so it will remain in a therapeutic state even in long cases. It is inexpensive. The dosage is one to two grams IV or IM prior to surgery and possible 1 gm after surgery for longer cases. Vancomycin is indicated in the presence of penicillin or cephalosporin allergy, the interaction of implants, and in scenarios where a high incidence of methicillin-resistant staphylococcus aureus is present. Ceftriaxone has the longest half life of the cephalosporins, but has a lack of anti-staphylococcal activity. Ciprofloxacin should not be used as its anti-staphylococcal activity is quite poor.

In type 1 diabetic mellitus undergoing surgery, what is the recommended regimen for their basal insulin? Bolus of short-acting hypoglycemic if the glucose levels are less than 200 mg/dL in the morning In emergency surgery give a bolus of short-acting hypoglycemic preoperatively Continue basal insulin at all times to avoid diabetic ketoacidosis in type 1 DM Continue metformin until morning of surgery for type 2 DM

Correct answer: (C) Continue basal insulin at all times to avoid diabetic ketoacidosis in type 1 DM. Explanation: Need basal insulin at all times to avoid diabetic ketoacidosis. The night before, continue usual dose of pm Glargine/NPH or a mixture. For insulin pump users, the usual overnight basal rate should be continued. The morning of the procedure, no boluses of short acting hypoglycemic should be given unless blood sugar is greater than 200 mg/dL and greater than 3 hours preoperatively. The insulin drip may be placed or the usual dose of glargine may be given if it is routinely taken in the morning. If the patient uses an insulin pump, the basal rate should be continued and D5 should be infused throughout the procedure. If the patient is on NPH or other insulin mixture, no short acting insulin should be given within 3-4 hours of the procedure, and instead half of the usual intermediate-acting insulin should be given, with D5 at a controlled rate throughout the surgery. If the procedure is done without continuous D5, no insulin should be given. And for emergency surgery the patient should not be given short-acting hypoglycemic preoperatively. The blood sugar should be monitored frequently through the surgery for every 30-60 minutes, and start insulin infusion if blood sugar is greater than 200 mg/dL. Metformin should be held 24 hours preoperatively and restarted 48 to 72 hours postoperatively, once normal renal function has been documented.

A 19-year-old pregnant woman at 30 weeks gestation is a Jehovah's Witness. She is involved in a motor-vehicle accident and requires a transfusion. The fetal monitor reveals distress, but the patient refuses transfusion or caesarean delivery. She is alert and understands that the potential consequences of her decision include fetal demise and her own death. Which of the following is the appropriate course of action? A. Obtain consent from father. B. Obtain court order for transfusion for health of fetus. C. Continue to treat patient while respecting her wishes. D. Transfer care to another provider if patient refuses lifesaving medical care. E. No consent is required since life of fetus is threatened.

Correct answer: (C) Continue to treat patient while respecting her wishes. Explanation: Since the patient is a competent adult, she cannot be transfused against her will, and she retains the right to refuse lifesaving treatment. Legally, the wishes of a pregnant woman must be respected, even when the pregnancy is in danger as a result of her medical decisions. (A) The father cannot give consent that supersedes the mother's right to refuse treatment. (B) (E) A fetus has no superseding legal rights prior to delivery. (D) Transfer of care could constitute abandonment and is inappropriate.

A 67-year-old male presents for a surgical consultation for hallux limitus. His X-ray is presented below. He does not wish to have a joint destructive procedure performed. Which of the following procedures is a joint-sparring option for hallux limitus? A. Jones tenosuspension B. Lengthening of the extensor hallucis longus C. Dorsiflexory osteotomy of the proximal phalanx D. Dorsiflexory osteotomy of the first metatarsal base

Correct answer: (C) Dorsiflexory osteotomy of the proximal phalanx. Explanation: A Jones tenosupsension and a dorsiflexory osteotomy of the first metatarsal base both function similarly from a biomechanics standpoint. Both cause an elevation of the first metatarsal which in turn will causing jamming of the phalanx on the metatarsal with dorsiflexion. These options therefore are counterproductive in the treatment of hallux limitus. Lengthening of the extensor hallucis longs tendon will only weaken active dorsiflexion of the metatarsophalangeal joint. Passive motion will still occur in response to the ground reactive forces and jamming of the joint will still occur. Correct reasoning will bring one to dorsiflexory osteotomy of the proximal phalanx also known as the Moberg osteotomy. The aim of the procedure is to move the limited arc of motion at the affected joint to a more dorsiflexed position so that the function is improved. It is applicable to patients with mild to moderate hallux limitus.

In a patient with CHF, diuretics, ACE inhibitors, and angiotensin II receptor blockers should be held the morning of surgery because of the following: A. Carry the risk of myositis B. Carry the risk of rhabdomyolysis C. Increase risk of hypotension during surgery D. Risk of bleeding

Correct answer: (C) Increase risk of hypotension during surgery. Explanation: Diuretics, ACE inhibitors, Angiotensin II receptor blockers should be held on the morning of surgery, especially if indication is CHF there is an increased risk of hypotension. (A) Cholesterol lowering drugs if not held one day prior to surgery, the patients carry the risk of rhabdomyolysis and myositis. (D) This is an incorrect choice as ACE; diuretics do not increase the risk of bleeding in patient undergoing non cardiac surgery.

Which mid shaft bunion procedure is the most unstable? A. Mau B. Scarf C. Ludloff D. Inverted Z

Correct answer: (C) Ludloff. Explanation: The Ludloff (oblique osteotomy of the first metatarsal in the transverse plane) is the least stable osteotomy of the group. It's osteotomy from dorsal proximal to plantar distal, while offering a very high degree of corruptibility, is inherently very unstable. The stability in this osteotomy is entirely from the internal fixation. (A) The Mau osteotomy has the exact opposite geometry as the Ludloff, but has far greater stability, but limited corrective potential. (B) The scarf osteotomy has a high degree of intrinsic stability due to its two opposing chevron cuts. (D) The inverted Z osteotomy, which is a modification of the scarf, also has great intrinsic stability.

Which of the following is true regarding spinal anesthesia? A. Commonly done with 18 gauge needle. B. Commonly given in the sitting position between S1 and S2. C. Marcaine is used more commonly than lidocaine. D. Can be used in patients with severe liver disease.

Correct answer: (C) Marcaine is used more commonly than lidocaine. Explanation: Spinal anesthesia as a whole is commonly completed using a small gauge needle- most commonly a 25 gauge. It is commonly given with the patient in the sitting position with the head and neck bent downwards. The anesthesiologist usually will palpate between L4 and L5 and give in that location. Most commonly marcaine (bupivicaine) is the drug of choice over lidocaine. Spinal anesthesia is not used in liver disease. This is primarily because the liver is responsible for making clotting factors and liver disease can lead to a decrease in clotting factors and an increased predisposition towards bleeding which is not good when bleeding occurs into the spinal column.

A patient presents with a history of surgical arthrodesis of the first metatarsophalangeal joint. The patient has difficulty in shoes due to the inability to dorsiflex the MTPJ. Which of the following would be an appropriate shoe modification? A. SACH heel B. Thomas heel C. Metatarsal bar D. Medial heel wedge

Correct answer: (C) Metatarsal bar. Explanation: A metatarsal bar is applied to the sole of a shoe with the most prominent aspect just proximal to the metatarsal heads. The bar tapers distally and proximally. This modification allows the foot to undergo the full gait cycle without requiring dorsiflexion of the forefoot. (A) The solid ankle custom heel (SACH) heel replaces the posterior wedge of the heel (base is plantar) with compressible foam rubber in order to simulate plantarflexion at heel strike. This does not accommodate for limited 1st MTPJ motion. (B) The Thomas heel flares the medial aspect of the standard heel approximately half an inch in order to reinforce the longitudinal medial arch. (D) A medial heel wedge would not be beneficial in accommodating lack of 1st MTPJ motion. Placing the hindfoot in a varus position would require more motion of the medial column in order to bring the first ray down to the ground.

Which one of the talocalcaneal coalitions is the most common? A. Anterior talocalcaneal coalition B. Posterior talocalcaneal coalition C. Middle talocalcaneal coalition D. The middle facet with the Anteater nose

Correct answer: (C) Middle talocalcaneal coalition. Explanation: The talocalcaneal coalition receives the most attention in the literature of all the tarsal coalitions. The coalition of the middle talocalcaneal joint region is the most frequently encountered of the three possible talocalcaneal coalitions, anterior middle and posterior. Anatomically the middle talocalcaneal coalition is either extra-articular, or intra-articular. It may either be complete, nearly complete, incomplete, or rudimentary. The anteater nose or sign refers to the presence of the calcaneonavicular coalition. It is best viewed in the medial oblique view, and can also be recognized in the lateral view alone as an extension or elongation of the anterior calcaneal process superiorly. The anterior and posterior coalitions are rare. The anterior medial talocalcaneal is identified in the medial oblique view.

A tendon transfer utilizing the EHB in the correction of hallux varus is the: A. Hawkins B. Johnson C. Myerson D. Valtin

Correct answer: (C) Myerson. Explanation: Myerson popularized the use of the EHB tendon (FAI) in the correction of hallux varus. As is true of all tendon transfers for treatment of this complication, the deformity must be flexible and the articulation free of DJD. He has shown that using this tendon is a stronger biomechanical construct than the EHL. The tendon is severed proximally at the musculotendinous junction, leaving the insertion intact. The tendon is passed beneath the scarred DTML from distal to proximal, and secured into the lateral side of the first metahead using a bone anchor, biotenodesis screw, or bone tunnel. (A) The Hawkins procedure utilizes the abductor hallucis tendon. It is severed from its insertion, then passed laterally beneath the DTML and secured to the lateral aspect of the proximal phalanx. A problem with this procedure is lack of adequate length of tendon. (B) Johnson popularized the use of the EHL to correct hallux varus. Originally, the EHL was transected at its insertion, passed from proximal to distal beneath the DTML and secured to the lateral/plantar aspect of the proximal phalangeal base. In addition to a lateral pull, plantarflexion is achieved. With complete detachment, an IPJ fusion is required. Thus, a modification is the split EHL transfer, where the EHL is split into a medial and lateral half, with the lateral half then transferred to the proximal phalanx. However, with this transfer, tension transferred to the medial half can theoretically alter its function. (D) Valtin described transfer of the first dorsal interosseous tendon to the proximal phalangeal base. It is not really popular; it is a small tendon and may not be strong enough to maintain reduction of the varus. Also, long-term effects on the second toe have not really been investigated.

The following sensations are lost with an effective local anesthetic blockade: proprioception, touch, pain, skeletal muscle tone and pain. Which option below has the above mentioned sensations in the correct order of sensation lost from beginning to end? A. skeletal muscle tone, proprioception, pain, touch, temperature B. touch, pain, temperature, proprioception, skeletal muscle tone C. pain, temperature, touch, proprioception, skeletal muscle tone D. pain, touch, temperature, proprioception, skeletal muscle tone

Correct answer: (C) Pain, temperature, touch, proprioception, skeletal muscle tone. Explanation: Peripheral nerves are divided into three types: A, B, and C. A nerve fibers are further divided into alpha, beta, gamma, delta. Alpha A fibers are the largest of the conductive fibers and are associated with motor function. A beta fibers are sensory fibers responsible for transmitting pain and temperature. A gamma fibers are associated with firing of muscle spindles that are part of joint proprioception. Lastly, A delta are the smallest of the fibers and conduct pain and light touch. The B fiber is the easiest and quickest fiber type to block in the body. The C fibers are unmyelinated, postganglionic nerve fibers that are the smallest in the entire body. They conduct pain, pressure and temperature sensation. Nerve fibers can vary in size from 0.5 to 2.0 microns. The larger the nerve fiber the more resistant the nerve fiber is to local anesthetic. As it pertains to the order of sensation lost, the order lost from local anesthetic block is first pain, followed by temperature, touch, proprioception and lastly skeletal muscle tone.

The patient pictured below presents for follow up on an interdigital soft corn. To date she has failed shoes with a wider toe box, interdigital padding and serial debridements. She is frustrated and requesting definitive treatment. Which of the surgical options below is best suited for definitive relief? Proximal interphalangeal joint arthroplasty of the third digit and distal interphalangeal arthroplasty of the fourth digit Proximal interphalangeal joint arthroplasty of the third digit Partial syndactilization Distal interphalangeal arthroplasty of the fourth digit

Correct answer: (C) Partial syndactilization. Explanation: Partial syndactilization is indicated in cases of recalcitrant soft corns. Once the skin in removed in the procedure there is no longer a possibility of a soft corn. The syndactilization is curative. (A) The proximal interphalangeal joint arthroplasty of the third digit and distal interphalangeal ar-throplasty of the fourth digit would help to straighten the digits and although might decompress the area, also might not provide any relief to the soft corn. (B) The proximal interphalangeal joint arthroplasty of the third digit would help straighten the digit and although might decompress the area, also might not provide any relief to the soft corn. (D) The distal interphalangeal arthroplasty of the fourth digit would help straighten the digit and although might decompress the area, also might not provide any relief to the soft corn.

According to Bojsen - Moller pronation of the calcaneocuboid joint allows the midtarsal joint to reach a packed position, resulting in maximum joint surface stability and congruence, therefore locking the midtarsal joint, muscles of the lower leg are also a big components of this locking mechanism. Which of the following muscles play a major role in locking the midtarsal joint in late midstance and propulsive period? A. Flexor digitorum brevis, peroneus brevis, and peroneus longus B. Posterior tibialis, flexor digitorium longus, peroneus longus C. Posterior tibialis, peroneus longus and peroneus brevis muscle D. Anterior tibialis, peroneus longus, and peroneus longus

Correct answer: (C) Posterior tibialis, peroneus longus and peroneus brevis muscle. Explanation: The locking mechanism of the MTJ is assisted in late stance and propulsive phases as the lesser tarsal bones are stabilized by the synergistic action of the posterior tibialis, the peroneus longus and brevis muscles. Plantar insertions of the posterior tibialis into the navicular, the three cuneiforms, the three central metatarsal bases, and the cuboid provide it with a posterior and medial stabilizing effect on the lesser tarsus. The peroneus longus, with its oblique insertion into the lateral aspect of the base of the fifth metatarsal, exerts a posterior and lateral stabilizing effect on the lesser tarsus. These tendons cross under the lesser tarsus and create broad stability when contracting with their posterior and cruciform action. The peroneus brevis with its insertion into the fifth metatarsal base assists the peroneus longus in providing lateral stabilization of the lesser tarsus.

During open reduction and internal fixation of fractures, neutralization plates are used to: A. permit functional use of the extremity during healing B. generate axial compression C. protect the primary lag fixation D. to treat infected fractures

Correct answer: (C) Protect the primary lag fixation. Explanation: Neutralization plates are used to protect the primary lag fixation. They protect against the deforming forces, which typically are rotation, torsion and shear. The stability and compression in fracture fixation comes from the lag screw fixation. They do not allow for an early or more rapid return to functional use. Additionally they do not provide any compression across the fracture. Dynamic compression plates can accomplish this task.

Mr. Johnson presents with a complaint of radiating pain along his medial ankle and into the arch of his right foot. He states the pain is constant and becomes more intense with ambulation and activity. Physical exam reveals intact pedal pulses. Tenderness on palpation along the retromalleolar area extending into the arch. Mr. Johnson reports these problems began about eight weeks ago. He also reports past surgical history of tarsal tunnel release on the same foot six months prior. He states that for the first few months after tarsal tunnel surgery his initial symptoms subsided. An MRI evaluation was performed and yielded findings consistent with post-surgical changes. An electrodiagnostic study was ordered on then right lower extremity. What is the most likely anticipated results of the electrodiagnostic study of the tibial nerve? A. Normal nerve conduction velocities; normal sensory and motor latencies B. Slowed nerve conduction velocities; normal sensory and motor latencies C. Slowed nerve conduction velocities; delayed sensory and motor latencies D. Normal nerve conduction velocities; delayed sensory and motor latencies

Correct answer: (C) Slowed nerve conduction velocities; delayed sensory and motor latencies. Explanation: Electrodiagnostic examination of the right tibial nerve would most likely yield slowed nerve conduction velocities. It was also demonstrated delayed sensory and motor latencies. The clinical scenario above describes the post-operative complication of extra-neural scarring of the tibial nerve. (A) Normal nerve conduction velocities would not be seen in the scenario described above. Furthermore, normal sensory and motor latencies would also not be seen. (B) Slowed nerve conduction velocities would be seen in the scenario described above. However, normal sensory and motor latencies would also not be seen. (D) Normal nerve conduction velocities would not be seen in the scenario described above. However, delayed sensory and motor latencies would be seen.

Which type of needle should be used for fragile tendon sheath, muscle or paratenon? A. Cutting B. Reverse Cutting C. Tapered D. Blunt

Correct answer: (C) Tapered. Explanation: Cutting needles have two opposing cutting surfaces along the needle point. These surfaces will cut a passage through dense connective tissues for suture material to follow. Reverse cutting needles are ideally suited for suturing fascia, ligaments, skin, tendon sheath, and oral mucosa. Taper point needles are sharp for initial tissue penetration and the smooth body moves tissues out of the needle's way rather than cutting them. This needle type is ideal for suturing fragile, regular and loose connective tissues such as tendon sheath, muscle, paratenon,blood vessels and fatty subcutaneous tissues. The blunt tipped needles are used to dissect rather than cut friable tissues such as that seen in the liver and kidney.

Regarding anticoagulation, "bridging" refers to: A. Substituting an antiplatelet drug with an anticoagulant B. Discontinuing aspirin therapy before surgery C. Temporary administration of a short-acting anticoagulant D. Utilization of a novel oral anticoagulant E. Administration of Vitamin K

Correct answer: (C) Temporary administration of a short-acting anticoagulant. Explanation: When anticoagulant therapy is stopped for surgery, temporary administration of a short-acting anticoagulant, usually heparin or its derivatives, to reduce the risk of a thrombotic event is done. This is referred to as "bridging."

Which joint is the key to stability of the midtarsal joint? A. The talonavicular joint B. The calcaneonavicular joint C. The calcaneocuboid joint D. The 1st tarsometarsal joint

Correct answer: (C) The calcaneocuboid joint. Explanation: Elftman contributed important work on the role of axis orientation in determining flexibility or stability of the midtarsal joint. The calcaneocuboid joint is the key to the midtarsal joint. He noted that they are fully attached; the cuboid and navicular function as a unit. He described two axes for the calcaneocuboid joint. The first lies within the calcaneus, is projected upward, and passes through the head of the talus. The second lies in the cuboid at a right angle to the first axis. The resultant axis of the two axes intersects the perpendicular line uniting both axes and runs just below the sustentaculum tali and talus. The resultant axis of the calcaneocuboid Joint as described by Elftman closely approximates the oblique axis described by Manter and Hicks: This model also describes two axes for the talonavIcular joint with a resultant axis approximating the longitudinal axis.

Which of the following is the most effective hemostatic agent in cutaneous surgery? A. Aluminum chloride, 20% B. Absorbable gelatin sponge C. Topical thrombin D. Epinephrine E. Hydrogen peroxide

Correct answer: (C) Topical thrombin. Explanation: Aluminum chloride denatures protein and can provide hemostasis in superficial wounds. Hydrogen peroxide provides only mild and very temporary hemostasis in very superficial wounds. Epinephrine provides hemostasis due to its vasoconstrictive effect and is useful in its combination with local anesthetic agents. Absorbable gelatin sponge provides a physical meshwork to facilitate blood clotting. Topical thrombin activates fibrinogen to form a fibrin clot that results in immediate coagulation.

To ensure reproducibility in weightbearing radiographs, each radiograph must be taken in the same angle and base of gait. What is meant by the base of gait? A. Angle formed between the feet and the line of progression during ambulation B. The distance between the heel bisections during ambulation C. The distance between both medial malleoli during ambulation D. The total angle formed between the medial borders of both feet

Correct answer: (C) The distance between both medial malleoli during ambulation. Explanation: The base of gait refers to the distance observed between the right and left medial malleoli during ambulation and is approximately 2 inches in the normal patient. The angle of gait refers to the angle formed between the feet and the line of progression during angulation. This line can easily be observed by having the patient ambulate a short distance prior to taking the X- ray. The usual angle of gait is between 10 and 15o.

Which of the following statements regarding distraction histogenesis is incorrect? Pain, swelling, toe contraction, excessive cutaneous tension, local pallor and hypothermia are all symptoms of an overly aggressive correction regimen. The standard rate of correction is 1mm every 24 hours. The latency period for callus distraction must be at most 4 days before distraction can be initiated. In order to avoid excessive tension, which causes tissue problems, it is necessary to distribute the total number of nut rotations in the 24-hour period over at least four sessions.

Correct answer: (C) The latency period for callus distraction must be at most 4 days before distraction can be initiated. Explanation: Distraction histogenesis is the process of applying continued, progressive tension across soft and hard tissue to cause the proliferation of tissue and elongate the tissue. If tension is applied too aggressively symptoms such as pain, swelling, toe contraction, excessive cutaneous tension, local pallor and hypothermia can occur. When distracting tissue, the most commonly used rate of correction is 1 mm of distraction per 24 hours. In order to minimize tissue problems the 1 mm of distortion must be divided over at least four sessions. Typically this requires 25% of one turn every 6 hours for a total of 1mm per 24 hours. In contrast, the latency period for callus distraction of bone is at minimum between 5-10 days depending on anatomic location.

A 59 year old female presents complaining of midfoot pain to her left foot. She states it has been present for over one year. The pain becomes worse the longer she is on her foot. Physical exam reveals intact neurovascular status. There is tenderness to the left midfoot, with a palpable bony ridge over the naviculocuneiform joint complex. Crepitus is noted with movement of the midfoot. The rearfoot is rigid. Patient relates history of left foot triple arthrodesis 12 years prior secondary to post traumatic arthritis from a previous calcaneal fracture. Radiographs confirm the presence of degenerative joint disease in the naviculocuneiform joint complex. What is the most likely cause of the midfoot arthritis? A. Gastrocnemius equinus B. Seronegative spondyloarthropathy C. The patient's previous triple arthrodesis D. Subacute midfoot instability

Correct answer: (C) The patients previous triple arthrodesis. Explanation: Hindfoot arthrodeses remove hindfoot motion and its shock absorbing function, which increases stress and motion demands in the articulations that surround the fusion. In time, the ligamentous and capsular stretching of these neighboring joints may occur, altering their contact loads and pressures, resulting in eventual cartilage degeneration. Stiffness of the foot brought on by a hindfoot arthrodesis procedure will cause midfoot osteoarthritis. In fact, following a triple arthrodesis, midfoot arthritis is exceedingly common, with reported rates up to 78%. Although gastrocnemius equinus does cause a tremendous amount of stress across the midfoot, with the presence of the triple arthrodesis it is unlikely the culprit. Seronegative arthropathies rarely first manifest in the midfoot.

What is the correct mechanism by which a variable pitch screw, as pictured below, provides compression? The larger diameter of the most proximal portion of the screw acts as a "head." The narrower pitched threads distally enters the bone faster than the trailing threads providing compression gradually as the screw is advanced further. The wider pitched threads distally enters the bone faster than the trailing threads providing compression gradually as the screw is advanced further. The distal threads engage the inner cortical wall and pulls the fragment proximally.

Correct answer: (C) The wider pitched threads distally enters the bone faster than the trailing threads providing compression gradually as the screw is advanced further. Explanation: This question focuses on the understanding of variable pitch compression screws. The mechanism by which a variable pitch compression screw provides compression across an osteotomy, arthrodesis, and/or fracture site is by the wider pitched threads distally entering the bone faster than the trailing threads providing gradual compression as the screw is advanced. There is no actual or functional "head" of this screw as suggested in the first answer. Furthermore, even when the distal threads would engage in the inner cortical wall the fact that the proximal portion is not statically set will not allow any compression to develop.

What is a potential complication of using an oscillating saw or rotary burr for joint preparation in metatarsocuneiform arthrodesis procedures? A. stress risers B. shortening of bone segment C. thermal necrosis D. damage to nearby structures

Correct answer: (C) Thermal necrosis. Explanation: Utilization of power equipment such as oscillation saws and/or rotary burs can cause thermal necrosis of arthrodesis sites. If the surgeon prefers the use of power equipment for preparation of fusion surface, the concomitant use of copious irrigation is highly recommended to reduce heat build up. For this reason, many surgeon prefer the use of hand instruments such as osteotomes, curettes, and rongeurs. Additionally, subchondral drilling and/or fish scaling would be performed to perforate the hard subchondral plate and facilitate fusion.

Which of the following is a disadvantage of a cohort study? A. It cannot evaluate multiple outcomes. B. They are not observational studies. C. They may be inefficient for studying rare diseases. D. They cannot discern temporal relationships between the exposure and the outcome.

Correct answer: (C) They may be inefficient for studying rare diseases. Explanation: A cohort study is a particular type of observational studies that expands upon a case series or cross-sectional study. They provide an estimate of the incidence of a disease or outcome. They also strengthen the evidence for causation by temporally dissociating the exposure from the outcome. However, they have several disadvantages. They are observational studies only, and therefore the outcomes observed may actually be associated with another factor than the one studied. On top of being inefficient for studying rare diseases, they are also poor studies for studying diseases with long latency periods.

What is a common occurrence following a hallux amputation in an otherwise intact foot? A. Ulceration at plantar first metatarsal head B. Metatarsus Primus Elevatus with resulting dorsal ulceration C. Transfer lesions to the lesser metatarsals D. Metatarsus adductus due to a lack of stability at the first metatarsal

Correct answer: (C) Transfer lesion to the lesser metatarsals. Explanation: Following amputation of a hallux at the level of the first metatarsophalangeal joint, the weight previously carried by the first metatarsal is transferred to the lesser metatarsals as an intact hallux acts as a buttress for the rotation of the first metatarsal head. The increased pressure on the lesser metatarsals, particularly the second metatarsal, increases the chances of ulcerations at the plantar metatarsal heads and stress fractures of the lesser metatarsals. When performing a hallux amputation, it is best to preserve the proximal one-centimeter of the proximal phalanx for biomechanical stability. The functions of the plantar fascia, intrinsic muscles, flexor hallux brevis, and the plantarflexion of the first metatarsal are partially preserved when the base is left intact.

A 10 year old boy presents with his mother complaining of intense pain in his great toe on his right foot. He says he was running through the house earlier in the day and stubbed his toe on the leg of a couch. On exam, his great toe is swollen and painful and there is diffuse ecchymosis in the digit. The nail plate is intact and there are no open lacerations or wounds on the foot. There is a subungal hematoma comprising approximately 40% of the nail. Xrays show no fractures. Which of the following treatment regimens would be most appropriate? A. Block the digit and avulse the nail to evaluate nail bed laceration B. Initiate antibiotics, block the digit and avulse the nail to evaluate nail bed laceration C. Trephine the nail to evacuate the hematoma D. Initiate antibiotics, trephine the nail to evacuate the hematoma

Correct answer: (C) Trephine the nail to evacuate the hematoma. Explanation: Traditionally, the treatment of subungal hematomas has centered around the idea that nail complete avulsion was indicated in any hematoma larger than 25% of the nail bed because of the increased risk of nail bed lacerations. Recent studies have challenged this notion. In patients with intact nail plate and margins and without phalanx fractures, outcome scores following simple trephination of the nail plate within 24-48 hours of injury were consistently high in the great majority of patients and complications were not predictable based on hematoma size, presence of fractures or infection. If the injury is over 48 hours old, the hematoma would most likely be coagulated and unresponsive to trephine and drainage. Antibiotic therapy is not widely used unless there is significant, contaminated soft tissue injury or other damage to the nail plate itself.

A 28 year old male patient presents to office with a chief complaints of heel pain which is worst in the AM when first waking up. States that he has been experiencing this pain for a few months and has been getting worse with time. The patient was diagnosed with plantar fasciitis and stretching exercises were discussed with the patient. An x-ray was taken to evaluate the heel and a radiolucent lesion was noted in the middle to anterior aspect of the of the calcaneus (extra- articular). The lesion was described as geographic in the radiologists report. In addition to plantar fasciitis, what other diagnosis should be discussed with the patient? A. Osteosarcoma B. Osteoid osteoma C. Unicameral Bone Cyst D. Enostosis

Correct answer: (C) Unicameral Bone Cyst. Explanation: The key words to pull out of this question are "radiolucent", "extra-articular" and "geographic appearance." The unicameral bone cyst is the only tumor listed that fits all of the above. Osteosarcoma is a malignant bone tumor that most frequently has a radiographic appearance described as mixed osteoblastic and osteolytic. It commonly has an aggressive periosteal reaction and is not described as being geographic in appearance. An osteoid osteoma is another bone tumor, although benign. The x-ray characteristics of this are typically lesions with an intracortical nidus. Typically the patient complains of pain that is resolved with salicylates. Lastly, an enostosis is a radiodense lesion and not radiolucent so this answer choice is completely incorrect.

A 25 year old female undergoing surgical correction of hypermobile first ray, with a Lapidus procedure. The past medical history is unremarkable, and she is only taking vitamin D supplement. She also denies use of oral contraceptives. What preoperative laboratory testing should be considered? A. Chemistry profile B. Coagulation profile C. Urine pregnancy test D. Renal and liver function tests

Correct answer: (C) Urine pregnancy test. Explanation: This is a 25 year old female undergoing correction of the hypermobile first ray with a Lapidus procedure (first tarsometatarsal arthrodesis). As standard of care, she is of childbearing age, urine pregnancy should be ordered. (A) If the patient has a history of hypertension, diuretic use, chronic obstructive pulmonary disease or obstructive lung disease, blood loss, renal disease, or chemotherapy, a chemistry profile should be ordered. (B) Is indicated if the patient is receiving anticoagulant therapy, has a family or personal history that suggests bleeding disorders, or has evidence of liver disease. (D) Renal and liver function tests are indicated for patients who have a medical condition or medication use that would serve as indications for the tests.

Thirty six hours after provisional external fixation of a highly comminuted, grossly contaminated first metatarsal fracture, the patient returns to the operating suite for a serial debridement a manipulation of the external fixator to provide a bit more compression. Out of the statements below which is true regarding the scenario just described? One to two more return trips to the operating room are to be expected until definitive closure and fixation can be obtained. The repeated debridement keeps the bioburden on the wound at a level that the body can regulate. 0.5 ml of tetanus toxoid should be administered post procedure. A first generation cephalosporin should be administered 24-48 hours after the procedure.

Correct answer: (D) A first generation cephalosporin should be administered 24-48 hours after the procedure. Explanation: When the wound is surgical manipulated, a fist generation cephalosporin is to be administered 24-48 hours afterward. If a patient has a penicillin allergy than Vancomycin can be used in place of the cephalosporin. (A) There is no validation to this statement. There is no way to predict the number of interventions that will be necessary to provide both definitive closure and fixation in the case of open fractures. (B) Even though debridement does reduce the bioburden, it is not adequate to replace the needs for antibiotics in the management of open fractures. (C) Repeated trips for manipulation of an open fracture have no bearing on the administration of tetanus prophylaxis.

Which of the following is not a structure that is released in the lateral release of the 1st interspace for hallux valgus repair? A. Adductor Hallucis Tendon B. Fibular sesamoidal ligament C. Fibular Sesamoid D. Abductor Hallucis Tendon

Correct answer: (D) Abductor Hallucis Tendon. Explanation: The lateral release is an important part of a bunion procedure to help relocate the sesamoids. The stepwise approach recommends first severing the adductor tendon, then the fibular sesamoidal ligament, then the lateral head of the flexor hallucis brevis. If there is still contracture after release of these, you can remove the fibular sesamoid. Hallux varus is a potential complication with excision of the fibular sesamoid. This should only be performed if the previous steps do not allow for adequate release.

The axis of the ankle joint is deviated from the transverse plane. What is the clinical significance of this deviation from the transverse plane? Allowing inversion and eversion of the ankle joint for walking on uneven terrain Allowing pronation and supination about the ankle joint to allow for compensation of an equinus deformity Allowing increased dorsiflexion and plantarflexion for greater propulsion Allowing for abduction/adduction coupled with dorsiflexion/plantarflexion allowing for absorption of rotational motions from the leg

Correct answer: (D) Allowing for abduction/adduction coupled with dorsiflexion/plantarflexion allowing for absorption of rotational motions from the leg. Explanation: Inman described the concept of the ankle joint axis being similar to that of a cone, rather than a cylinder, with the base of the cone directed towards the fibula (with a longer radius laterally rather than medially.) This radius difference cause larger displacement of the fibula, or lateral malleolus, than the medial malleolus when the foot is in closed kinetic chain. A motion described in open kinetic chain at a given joint creates an equal and opposite motion proximal to said joint in closed kinetic chain. Therefore, when the ankle is in closed kinetic chain, such as in midstance, ankle joint abduction/adduction is translated as internal and external rotation of the leg.

The tension band wiring technique would be an indicated fixation technique in which of the following? A. Dorsiflexory wedge osteotomy of the first metatarsal B. Akin Osteotomy C. Sesamoid fracture D. Avulsion fracture of the lateral malleolus

Correct answer: (D) Avulsion fracture of the lateral malleolus. Explanation: Tension band wiring is a preferred method of fixation for an avulsion fracture of the lateral malleolus. With this example of an avulsion fracture as well others the key point is that the fracture fragment is so small that it does not allow for compression screw fixation or tension band plating. Although cerclage wire can be used in the fixation of the dorsiflexory wedge osteotomy of the first metatarsal and akin osteotomy it is not a tension band technique. Tension band wiring fixation is neither indicated nor possible in fixating a sesamoid fracture. The critical requirement of the fracture complex is a definite mechanism of counter pull or resistance to the compressive forces of the tension band wire. This is not afforded in the sesamoid fracture.

What would be an appropriate biopsy technique the condition pictured below? A. Shave biopsy B. Scissor biopsy C. Punch biopsy D. Both A & B

Correct answer: (D) Both A & B. Explanation: Pictured above is an acrochordon on the anterior ankle. For this lesion both shave and scissor biopsies are indicated. The fact that these lesions have a narrow stalk makes them suitable for both scissor and shave biopsy. Neither the scissor nor the shave biopsy generally require anesthesia. Typically minimal if any bleeding is encountered and often a bandage is not necessary. The punch biopsy is the primary technique to obtain full thickness skin specimens. This lesion does not present deeper than the epidermis so a punch biopsy would have no merit in this clinical setting.

A 30 y/o patient presents with transfer metatarsalgia to the second-fourth metatarsals s/p HAV surgery. The surgery resulted in a shortened first metatarsal. The best surgical treatment option is: A. Hoffmann pan-metahead resection B. Weil osteotomy of 2-4 metatarsals C. Helal osteotomy of 2-4 metatarsals D. First metatarsal lengthening procedure

Correct answer: (D) First metatarsal lengthening procedure. Explanation: It is easier to bring one metatarsal out to length, than shortening three others to restore the parabola. Length can be restored via callus distraction, bone block interposition, or a first metatarsal step-cut, 'Z' type osteotomy. (A) The patient is young. A severe joint destructive procedure like a Hoffmann will not stand the test of time. Bony regrowth and continued metatarsalgia can be problematic. (B) Many may advocate this approach. However, pain and stiffness to the toes, as well as floating toes, have been reported. (C) Helal osteotomies have a high reported non-union rate (up to 40%), with continued pain.

Which of the following is contraindicated for a split tibialis anterior tendon transfer (STATT)? A. Recurrent clubfoot deformity B. Transmetatarsal amputation C. Spastic equinovarus D. Flexible pes planovalgus

Correct answer: (D) Flexible pes planovalgus. Explanation: The STATT procedure involves splitting the tibialis anterior tendon in half and anchoring the lateral half into the lateral cuneiform or cuboid. This decreases the varus pull of the tibialis anterior and restores a more balanced dorsiflexory pull of the foot. In flexible pes planovalgus, weakening the varus pull of the tibialis anterior will further exaggerate the pronatory deformity and is not an indication for STATT. (A) STATT procedures are indicated for equinovarus deformities and may be considered for recurrent clubfoot deformities. Anchoring the lateral half of the tibialis anterior tendon into the lateral column will decrease the varus pull in clubfoot deformity. (B) Transmetatarsal amputation often leads to equinovarus deformity due to the unopposed gastrocnemius, tibialis anterior and tibialis posterior with the EHL and EDL. This is addressed with the STATT procedure to prevent a resultant equinovarus deformity following amputation. (C) Spastic equinovarus is a common indication for STATT to decrease the varus pull of the overpowering tibialis anterior. In rigid deformities this soft tissue procedure should not be performed alone for adequate correction.

Which of the following procedures is best indicated for the severely overlapping fifth toe pictured below? A. Ruiz-Mora Procedure B. Thompson Procedure C. Post Procedure D. Lapidus Procedure

Correct answer: (D) Lapidus Procedure. Explanation: The Lapidus procedure is the only procedure listed above indicated for severely overlapping fifth toes. It consists of a curvilinear incision over the dorsum of the fifth MTP joint and proximal phalanx with a secondary more proximal incision over the extensor tendon. The extensor tendon is transacted proximally and brought through the distal incision. The dorsal, medial and lateral tmp capsule are released. The extensor tendon is wrapped around the phalanx from dorsal medial to plantar lateral and attached to periosteum of the head of the fifth metatarsal, soft tissue or muscle proximal to the fifth metatarsal head. The Ruiz-More procedure is indicated for a cock-up fifth toe. It consists of an elliptical plantar incision, resection of the proximal phalanx and suturing of the incision to medial deviate and plantar flex the toe with closure. The Thompson procedure is indicated for an underlapping fifth toe. It consists of a Z- type incision dorsally over the proximal phalanx, the resection of the fifth proximal phalanx in its entirety, reefing of the capsule to fill the dead space and corrective closure of the Z incision. The Post procedure is for correction of the rectus hammertoe. It consists of resection of the head of the proximal phalanx through either a linear or transverse elliptical incision and reefing of the capsule with closure.

Which of the following is not a perioperative concern for patients with rheumatoid arthritis? A. Poor bone quality B. Glucocorticoid regimen C. Neck or jaw pathology D. Pain medication regimen

Correct answer: (D) Pain medication regimen. Explanation: Many patients with RA take pain medication for chronic pain. These medications have not been shown to interfere in the perioperative period with healing. Patients taking NSAIDs along with pain medication would be required to stop the NSAIDs perioperatively to prevent bleeding complications. A concern in the perioperative setting in patients with arthritis, especially those with RA, is poor bone quality. Many RA patients are receiving glucocorticoids before surgery. The dosage should be reduced to the lowest possible maintenance dose in the perioperative setting to reduce the risk of postoperative infection and wound healing. Patients with arthritis should also be assessed for problems with the neck and jaw, which could interfere with intubation. Flexion radiographs of the neck may show atlantoaxial subluxation, especially in patients with RA and ankylosing spondylitis.

The axial calcaneal projection simply isolates and images the calcaneus. The patient stands on the film cassette in the angle and base of gait and they lean slightly forward. The tube is angled from 25o from vertical. What anatomic structure is targeted by the X-ray beam? A. inferior aspect of the calcaneus B. posterior ankle joint C. tibial plafond D. posterior talocalcaneal joint

Correct answer: (D) Posterior talocalcaneal joint. Explanation: The calcaneal axial projection is a specialized projection used in podiatry. It is useful for identifying calcaneal alignment and structural problems within the bone itself. The patient stands on the film cassette in the angle and base of gait while simultaneously leaning slightly forward. The tube is angled from 25o and targeted at the posterior talocalcaneal joint.

Which of the following is the least likely arthritic process to occur in a monoarticular pattern? A. Septic arthritis B. Post-traumatic arthritis C. Gouty arthritis D. Rheumatoid arthritis

Correct answer: (D) Rheumatoid arthritis. Explanation: Multiple joint involvement is commonly seen in inflammatory arthritic processes such as rheumatoid arthritis, psoriatic arthritis, ankylosing spondylitis and reactive arthritis. Monoarticular involvement is typically seen in infective arthritis, traumatic arthritis, crystal deposition arthritis, neuropathic arthritis and osteoarthritis.

Which of the following is the best diagnostic test used to diagnose Malignant Hyperthermia? A. CBC B. EMG/NCS C. Serum creatinine D. Skeletal muscle biopsy

Correct answer: (D) Skeletal muscle biopsy. Explanation: Along with preoperative screening for family or personal history of malignant hyperthermia, skeletal muscle biopsy is the best definitive test of diagnosis of malignant hyperthermia. Malignant hyperthermia is a potentially life-threatening, but rare disorder characterized by extremely high fever, muscle rigidity, cardiac arrhythmias, and acidosis. It may be precipitated by inhaled anesthetics such as halothane, both depolarizing and nondepolarizing neuromuscular blocking agents.

Shown below is a radiograph of a 57 year old male who fell six feet and presented to the emergency room two hours after injury. Which diagnostic test listed below is the most important to order? A. Axial radiograph of the foot B. Computed tomography scan of the foot C. Magnetic resonance imaging of the foot D. Spine radiograph

Correct answer: (D) Spine radiograph. Explanation: Because the majority of these fractures results from falls from a height, a full body assessment is paramount. Approximately 10% of these patients have an associated fracture of the spine, with L1 being the most frequently involved. Avulsion fractures of the posterior calcaneus generally occur in men older than 50, whereas Achilles tendon ruptures typically occur in men between 30 and 50 years old. Concomitant injury of the extremities, such as the ankle, femur and wrist occur approximately 26% of the time.

What is the primary mechanism of establishing hemostasis of the surgical wound during and after hallux abducto valgus surgery? A. The use of local anesthetic B. The use of tourniquet C. The exsanguination of the leg with elevation and application of an esmarch D. The anatomic dissection technique

Correct answer: (D) The anatomic dissection technique. Explanation: The underlying concept of anatomic dissection is the preservation of blood supply to the soft tissues during surgery. The incision placement is oriented parallel to and between any critical structures. The dissection of the subcutaneous tissues is the fundamental basis of anatomic dissection. The superficial soft tissues must be reflected In a manner that preserves the skin and subcutaneous layers and avoid laminar contact of these two layers and the linear structures they contain. This approach not only maintains the viability of healing potential of these tissues, but it also serves as the primary mechanism of establishing hemostasis of the surgical wound, whether the procedure is performed with or without tourniquet. The primary effective way to control bleeding during and after surgery is most effectively accomplished with surgical dissection technique. This is key to reduce bleeding, hematoma, and edema. In addition to improved soft tissue healing, tissue plane dissection maintains a distinct separation of the superficial and deep tissues even after the healing process has occurred.

In regards to using polymethylmethacrylate (PMMA) as a local antibiotic delivery vehicle which statement below is correct? The endothermic reaction that occurs as the PMMA sets up limits the choice of antibiotics that can be used. The elution profile is too short to make it a viable choice in most clinical scenarios. Often left in surgical site long term to provided sustained antibiotic elution. The exothermic reaction that occurs as the PMMA sets up limits the choice of antibiotics that can be used.

Correct answer: (D) The exothermic reaction that occurs as the PMMA sets up limits the choice of antibiotics that can be used. Explanation: Polymethylmethacrylate (PMMA) is a commonly used local antibiotic delivery modality. It produces a highly exothermic reaction that can cause denaturing of heat-labile antibiotics. Only certain antibiotics are heat-stabile enough to withstand this reaction. Therefore the exothermic reaction limits the choice of antibiotics that can be used. On the other hand, PMMA has an excellent long term elution profile. It is great for open injuries and for larger, staged procedures. PMMA is nearly always temporary placed and removal is necessary.

Which of the following statements regarding lag screws is correct? A. Cortical screws come in both full and partially threaded versions. B. A lag screw purchases both the proximal and distal cortices. C. Cancellous screws have a more narrow pitch. D. The proximal fragment of bone must be overdrilled.

Correct answer: (D) The fragment of bone must be overdrilled. Explanation: In order for the screw to glide through the proximal portion of the bone the proximal fragment must be overdrilled. The screw must pass freely and not engage the proximal cortex and only engaged the distal cortex. This will allow the screw to pull the distal fragment toward the proximal fragment as the screw head engages the proximal cortex. In regards to pitch of screw, it is the cortical screw that has the more narrow pitch to optimize the amount of thinner cortical bone that is purchased and cancellous screws have a wider pitch to grab more bone. Regarding the first statement only cancellous screws come in fully and partially threaded versions. Cortical screws only come fully threaded.

In a weight-bearing radiographic study, what is the purpose of using angle and base of gait? To visualize positional relationships of the foot bones during toe off To visualize the degree of abduction and adduction of the foot from the midline To see the separation or the distance between both heels during the gait cycle To standardize the weight-bearing radiographic technique in order to visualize positional relationships of the foot bones

Correct answer: (D) To standardize the weight-bearing radiographic technique in order to visualize positional relationships of the foot bones. Explanation: In a weight-bearing radiographic study, the purpose of using angle and base of gait is to standardize the weight-bearing radiographic technique in order to visualize positional relationships of the foot bones. (B) Base is known as angle of gait only. (C) is also known as base of gait.

This is a resistance of a solid body to bend or defect by an applied force: A. Modulus elasticity B. Passivation C. Fatigue strength D. Shear E. Stiffness

Correct answer: (E) Stiffness. Explanation: Stiffness is resistance of a solid body to bend or defect by an applied force. Modulus elasticity is a measure of stiffness of a substance along its elastic range. It is calculated by dividing stress load by the amount of strain or deformation. Passivation is the corrosion resistance by a surface of reaction products. This layer is normally an oxide layer, which is impervious to gas and water. Shear is the mechanical term, for which there is a parallel deformation of internal surfaces of a material.

A patient presents to the emergency department with severe hypothyroidism with depressed mental status but no pericardial effusion or heart failure. The patient presents with necrotizing fasciitis of the left foot, which is rapidly progressing. You decide that a fasciotomy with incision and drainage will be required. What is the next step? (Select all that apply) A. Give Propylthiorouracil to inhibit peripheral conversion of T4 to T3. B. Intravenous levothyroxine 200-500 mg intravenously given over 30 minutes. C. Daily dose of levothyroxine 50-100 mg IV postoperatively. D. Dextrose should be avoided for risk of fluid overload. E. Glucocorticoids concurrently with IV levothyroxine.

E - Glucocorticoids concurrently with IV levothyroxine.,C - Daily dose of levothyroxine 50-100 mg IV postoperatively.,B - Intravenous levothyroxine 200-500 mg intravenously given over 30 minutes.. Correct answer: (B) (C) (E) Explanation: Intraoperative fluids should include dextrose with normal saline. Controlled ventilation is recommended as these patients are at risk for hypoventilation. In patients presenting with severe hypothyroidism with an emergent condition in need of prompt surgical intervention, intravenous levothyroxine 200-500 mg should be given over 30 minutes; this is followed by a daily dose of 50-100 mg intravenously. As many patients with hypothyroidism also have adrenal insufficiency, glucocorticoids should be administered.

Which type of calcaneal fracture would most likely be treated with closed reduction and immobilization? A. Fracture of the anterior process of the calcaneus B. Fracture of the posterior beak of the calcaneus C. Achilles tendon avulsion fracture of the calcaneus D. Intra-articular fracture involving the subtalar joint

Which type of calcaneal fracture would most likely be treated with closed reduction and immobilization? 79% of exam takers got this question correct. Explanation: Correct answer: (A) Fracture of the anterior process of the calcaneus. Explanation: A fracture of the anterior process of the calcaneus is a Rowe Type IC fracture. Rowe 1A is a fracture of the medial tuberosity and type B is a fracture of the sustentaculum tali. Due to the stability of these fractures, Row Type I fractures are often treated conservatively with closed reduction and immobilization. (B) A fracture of the posterior beak of the calcaneus is a Rowe Type IIA fracture. Whereas Type I fractures are often treated conservatively, Rowe II, III and IV are often treated surgically. (C) An Achilles tendon avulsion fracture of the calcaneus is a Rowe Type IIB fracture. Rowe II, III and IV are often treated surgically. ORIF principles may be utilized to fixate the avulsed segment in order to stabilize the Achilles tendon. (D) An intra-articular fracture involving the subtalar joint is a Rowe Type IV fracture and often requires intervention, often with internal or external fixation.


Related study sets

NU 300 NCLEX challenge questions

View Set